Sie sind auf Seite 1von 158

Instructor's Manual If Test Bank to accompany

INTRODUCTION TO

General, Organic & Biochemistry


Fifth Edition
and

INTRODUCTION TO

Organic & Biochemistry


Third Edition

Frederick BeHelheim
Jerry March
Adelphi University

SAUNDERS GOLDEN SUNBURST SERIES


Saunders College Publishing
Harcourt Brace College Publishers

Fort Worth Philadelphia San Diego New York Orlando Austin


San Antonio Toronto Montreal London Sydney Tokyo
Copyright © 1998, 1995, 1991, 1988 by Saunders College Publishing

All rights reserved. No part of this publication may be reproduced or


transmitted in any form or by any means, electronic or mechanical,
including photocopy, recording, or any information storage and retrieval
system, without permission in writing from the publisher.

Although for mechanical reasons all pages in this publication are perforated,
only those imprinted with Saunders College Publishing are intended for
removal.

Requests for permission to make copies of any part of the work should be
mailed to: Permissions Department, Harcourt Brace & Company, 6277 Sea
Harbor Drive, Orlando, Florida 32887-6777.

Printed in the United States of America

Bettelheim: Instructor's Manual and Test Bank to accompany Introduction


to Organic and Biochemistry. Third Edition, and Introduction to General
Organic and Biochemistry Fifth Edition.

0-03-020433-X

567 021 7654321

..
CONTENTS

Introduction . . . . . . . . . . . . . . . . . . . . . . . . . . . . . . . . . . . . . . . . . . . . . . . . . . . . 1

Concordance of the Two Books . . . . . . . . . . . . . . . . . . . . . . . . . . . . . . . . . . . . . 4

Suggested Course OutliIles . . . . . . . . . . . . . . . . . . . . . . . . . . . . . . . . . . . ...... 5

Examination Questions. .. . . . . ................................... 13


.

Answers to Examination Questions . . . . . . . . . . . . . . . . . . . . . . . . . . . . . . . . . 99

Answers to Even-Numbered End-of-Chapter Problems . . . . . . . . . . . . . . . . . 103


INTRODUCTION

The purpose of this manual is to provide instructors with material that will help
them when they teach courses that use our textbooks Introduction to General, Organic,
and Biochemistry, fifth edition, and Introduction to Organic and Biochemistry, third
edition. Because 17 of the 19 chapters in the latter (shorter) book are virtually identical
t o corresponding chapters in the former (longer) book, we have combined the two
Instructor's Manuals/Test Banks into one. A table of concordances between the chapters
in the two books is on page 4.
The manual is divided into three parts:

a. organization of the courses


b. examination questions, arranged by chapter, with answers
c. answers to the even-numbered end-of-chapter problems

Organization of the courses:

three fields of
The iOllger book: All the essential topics needed to understand the
chemistry are covered in the 26 chapters of the text. The material is so distributed that
the emphasis is on both general chemistry and biochemistry. The former is dealt with in
the first nine chapters. The last eleven chapters are devoted to biochemistry. The
remaining six chapters cover those topics of organic chemistry that are needed to
understand biochemistry.
Most of the material can be covered in a three-quarter or two-semester course. A
good deal of it can be covered in a two-quarter course, but selective omissions are
necessary. In the schedules provided in this Manual, we assume that a three-quarter
course will have 28 50-minute sessions in each quarter, for a total of 84 sessions. For a
two semester course we assume three 50-minute s essions per week for 14 weeks plus a
final-exam session, or 43 sessions per semester, for a total of 86 sessions. For a two­
quarter course, we assume four 50-minute sessions per week for a total of 80 sessions.
The shorter book: The material is so distributed that somewhat more emphasis
is on biochemistry than on organic chemistry. The first eight chapters cover organic
chemistry, including bonding and acids and bases. The last eleven chapters are devoted
to biochemistry.
Most of the material can be covered in a two-quarter or one-semester course. A
good deal of it can be covered in a one-quarter course, but selective omissions are
necessary. In the schedules provided in this Manual, we assume that a two-quarter course
will have 28 50-minute sessions in each quarter, for a total of 56 sessions. For a one
semester course we assume three 50-minute sessions per week for 14 weeks plus a final­
exam session, or 43 sessions.

1
© Saunders CoUege Publishing
Both books: The schedules we recommend cover as much material as is possible
in the time-frame given. Some instructors may prefer a more leisurely approach, in which
case they can spend more time on individual topics that they find more important and
interesting, and so cover less material, but in more depth. Such selection is governed not
only by the preferences of the individual instructor, but also by the circumstances of
class size, student background, the need for recitation sections, etc.

Examination questions:

We provide examination questions for all 26 chapters in the larger book and all
19 chapters in the shorter book. Instructors may choose whichever questions they feel
are most suitable, from among the wide choice given. Included among them are questions
on the boxes in the text, so that instructors who assign particular boxes will have
examination questions for them. The questions are ananged in the order that they appear
in the larger book, with questions pertaining to Chapters 1 and 2 of the shorter book
given at the end (pp. 92-97). The sequence of the remaining 17 chapters of the shorter
book is the same as that of chapters 1 1 to 26 of the longer book, except that chapter 6
of the shorter book (conesponding to Chapter 8 of the longer one) is out of this
sequence (see the Table on p. 4).
The examination questions are all of the multiple choice variety. This is the
standard way of testing in colleges and universities with large classes. Answers to all of
the examination questions are given, beginning on page 105.
The publisher of this book has agreed to waive the copyright on the examination
questions provided in this Manual, as long as they are used for testing purposes only.
Therefore, the instructor may photocopy or otherwise use these questions without
violating copyright laws.

Answers to the problems:

The answers to the in-chapter problems and to the odd-numbered end-of-chapter


problems are given in the textbooks. Answers to the even-numbered end-of-chapter
problems are included in this Manual and in the Study Guides. For those chapters that
are virtually identical in both books, the numbering given for these answers is that of the
longer book, as is the sequence. Users of the shorter book will find those chapter
numbers are also given at the head of the group of problems for each chapter. Also, the
table giving the concordance of chapters between the two books (on p. 4) may be
consulted.

© Saunders College Publishing 2


We sincerely hope that you will find our textbooks helpful in instructing your
students in chemistry, and that your students will like them also. We tried to provide an

easy, informal style for readability and to emphasize the potential applications of the
principles taught by including a large number of boxes. The boxes can be assigned or
skipped by the instructor, but simply because they are there they pro vide additional
reading material to satisfy the curiosity of the students.
We hope that, after using our text, you will take the time to let us know your
opinion of it: its strengths and weaknesses, and to suggest improvements wh ere you feel
they are needed.
We would like to thank David March for his invaluable help in making production
of this Manual possible.

Frederick A. Bettelheim

Jerry March

3
© Saunders CoUege Publishing
CONCORDANCE OF THE TWO BOOKS

Chapter Topic Number in Smaller Book Number in Larger Book

Organic Chemistry, Bonding 1


Alkanes 2
Alkenes, Alkynes, Aromatics 3 11
Alcohols, Phenols, Ethers, Halides 4 12
Aldehydes, Ketones 5 13
Acids and Bases 6 8
Carboxylic Acids, Esters 7 14
Amines, Amides 8 15
Carbohydrates 9 16
Lipids 10 17
Proteins 11 18
Enzymes 12 19
Bioenergetics 13 20
Specific Catabolic Pathways 14 21
Biosynthetic Pathways 15 22
Nucleic Acids, Protein Synthesis 16 23
Chemical Communication 17 24
Body Fluids 18 25
Nutrition, Digestion 19 26

© Saunders College Publishing 4


SUGGESTED COURSE OUTLINE

for Introduction to General, Organic, and Biochemistry

SESSION'"
3Q 2S 2Q SECTIONS
1 st 1 st 1st TOPICS IN
� �Q. TEXT
1 1 1 Scientific method, measurements 1 . 1 - 1 .4
lA-1B
2 2 2 Unit conversions, states of matter 1 .5- 1 .6
3 3 3 Density, energy, heat 1 .7- 1 .9
lC-lD
4 4 4 Classification of matter, atomic theory, inside the atom 2. 1 -2.4
2A-2B
5 5 5 Periodic table, electronic structure 2.5-2.6
2C-2D
6 6 6 Electronic configuration and periodic table, 2.7-2.8
size of atoms 2E
7 First hour exam
8 7 7 Ions, ionic bonding 3. 1 -3.3
3A-3B
9 8 8 Covalent bonding, shapes of molecules, 3.4-3.7
electronegativity 3C-3D
10 9 9 Factors in bonding, polyatomic ions, 3.8-3. 1 1
inorganic nomendature
- 10 10 First hour exam
11 11 1 1 Formula weight, moles, equation balancing 4. 1 - 4.4
12 12 12 Stoichiometry, percent yield, ionic reactions 4.5-4.7
13 13 13 Oxidation-reduction, heat of reaction 4.8-4.9
4A-4D
14 Second hour exam
15 14 14 Organization of matter, gases, 5. 1 -5.4
pressure, gas laws 5A-5E
16 1 5 15 Gas laws, intermolecular forces 5.5-5.7
5F

*The first column (3Q) lists sessions in a three-quarter course (28 sessions per quarter). The
second column (2 S) lists sessions in a two-semester course (43 per semester). The third
column (2Q) lists sessions in a two-quarter course (40 per quarter).

5
© Saunders College Publ�hing
3Q 28 2Q
17 16 16 Liquids, vapor pressure, 5.8-5. 1 1
solids, phase changes 5G-5H
18 17 17 Types of solutions, solubility, 6. 1 -6.5
concentration units 6A-6B
19 1 8 18 Concentration units, water as a solvent 6.5-6.6
6C-6D
20 19 19 Colloids, colligative properties, 6.7-6. 1 0
osmosis, dialysis 6E-6F
21 20 20 Second or third hour exam
22 2 1 21 Kinetics, molecular collisions, activation energy 7.1 -7.3
23 22 22 Factors affecting rates, equilibrium 7.4-7.5
7A-7C
24 23 23 Equilibrium constants, 7.6-7.8
Le Chatelier's principle 7D-7E
25 24 24 Acid and base strength, 8. 1 -8.4
Bf0nsted-Lowry theory, Ka 8A
26 25 25 Properties of acids, 8.5-8.8
Kw, pH 8B-8D
27 26 26 Hydrolysis, buffers, titration, normality 8.9-8. 1 1
8E-8F
28 Final exam
2n d
J2.
1 27 27 Radioactivity, natural transmutation 9. 1 -9.2
9A
2 28 28 Half-life, characteristics of radiation, 9.3-9.5
dosimetry and health 9B-9D
3 29 - Medical uses of radiation, 9.6-9.8
nuclear fusion, fission 9E-9G
30 29 Third hour exam
4 31 30 Organic compounds, hybrid orbitals, hydrocarbons 1 0. 1 - 1 0.5
lOA
5 32 31 Alkanes, lUI' AC system, cycloalkanes, 1 0.6- 1 0.9
cis-trans isomers lOB
6 33 32 Alkanes: physical and chemical properties, 1 0. 1 0- 1 0. 1 2
functional groups 1 0C- IOG
7 First hour exam

6
C Saunders CoDege Publishing
3Q 2S 2Q

8 34 33 Alkenes: nomenclature, 1t orbitals, 1 1 . 1 - 1 1 .4


cis-trans isomers 1 1 A- 1 1 C
9 35 34 Alkenes: physical and chemical properties, 1 1 .5- 1 1 .7
addition polymers 1 1 D- 1 1 F
10 36 35 Alkynes, aromatic hydrocarbons: structure, nomenclature 11.8-11.10
11 37 36 A romatic substitution, 1 1 .1 1-1 1.13
fused and heterocyclic rings 1 1G
12 38 37 Alcohols: names and reactions 1 2. 1 - 1 2.3
13 39 38 Important alcohols, phenols 1 2.4- 1 2.5
1 2A- 1 2D
14 40 39 Ethers, physical properties of 1 2.6-12.7
alcohols, phenols, ethers 1 2E- 1 2G
41 Sulfur compounds, halides 1 2.8-12.9
1 2H- 1 2I
42 -
Review
15 Second hour exam
43 40 Final Exam
2nd 2nd
.-S...�
16 1 1 Aldehydes, ketones: naming, physical properties, preparation 1 3. 1 - 1 3.4
17 2 2 Aldehydes, ketones: chemical properties, acetals, 1 3.5-13.7
important aldehydes and ketones 13A-13D
18 3 3 Carboxylic acids 1 4. 1 - 1 4.6
1 4A
19 4 4 Carboxylate salts, carboxylic esters 1 4.7- 1 4.9
1 4B-14C
20 5 5 Carboxylic ester preparation and hydrolysis, 1 4. 1 0- 1 4. 1 2
phosphates 14D-14F
21 Third hour exam
6 Amines 1 5. 1 - 1 5.4
1 5A-15B
7 Amide nomenclature, properties, 1 5.5-15.7
preparation 15C-15E
8 Amide hydrolysis, alkaloids 1 5.8- 1 5.9
1 5F- 1 5G
22 6 Amines: nomenclature, basicity 1 5. 1 - 1 5.2

7
© Saunders CoUege Publishing
15.4, 15A
3Q 28 2Q

23 7 Amides 15.5.;..15.8
ISC-15F
9 8 First hour exam
24 10 9 Optical activity, chirality 16.1-16.4
25 11 10 Monosaccharide:; 16.5-16.8
16A
26 12 11 Monosaccharide derivatives, 16.9-16.11
di and polysaccharides 16B-16D
27 13 12 Acidic polysaccharides, 16.12-16.13
chiral compounds in biological organisms 16E-16F
28 Final Exam
3rd
...Q..
1 14 13 Fats: structure and properties 17.1-17.3
17A-17D
2 15 14 Complex lipids, membranes, glycerophospholipids 17.4-17.8
sphingolipids, glycolipids 17E-17F
3 16 15 Steroids, bile salts, prostaglandins, 17.9-17.12
leukotrienes 17G-171
4 17 16 Protein functions, amino acids, zwitterions 18.1-18.4
5 18 17 Proteins: primary and secondary structure 18.5-18.8
18A-18E
6 19 18 Proteins: higher structure, 18.9-18.11
denaturation, glycoproteins 18F-18H
7 20 19 First or second hour exam
8 21 20 Enzymes: names, classifications, 19.1-19.3
common terms 19A-19B
9 22 21 Factors affecting enzyme activity, 19.4-19.5
mechanism of enzyme action 19C-19E
10 23 22 Enzyme regulation, 19.6-19.7
enzymes in medicine 19F
11 24 23 Bioenergetics, cells and mitochondria, 20.1-20.3
the common catabolic pathway 20A
12 25 2 4 Citric acid cycle, electron and H+ transport 20.4-20.5
13 26 25 Phosphorylation, chemiosmotic pump, 20.6-20.8
energy yield, energy conversions 20B-20C

8
© Saunders College Publishing
14 Second hour exam

3Q 2S 2Q
15 27 26 Specific pathways, glycolysis 21.1-21.2
21A
16 28 27 Energy yield from glucose, p -oxidation, 21.3-21.7
energy yield from stearic acid, ketone bodies 21B-21C
17 29 28 Catab olism of amino acids and heme 21.8-21.10
21D-21G
18 30 29 Biosynthesis of carb ohydrates and 22.1-22.3
fatty acids 22A
31 Biosynthesis of membrane lipids 22.4-22.5
and amino acids 22B-22C
32 30 Third hour exam
19 33 31 Nucleic acids, DNA and RNA structure 23.1-23.3
23A
20 34 32 DNA replication, types of RNA, transcription 23.4-23.7
21 35 3 3 Translation and protein synthesis, 23.8-23.10
genetic code 23B-23C
22 3 6 34 Genes, exons, introns, gene regulation, 23.11-23.14
mutations, recomb inant DNA 23D-23I
23 Third hour exam
24 37 35 Neurotransmitters and hormones 24.1-24.6
24A-24G
25 38 3 6 Body fluids, composition and functions of blood, 25.1-25.5
blood as carrier of O2 and CO2, blood butTers 25A-25B
26 39 37 The kidney, water and salt balance in blood and kidneys, 25.6-25.9
b lood pressure 25C-25E
40 38 Immunoglobulins 25.10
25F
27 41 39 Nutrition, calories, diet, digestion 26.1-26.10
26A-26C
42 -
Review
28 43 40 Final exam

C Saunders CoUege Publishing 9


SUGGESTED COURSE OUTLINE
for In troduction to Organic and Biochemistry

SESSION'"
2Q IS SECTIONS
1 st TOPICS IN
JL TEXT
1 Chemical bonds, shapes of molecules l.1-l.6
lA-IB
2 1 Hybrid orbitals, organic compounds l.7-l.9
lC
3 2 Alkanes, IUP AC system, cycloalkanes, 2.1-2.5
cis-trans isomerism 2A
4 3 Properties of alkanes, functional groups 2.6-2.8
2B-2F
5 4 Alkenes: nomenclc.ture, sJi hybridization, 3.1-3.5
cis-trans isomerism, physical properties 3A-3C
6 5 Alkenes: chemical properties, mechanism, 3.6-3.8
addition polymers, alkynes 3D-3F
7 6 Aromatic hydrocarbons, fused and heterocyclic rings, 3.9-3.1 3
3G
8 First hour exam
9 7 Alcohols: nomenclature, chemical properties 4.1-4.3
10 8 Important alcohols, phenols, ethers 4.4-4.6
4A-4G
11 Physical properties of alcohols, phenols, ethers; 4.7-4.9
sulfur compounds, halides 4H-4I
12 9 Aldehydes, ketones: nomenclature, physical props, preparation 5.1-5.4
13 10 Aldehydes, ketones: chemical properties, acetals, 5.5-5.7
important aldehydfs and ketones 5A-5D
14 Acid, base strength, Bf0nsted-Lowry theory, Ka 6.1-6.4
6A
15 Acid, base properties, reactions, pH, self ionization of water, 6.5-6.11
buffers, normality, titration 6B-6F
16 11 Second or first hour exam
17 Carboxylic acids 7.1-7.6
7A
·The first column (2Q) lists sessions in a two quarter course (28 sessions per quarter). The

10
e Saunders CoUege Publkhing
second column (1 S) lists sessions in a one semester course (43 sessions).
2Q IS

18 12 Carboxylate 3alts, carboxylic esters 7.7-7.9


7B-7C
19 13 Carboxylic ester preparation and hydrolysis, 7.10-7.12
phosphates 7D-7F
20 14 Amines 8.1-8.4
8A-8B
21 15 Amide nomenclature, properties, 8.5-8.7
preparation 8C-8E
22 Amide hydrolysis, alkaloids 8.8-8.9
8F-8G
23 Third hour exam

24 16 Optical activity, chirality 9.1-9.4


25 17 Monosaccharides 9.5-9.8
9A
26 18 Monosaccharide derivatives, 9.9-9.11
di and polysaccharides 9B-9D
27 19 Acidic polysaccharides, 9.12-9.13
chiral compounds in biological organisms 9E-9F
28 Final Exam
2nd
-.Q..
1 20 Fats: structure and properties 10.1-10.3
10A-10D
2 21 Complex lipids, membranes, glycerophospholipids 10.4-10.8
sphingolipids, glycolipids 10E-lOF
3 22 Steroids, bile salts, prostaglandins, 10.9-10.12
leukotrienes 10G-IOJ
4 23 Protein functions, amino acids, zwitterions 11.1-11.4
5 24 Proteins: primary and secondary structure 11.5-11.8
11A-11E
6 25 Proteins: higher structure, 11.9-11.11
denaturation, glycoproteins I1F-IIH
7 26 First or second hour exam
8 27 Enzymes: names, classifications, 12.1-12.3
common terms 12A-12B
9 28 Factors affecting e-.1zyme 12.4-12.5
mechanism of enzyme action 12C-12E

11
© Saunders CoUege Publishing
2Q IS
10 29 Enzyme regulation, 12.6- 12.7
enzymes in medicine 12F
11 30 Bioenergetics, cells and mitochondria, 13. 1- 13.3
the common catabolic pathway 13 A
12 31 Citric acid cycle, electron and H+ transport 13.4- 13.5
13 32 Phosphorylation, chemiosmotic pump, 13.6- 13.8
energy yield, energy conversions 13B- 13 C
14 33 Specific pathways, glycolysis 14. 1- 14.2
14A
15 34 Energy yield from glucose, p-oxidation, 14.3- 14.7
energy yield from stearic acid, ketone bodies 14B- 14C
16 Catabolism of amino acids and heme 14.8- 14. 10
14D- 14G

17 Second hour exam


18 35 Biosynthesis of carbohydrates and 15. 1- 15.3
fatty acids 15A
19 Biosynthesis of membrane lipids 15.4- 15.5
and amino acids 15B- 15C
20 36 Nucleic acids, structure of DNA 16. 1- 16.4
and RNA, DN A replication 16A
21 37 RNA, transcription, translation, 16.5- 16. 10
protein synthesis 16B- 16C
22 38 Genes, exons, introns, gene regulation, 16. 1 1- 16. 14
mutations, recombinant DNA 16D- 16I
39 Third hour exam
23 40 Neurotransmitters and hormones 17. 1- 17.6
17A- 17G
24 41 Body fluids, blood 18. 1- 18.5
18A- 18E
25 Third hour exam
26 42 The kidney, immunoglobulins 18.6- 18. 10
18F
27 Nutrition, calories, diet, digestion 19. 1- 19. 10
19A- 19C
28 43 Final exam

12
© Saunders CoOege Publishing
EXAMINATION QUESTIONS

Chapter 1 of "Introduction to General, Organic, and Biochemistry'; fifth edition:


Matter, Energy, and Measurements

1. A statement that i s obvious to anyone and based on direct experience is called


(a) a hypothesis (b) a theory (c) a fact (d) a thesis

2. Which is a chemical change?


(a) melting of snow (b) making tea from a tea bag
(c) distilling ocean water (d) rusting of iron
3
3 . The number 5 1 0- , written in exponential notation, can b e written in full as
x

(a) 0.005 (b) 0.05 (c) 500 (d) 5 000


3
4. ( 5 . 9 1 0-8 )/( 8.3 1 0- )
x x =

(a) 7.1 1 05 (b) 7.1 1 01 1 (c) 7.1


x x x 1 0-6 (d) 7 . 1 x 1 0-5

5. Subtracting 3 .9 1 0-3 from 2 . 1 9 1 0-2 the result


x x
3 is
x
2
(a) 1 .80 1 0-2 (b) 6 . 09 1 0- (c) 1 .8 1 0- (d) - 1 .7 1
x x x 1 0-2

6. Which is a measure of volume?


(a) kg (b) mm (c) mL (d) nm

7 . Which of the following prefixes denotes 1 1 1 00 of a basic unit in the metric


system?
(a) milli (b) centi (c) deci (d) kilo

8. A temperature of 1 2 3 K is the same as


(a) OaF (b) - 1 5 0 ° C (c) -35 ° F (d) 3 96 ° C

9. A distance in kilometers is about 1 . 5 times more than in miles. Which is a


reasonable estimate of the distance in kilometers between Boston and New York
which is approximately 200 miles?
(a) 1 5 0 km (b) 3 00 km (c) 1 500 km (d) 3 000 km

1 0 . 1 kg 2.205 lb; l Ib 1 6 ounces. How many ounces are present in 2 . 86 kg?


= =

(a) 0.08 1 oz (b) 20.8 oz (c) 1 0 1 oz (d) 1 2.3 oz

1 1 . 1 mile 1 .609 km. A speed of 1 7.8 miles per hour is the same as
=

(a) 7960 meters/sec (b) 7.96 meters/sec


(c) 1 1 .1 meters/sec � d) 478 meters/sec

13
© Saunders College Publishing
1 2 . The density of ethyl alcohol is approximately 0.8 g/mL. A reasonable estimate of
the mass of 20 L of ethyl alcohol is
(a) 16 g (b) 0.04 kg (c) 16 kg (d) 25 kg

1 3 . 5 0 . 1 mL of bromoform weighs 145 g. The density of bromoform is therefore


(a) 2 . 8 9 g/mL (b) 0 . 345 g/mL (c) 2890 g/mL (d) 7250 g/mL

14. Matter is highly compressible in which of these states?


(a) gas (b) liquid (c) solid (d) liquid and solid

15 . Which is not a form of energy?


(a) electricity (b) heat (c) light (d) temperature

1 6 . The energy of motion is called


( a) stored energy (b) chemical energy (c) kinetic energy (d) potential energy

17. The law of conservation of energy states that


(a) things in nature tend to seek the lowest possible potential energy
(b) energy can neither be created nor destroyed
(c) potential energy is stored energy
( d ) it is impossible to convert potential energy to kinetic energy
1 8 . How many calories are required to heat 755 g of iron from 23 ° C to 1 75 ° C if the
specific heat of iron i s 0.11 cal/g· deg?
(a) 1 3 0 cal (b) 1 4 3 0 cal Ic) 56 cal (d) 1 3 kcal

1 9 . The specific heat of aluminum is 0.22 cal/g · deg. If 300 cal are ad de d to 1 0 . 0 g of
aluminum at 25 ° C what will its final temperature be?
(a) 208 ° C (b) 1 6 1 ° C (c) 1 3 6 ° C (d) 5 5 ° C

20. A breakfast cereal contains 1 1 0 nutritionist' s Calories/oz. How much is this in


cal/g if 1 oz 2 8 . 3 5 g?
=

(a) 3 8 80 cal/g (b) 3 1 1 8 cal/g (c) 3 . 1 1 8 cal/g (d) 3 . 8 8 cal/g

2 1 . The specific heat of water is 1 call g; that of iron is about 0. 1 cal/g. Give a
reasonable estimate as to which will require the most heat:
( a) heating 100 g water from 10 to 50 ° C (b) heating 1 kg of iron from 20 to 80 ° C
(c) heating 1 kg water from 5 to lOoC (d) heating 1 0 g iron from 0 to 200 ° C
Box 1A
22. A diagnosis made by & physician on the basis of laboratory tests
(a) is a fact (b) is a hypothesis based on facts
( c) is a proof of a hypothesis (d) must be true

14
© Saunders College Publishing
Box 1 B
23 . l Ib =0 . 453 kg. I f the recommended dosage of a prescribed drug i s 2 mglkg of
body weight, the amount of the drug that should be administered to a 12 pound
infant is
(a) 1 1 mg (b) 4 mg (c) 3 mg (d) 22 mg

Box 1 C
24 . Which of these causes hyperthermia?
(a) high fever (b) freezing outside temperatures
(c) high specific heat (d) overeating

Box 1 D
25 . Which of these liquids, when used as a cold compress, will need to be changed
least frequently? A liquid with a specific heat of
(a) 0 . 6 cal/g · deg (b) 0.06 cal/g · deg (c) 1 . 0 cal/g · deg (d) 0. 09 cal/g·deg

Chapter 2 of "Introduction to General, Organic, and Biochemistry': fifth edition:


A toms

1 . A pure substance made up of two elements in a fixed proportion by weight is a


(a) compound (b) element (c) mixture (d) atom

2. The symbol for the element potassium is


(a) P (b) Pt (c) K (d) Pm

3 . Which is an element?
(a) water (b) table salt (c) sugar (d) carbon

4 . An example o f a mixture is
(a) milk (b) nitrogen (c) water (d) iron

5 . An example of pure substance is


(a) blood (b) table salt (c) milk (d) air

6. The difference between the atomic theories of Democritus and Dalton is


(a) The atom of Democritus is divisible; that of Dalton is indivisible.
(b) The atom of Dalton is divisible; that of Democritus is indivisible.
(c) In both theories the atoms are indivisible, but Dalton' s theory was based on
evidence, Democritus' theory on speculation.
(d) There is no difference.

15
o Saunders College Publishing
7. Which of the following is not evidence for Dalton' s atomic theory?
(a) matter cannot be created or destroyed
(b) sulfur and iron can be mixed in any proportions
(c) a compound is always made up of elements in the same proportions by
weight
(d) all water molecules are identical

8. Which elementary particles attract each other?


(a) proton and neutron (b) electron and electron
(c) proton and electron (d) electron and neutron

9. Which elementary particle has no charge?


(a) electron (b) neutron (:;) proton (d) they all have a charge

10. An atom with 6 protons and 8 neutrons is an atom of which element?


(a) oxygen (b) carbon (c) silicon (d) helium

1 1 . An atom with 40 protons, 5 1 neutrons and 40 electrons has a mass number of


(a) 5 1 (b) 40 (c) 1 3 1 (d) 9 1

1 2 . Which zinc isotope has 3 5 neutrons?


(a) Zn-35 (b) Zn-66 (c) Zn-65 (d) Zn-34
2
1 3 . The ion Sr + has how many electrons?
(a) 36 (b) 3 8 (c) 86 (d) 8 8

1 4 . An ion with a mass number o f 5 6 that contains 26 protons and 2 3 electrons is


2 2
(a) Cr2+ (b) Fe + (c) Fe3+ (d) Sb +

1 5 . Which of these are isotopes?


1 . a particle with 33 protons, 36 neutrons, 33 electrons
2. a particle with 36 protons, 36 neutrons, 36 electrons
3. a particle with 3 6 protons, 3 8 neutrons, 36 electrons
4. a particle with 3 3 protons, 36 neutrons, 36 electrons
(a) 1 and 2 (b) 2 and 3 (c) 1 and 4 (d) 2 and 4

1 6 . Which of these is a transition element?


(a) Se (b) Ra (c) Ru (d) Ar

1 7 . The mass of a single atom consisting of 2 protons, 2 neutrons, and 2 electrons is


2
(a) 2 amu (b) 6 amu (c) 6.7 x 10- 4 g (d) 4 g

16

© Saunders CoUege Publishing


1 8 . The atomic weight of tin is 1 1 8 . 7 amu. From this we can conclud,e that
(a) every tin atom has a mass of 1 1 8 . 7 amu
(b) every tin atom weighs 1 1 8 . 7 g
(c) tin is a mixture of atoms of different masses, the average of which 1 1 8 . 7 amu
(d) tin is an unusual element

1 9 . If magnesium had only two isotopes, with natural abundances o f 65 . 00% magnesium-
24 and 3 5 . 00% magnesium-25, the atomic weight of magnesium
would be
(a) 24 . 3 5 amu (b) 24.50 amu (c) 24 . 3 05 amu (d) 24 . 65 amu

20. The elements in group IA of the periodic table are called


(a) halogens (b) alkali metals (c) noble gases (d) transition elements

2 1 . The elements that have the greatest tendency to accept electrons are in group
(a) VII A (b) 0 (c) IA (d) IlIA

22. Within the fourth principal energy level the orbital with the lowest energy is
(a) 4 f (b) 4 d (c) 4 p (d) 4 s

23 . The ground state electronic configuration of silicon is


(a) Is 2 2 s 2 2 p6 3 s 2 3 p 2 (b) Is 2 2 s 2 2 p6 3 s 4 (c) I s 2 1 p 6 2 s 2 2 p 4 (d) Is 2 2 s 2 3 s 2 3p6 4 s 2

24. In the ground state of an atom


(a) the energy of the electrons is at a maximum
(b) the excited states are all filled
(c) all the electrons are in their lowest energy levels
(d) the protons and neutrons fill all the available energy states

25 . Whi ch of these is the ground state electronic configuration of a transition


element?
(a) [Ne]3 s 2 (b) [Kr]4j 2 5 s 2 (c) [Ar]4 s 2 4 p 4 (d) [Ar]4 s 2 3 d1 0 4pl

26. The name is associated with the principle that if two electrons are in an orbital they
must have opposite spin is
(a) Bohr (b) Pauli (c) Hundt (d) Mendeleev

27. Which of these elements has the highest ionization energy?


(a) Mg (b) Al (c) P (d) Cl

Box 2A
28. Among the four necessary elements most important for human life is
(a) calcium (b) oxygen (c) sodium (d) chlorine

17

SaImders CoUege Publishing


Box 2B
29. Chemicals with long complicated names are often assumed to be harmful by people
who know little about chemistry. Which statement about chemicals is false?
(a) All chemicals are poisonous in any quantities.
(b) Some chemicals may be poisonous, but others are essential for life .
(c) Some chemicals may b e beneficial in small quantities and harmful in large
quantities.
(d) Vitamins are essential for nutrition; they are chemicals.
Box 2C
30. The isotope that is released by nuclear explosions and is absorbed by the human body
from contaminated dairy food is
(a) calcium-40 (b) stront�um-8 7 (c) strontium-90 (d) barium-90
Box 20
3 1 . In which of these periods were malleable tools first used?
(a) the stone age (b) the copper age (c) the bronze age (d) the iron age
Box 2E
32 . The most abundant element by weight both in the earth' s crust and in the human body
IS
(a) H (b) C (c) Ca (d) 0

Chapter 3 of "Introduction to General, Organic, and Biochemistry': fifth edition:


Chemical Bonds

1 . When a calcium atom loses two electrons the resulting species i s called
(a) a calcide (b) a calciumide (c) a calciumite (d) a calcium ion

2. In order to achieve a complete outer shell how many electrons must an atom of
strontium gain or lose?
(a) lose 2 (b) lose 3 (c) gain 1 (d) gain 2
3. If an atom X can accept two electrons from another atom in the process of forming a
stable ion, then the atom X
(a) has two protons in its nucleus (b) is a metal
(c) belongs to group IIA of the periodic table
(d) is capable of forming a molecule of the formula H 2 X

4 . The structure o f solid CaCl2 i s a regular three dimensional array o f which


particles?
(a) calcium atoms and chlorine molecules (b) calcium atoms and chlorine atoms
(c) calcium ions and chloride ions (d) calcium atoms and chloride ions

18

© Saunders CoUege Publishing


5. Which o f these does not have a complete outer shell?
(a) Ne (b) K+ (c) B2+ (d) CI-

6. Which of these is not a stable ion?


2
(a) Ba+ (b) Li+ (c) Br- (d) Se -

7. The formula for the compound between boron and fluorine is


(a) BF2 (b) B2F3 (c) BF3 (d) B2F

8. The formula for magnesium phosphate is


(a) MgP04 (b ) Mg3(P04)2 (c) MgiP04)3 (d) Mg3P04

9 . I n a Cl2 molecule the two chlorine atoms are held together b y a sigma molecular
orbital formed from the overlap of which two atomic orbitals?
(a) two s orbitals (b) an s orbital and a p orbital (c) two p orbitals
(d) two s orbitals and one p orbital

1 0 . Which of the following formulas is incorrect because it contains an atom that


does not have its normal number of covalent bonds?
H
I
(a) H-C=N (b) H-C=O (c) o=c=o (d) F-C-F
I I I
H H H

1 1 . Which element can form a triple bond?


(a) 0 (b) CI (c) N (d) Br

1 2. The Lewis structure of a carbon atom is


•• •

(a) C (b) :C: (c) :C (d) ·C·


.. .

1 3 . The Lewis structure of the CO molecule is


., r. ··

(a) :C=O: (b) :C=O: (c) :C-O: (d) :C-O:

14. The Lewis structure of the nitrite ion (N0 2 -) is


•• •• It •• I. .,. .e •• •• •• ••

(a) [:O-N=O :] (b) [:O-N-O:] (c) [:O=N=O:] (d) [:O=N=O:]


.. . . . .

1 5 . The Lewis structure of NO shows a total of 1 1 outer-shell electrons This


molecule is an example of
(a) a free radical (b) a molecule that obeys the octet rule
(c) a molecule that contains only paired electrons
(d) a molecule that contains a triple bond

19
e Saunders CoUege Publishing
1 6 . Which of these has a coordinate covalent bond?
(a) H -Cl: (b) H-N-H (c) :Ci -C l: (d) [H-O -Hr
·
•• I . . •• I
H H

1 7 . Which of these molecules has a linear shape?


(a) HCN (b) H2 0 (c) NH 3 (d) CH4

1 8. The shape of the molecule BCl3 is


(a) linear (b) planar triangular (c) tetrahedral (d) none of these

1 9 . The number of degrees in the angle between the two bonds CI-C -Cl in the
molecule CCl4 is
(a) 1 80 (b) 1 20 (c) 90 (d) 1 09. 5

20. Which of these molecul es is the least polar?


(a) FCI (b) FBr (c) HCl (d) HBr

2 1 . Which of these molecules is the most polar?


(a) HBr (b) CO 2 (c) CCl 4 (d) CBr4

22. Which compound contains both ionic and covalent bonds?


(a) CH4 (b) KCl (c) BaS0 4 (d) H 2 0

23. The ionic compound Ag2 S is called


(a) argon sulfide (b) silver sulfide (c) sulfur arsenide (d) silver sulfite

24. The compound iron(III) sulfite has the formula


(a) FeiS03)3 (b) I 2 S (c) FeS (d) Fe 2 S 3

2 5 . Which of these pairs will form ionic bonds?


(a) N, C (b) Na, Ca (c) ('s, Br (d) S, CI

Box 3A
26. The principal positive ion inside cells is
2 2
(a) Na+ (b) Ca + (c) K + (d) Mg +

Box 3B
27. Lithium carbonate is used in medicine as
(a) an antidepressive (b) an antacid (c) an expectorant (d) a diuretic agent

20

© Saunders College Publishing


Box 3C
2 8 . When it picks up oxygen in the lungs, the iron in the center of a heme molecule
binds to the oxygen by which type of bond?
(a) ionic (b) polar covalent (c) nonpolar covalent (d) coordinate covalent

Box 3D
29. What property of nitric oxide allows it to penetrate cell membranes with ease?
(a) it is a free radical (b) it reacts rapidly with molecular oxygen
(c) it reacts with the iron of hemoglobin
(d) its low polarity and small size

Chapter 4 of "Introduction to General, Organic, and Biochemistry ': fifth edition:


Chemical Reactions
1 . The formula weight for Ca(CI0 4 ) 2 , to the nearest integer, is
(a) 1 3 9 amu (b) 1 90 amu (c) 23 9 amu (d) 274 amu

2 . How many moles o f propanol, C 3 HsO, are there in 42. 1 g o f propanol?


(a) 0.252 (b) 0 . 702 (c) 1.43 (d) 8 . 77

3. How many moles of N atoms are there in 0. 882 moles of (NH2 ) 2 CO?
(a) 0 . 44 1 (b) 0.8 82 (c) 1.76 (d) 3.53

4. 1 . 62 moles of CaS0 4 weighs


(a) 22 1 g (b) 3 4 . 6 g (c) 15 8 g (d) 22 1 kg

5. How many molecules are in a cube of dry ice (C0 2 ) that weighs 440 g?
(a) 10 (b) 1 1 0 24 (c) 6 . 02 1 023 (d) 6 . 02 10 24
x x x

6 . Which quantity just contains Avogadro ' s number o f molecules?


(a) 8 . 8 g CO 2 (b) 34 g NH3 (c) 9 g H2 0 (d) 98 g H 2 S0 4

7. Moles are used in chemistIy because they


(a) help us balance equations
(b) allow us to measure Ollt a given number of atoms or molecules by weighing
out a sample
(c) allow us to calculate atomic weights
(d) allow us to calculate formula weights

8 . Which o f these samples contains the largest number o f particles?


(a) 0 . 20 mole of C atoms (b) 0. 10 mole of O 2 molecules
(c) 1.08 g of B atoms (d) 0.50 g ofH2 molecules

21
e Saunders CoUege Publishing
9 . When this equation is properly balanced
C 3 H6 + O 2 � CO 2 + H2 0
the coefficient of O 2 is
(a) 2 (b) 4 (c) 6 (d) 9

1 0. For the reaction


2P + 3 Cl 2 � 2PCl3
if 3 2 . 5 g of Cl 2 reacts completely with excess P, how many g of PCl3 will be
produced?
(a) 42 . 0 g (b) 62 . 9 g (c) 8 3 . 9 g (d) 94.4 g

1 1. For the reaction


CH4 + 2Br2 � CH2 Br2 + 2HCI
how many grams of Br2 are required to produce 24 . 9 g of CH2 Br2 , assuming
e xc ess CH4 is present?
(a) 1 1 . 5 g (b) 22 . 9 g (c) 27. 4 g (d) 45. 8 g

1 2. A ch e mist calculated the theoretical yield of a product to b e 20. 1 g. The


perc entag e yield was 8 7.0%. How many grams of product did the chemist obtain?
(a) 7 . 5 g (b) 1 7 . 5 g (c) 1 8 g (d) 23. 1 9 g

13. In this equation


Ag+ + N03 - + Na+ + Cl- � AgCI (s) + Na+ + N0 3 -
the spectator ions are
(a) Ag+ and CI- (b) Na+ and CI- (c) Na+ and N0 3 - (d) Ag+ an d N 0 3 -

14. When a solution of (NH 4 ) 2 S is added to a solution of Cd (N0 3 ) 2 a yellow


'

p rec ipitate of CdS is formed. A balanced ionic equation for this is


(a) (NH4) 2 S + Cd(N0 3 ) 2 � CdS + 2NH4N0 3
(b) Cd2 + + S- � CdS
(c) Cd2 + + S 2 - � CdS
(d) NH/ + N03 - � NH4 N03

1 5. When a solution of H2 S04 is added to a solution of Na2 C0 3 , the gas CO 2 is


released. A balanced ionic equation for this process is
(a) 2Na+ + S O/- � Na2 S04
(b) H 2 S04 + Na2 C03 � CO 2 + Na2 S0 4 + H 2 0
(c) 2H+ + C0 3 2 -� CO 2 + H2 0
(d) H+ + C03- � CO 2 + OH-

22

e Saunders CoUege Publishing


1 6. Which of these ionic compounds is insoluble in water?
(a) Fe(OH) 3 (b) K2 S (c) CU(N03 ) 2 (d) NH4Cl

1 7 . The loss of an electron is classified as which kind of reaction?


(a) oxidation (b) reduction (c) decomposition (d) combination

1 8. In the reaction
2Mn2+ + Br2 � 2Mn3 + + 2Br­
the species that is oxidized is
(a) Mn2+ (b) Br- (c) Mn3+ (d) Br2

19. When an oxidation reaction takes place


(a) the oxidizing agent gets oxidized
(b) the oxidizing agent loses hydrogen or gains oxygen
(c) one or more electrons are transferred
(d) there does not need to be a simultaneous reduction reaction

20. All of these are redox reactions except


(a) bleaching of a colored fabric (b) rusting of iron
(c) breathing (d) the reaction between solutions of AgN03 and NaCI to give AgCl

21. The decomposition of S03 to S02 and O 2 requires 24 kcal/mole heat input. This
reaction is
(a) exothermic (b) endothermic (c) exergonic (d) a combustion reaction

22. If 94 kcal of heat are given off when 12 g of carbon are completely burned, how
many kcal will be given off if 2.0 g of carbon are completely burned?
(a) 1 6 kcal (b) 28 kcal (c) 47 kcal (d) 94 kcal

23. An endothermic reaction


(a) is always a combustion reaction (b) takes in heat
( c) cannot happen at high temperatures
(d) cannot happen without an exothermic reaction taking place at the same time

Box4A
24 . In the voltaic cell containing Cu2+ and Zn metal
(a) the flow of electrons through a wire provides the electric current
(b) Zn is oxidized
(c) Cu2+ is reduced
(d) all of these

23

s-.ders CoUege Publishing


Box 4B
25 . An example of an oxidizing disinfectant used to treat swimming pools or
municipal water supplies is
(a) chlorine (b) iodine (c) KMn04 (d) H2 02

Box 4C
27. The key chemical process in photography is
(a) oxidation of metallic Ag to AgBr
(b) reduction of metallic Ag to AgBr
(c) oxidation of AgBr to metallic Ag
(d) reduction of AgBr to metallic Ag

Box 4 D
2 7 . The caloric values of some foods are as follows:
8 oz milk = 165 Cal 1 slice bread = 60 Cal
1 tbs honey 60 Cal
= 1 tbs butter = 1 00 Cal
2 oz beef steak = 115 Cal 5 oz potato = 90 Cal.
Which combination will give the fewest Calories?
(a) 8 oz milk, 1 slice bread, \12 tbs butter
(b) 2 oz beefsteak, 1 slice bread
(c) 2 slices of bread, \12 tbs butter, 1 tbs honey
(d) 2 oz beefsteak, 10 oz potato

Chapter 5 of "Introduction to General, Organic, and Biochemistry'; fifth edition:


Gases, Liquids and Solids

1. Which does not happen when the temperature rises?


(a) molecules move faster (b) molecules move more randomly
( c) attractive forces between molecules increase
(d) molecules have more kinetic energy

2. Which statement best describes an ideal gas? Molecules


(a) are strongly attracted to each other by hydrogen bonds
(b) are strongly attracted to each other by dipole-dipole interactions
(c) do not have volumes (d) have real volumes

3. A pressure of 0. 800 atm is the same as


(a) 950 mm Hg (b) 608 mm Hg (c) 60.8 mm Hg (d) 200 torr

4. A gas with a volume of 5.0 L at a pressure of 0.80 atm is allowed to expand until
the pressure drops to 0.20 atm at constant temperature. The new volume is
(a) 10 L (b) 5.0 L (c) 20 L (d) 2.0 L

24

© Saunders College Publishing


5. A gas occupies 3 . 0 L at 20 ° C and a pressure of 500 torr. What will be the new
pressure if the gas is heated to 1 00 ° C and its volume is decreased to 1 .2 L?
(a) 1 . 6 x 10 3 torr (b) 6 . 3 x 1 0 3 torr (c) 1 . 2 atm (d) 8 . 2 atm
6. A gas occupies 4 . 00 L at 3 75 mm Hg and 86 ° C. If the temperature is lowered to
45 ° C and the pressure increased to 820 mm Hg, what is the new volume?
(a) 0.957 L (b) 775 mL (c) 9 . 8 7 L (d) 1 . 62 L

7. STP means
(a) O ° C and 760 torr (b) 273 ° C and 1 atm
(c) 273 K and 760 atm (d) the conditions at sea level

8 . The units o f the universal gas constant, R 0. 0821 are =

(a) L · atml° C·mole (b) L·atmlK·mole


(c) L torr/K . mole (d) mL torr;oC mole
• • •

9. The volume of 1 mole of an ideal gas at 4 atm and O ° C is


(a) 5 . 6 L (b) 1 5 .29 L (c) 22. 4 L (d) 89.6 L

10. In the equation PV nRT, =

(a) n is the number of molecules


(b) n is the number of moles
(c) Tis the temperature in OF
(d) Tis the temperature in ° C

11. If 1 0 g of CO 2 gas, molecular weight 44, occupies 1 0 . 0 L at 3 5 0 K what is the


=

pressure?
(a) 0 . 3 3 atm (b) 0 . 65 atm (c) 660 mm Hg (d) 1 atm

12. We can increase the volume of a gas in an expandable container (such as a


balloon) by
(a) increasing the pressure (b) decreasing the temperature
(c) adding more gas (d) allowing some of the gas to escape

13 . The label of a 1 0 . 0 L-gas cylinder fell off. To identify the g as a chemist weighed
the cylinder at 298 K and 1 2 . 2 atm pressure. The gas in the cylinder was found to
weigh 1 0 . 0 g. The gas was
(a) N2 (b) CO (c) H2 (d) O 2

14. A cylinder contains a mixture of two gases. The partial pressure of g as A is 700
torr; that of gas B is 0 . 5 00 atm. The total pressure of the gases in the cyl inder is
(a) 0 . 0 8 atm (b) 1 . 08 atm (c) 1 080 torr (d) 1450 torr

1 5 . Which gas will diffuse faster than C 2 H6 ?


(a) CI2 (b) NH3 (c) S02 (d) H202

25

, SuDde rs College Publishing


16. Real gases resemble ideal gases most at
(a) low temperature and high pressure (b) low temperature and low pressure
(c) low volume and high pressure (d) high temperature and low pressure

17. Which of these is not an intermolecular force?


(a) dipole-dipole attraction (b) hydrogen bond
(c) covalent bond (d) London dispersion force

1 8 . The intermolecular forces operating between two CH4 molecules are


(a) London dispersion forces (b) hydrogen bonds
(c) dipole-dipole attraction (d) ion-dipole attraction

19. The molecules of which of these compounds exhibit hydrogen bonding?


(a) CH4 (b) NO (c) NH3 (d) CO

20. Which of these compounds has the highest boiling point?


(a) CH4 (b) NO (c) CO (d) HF

2 1 . Which of these compounds has the highest boiling point?


CH3
I
(a) CH4 (b) CH3-CH2-CH2-CH2-CH3 (c) CH3-C-CH3 (d) CH3-CH2 -CH3
I
CH3
22. Which of these molecules cannot form hydrogen bonds with another molecule of
itself?
(a) CH3CH2 -OH (b) CH3-O-CH3 (c) CH3-NH-CH3 (d) HF

23 . A regular three-dimensional array of atoms, ions or molecules is called


(a) an amorphous solid (b) a liquid crystal (c) glass
(d) a crystal lattice

24. The newest solid form of carbon, buckminsterfullerene,


(a) is a network solid (b) has an amorphous structure
(c) has molecules that resemble the shape of a soccer ball
(d) comes in sheet-like structures

25 . When ice changes directly to water vapor, the phase change is called:
(a) condensation (b) sublimation (c) melting (d) boiling

Box 5A
26. Which form of mercury has the lowest entropy?
(a) solid Hg at 10 K (b) liquid Hg at lOoC
(c) liquid Hg at lOO°C (d) Hg vapor at 1 00°C

26

© Saunders College Publishing


Box 5B
27. Prolonged inhalation of mercury vapor may cause
(a) teeth to fall out (b) uncontrolled twitches
(c) nervous disorders (d) all of these
Box 5C
28. The systolic pressure is read on the sphygmomanometer when
(a) the first tapping sound is heard
(b) the highest inflation point is reached
(c) the last tapping sound is heard
(d) the pressure in the inflating cuff reaches 1 atm

Box 5D
29. Raising the rib cage
(a) increases the volume of the chest cavity
(b) decreases the volume of the chest cavity
(c) increases the pressure of the chest cavity
(d) increases the temperature of the chest cavity

Box 5E
30. In carbon monoxide poisoning oxygen is administered in hyperbaric chambers.
This treatment is efficient because at 2 or 3 atm pressure of pure oxygen
(a) the oxygen replaces the CO from the hemoglobin
(b) more oxygen is dissolved in the plasma, so that more oxygen reaches the
tissues
(c) the oxygen in the gas form diffuses through the skin and reaches the tissues
(d) all of these

Box 5F
31. Where is the partial pressure of oxygen lowest?
(a) in the air (b) in arterial blood (c) in the tissues (d) in venous blood

Box 5G
3 2 . Frostbite, the destruction of cells in freezing weather, is caused by the fact that
(a) ice has a larger volume than an equal weight of liquid water
(b) ice is denser than liquid water
(c) ice has closer packing of molecules than liquid water
(d) ice has a smaller volume than an equal weight of liquid water

Box 5H
3 3 . A compound used to alleviate pain by reducing swelling in athletic injuries is
(a) methane (b) ethane (c) ethyl chloride (d) ethanol

27

Swmders College Publishing


Chapter 6 of "Introduction to General, Organic, and Biochemistry'; fifth edition:
Solutions and Colloids

1 . Which statement is incorrect? A true solution


(a) is always transparent
(b) cannot be separated into its components by filtration
(c) can have only one composition
(d) will not separate into its components on standing

2 . An example o f a liquid dissolved i n a liquid i s


(a) sea water (b) beer (c) syrup (d) tap water

3. A solution is stable and transparent, but when a single crystal is adde d more
crystals suddenly precipitate. Such a solution is called
(a) saturated (b) unsaturated (c) supersaturated (d) nonpolar

4. The solubility of a gas in a liquid mostly


(a) increases with increasjng temperature
(b) increases with decreasing pressure
(c) is unaffected by pressure
(d) decreases with increasing temperature

5 . How many grams of NaCI are needed to make a 5% w/v saline solution having a
total volume of 300 mL?
(a) 6 g (b) 1 2 g (c) 1 5 g (d) 0. 1 5 g

6. What volume of a 9 . 0% (w/v) solution of sugar in water contains 3.5 g of sugar?


(a) 2 . 5 mL (b) 3 9 mL (c) 100 mL (d) 3 9 L

7. When 4.0 g of NaOH is dissolved in 5 0 mL of aqueous solution the molarity of


the solution is
(a) 1 .0 M (b) 2.0 M (c) 0. 1 0 M (d) 0.20 M

8 . 15 0 mL o f a 5 . 0 M solutifJn of CH 2 C12 contains how many moles of CH2 C12?


(a) 0 . 75 mole (b) 7 . 5 moles (c) 3 0 moles (d) 0 . 3 mole

9. The concentration of NaCI in blood serum is 0.14 M What volume of blood


serum contains 4 . 0 g NaCI?
(a) 490 mL (b) 240 mL (c) 5 5 5 mL (d) 3 8 . 5 mL

10. How many mL of water must be added to a 1 00 mL of 0 . 6 M solution to make it


0 .4 M ?
(a) 1 5 0 mL (b) 3 0 0 mL (c) 3 0 mL (d) 5 0 mL

28

© Saunders College Publishing


1 1. You have a 5 0 . 0% stock solution of NaOH. How many mL of stock solution do
you need to make 3 5 0 mL of a 2 . 5 0% NaOH solution?
(a) 7 . 0 mL (b) 1 4 . 0 mL (c) 17. 5 mL (d) 35 mL

1 2. A contaminated water supply contains 12. 5 mg of cadmium in 100 kg of water.


This concentration is
(a) 12 . 5 ppm (b) 1 25 ppb (c) 1 . 25 ppb (d) 125 ppm

1 3. When water is part of the crystal of a molecule, the crystal is called


(a) anhydrous (b) a colloid (c) an electrolyte (d) a hydrate

1 4. Which of these is the least soluble in water?


(a) CH30H (b) HCOOH (c) CH3Br (d) CF30H

15 . Which of these is a strong electrolyte?


(a) distilled water (b) 1 M CH3COOH in water (c) 1 M sugar in water
(d) O. 1 MNaCI in water

1 6 . Fog is an example of a colloidal system in which


(a) a liquid is dispersed in a liquid (b) a gas is dispersed in a liquid
(c) a liquid is dispersed in a gas (d) a gas is dispersed in a gas

17. The Tyndall effect is, in essence


( a) light scattering (b) freezing point lowering
(c) electrical conductance (d) a colligative property

1 8. The freezing point of an aqueous solution that contains 2 . 0 moles of CaCl2 per
1000 grams of water is
(a) - 1 . 86 ° C (b) -3 . 72 ° C (c) - 1 1 . 2 ° C (d) -5 . 5 8 ° C

1 9 . Which solution has the greatest osmotic pressure?


(a) 0.6 M NaCI (b) 0 . 3 5 M AICl3 (c) 0.8 % w/v NaCI (d) 0.2 M CaBr2•

20. � semipermeable membrane

A B

If compa11ment A contains 0.4 M B aCI2 , which of these solutions in compartment


B will cause the level in A to rise?
(a) 0 .4 M AI 2 (S04)3 (b) 0 . 4 MK2 C03 (c) 0.4 MNaBr (d) 0.5 MBaCl2

29

C Saunders College Publishing


2 1 . Hypertonic solutions cause
(a) red blood cells to shrivel (b) red blood cells to burst
(c) hemolysis of red blood cells (d) no effect on red blood cells

22. The osmolarity of a 0 . 3 0 M Na3P0 4 solution is


(a) 0. 1 0 osmol (b) 0 . 90 osmol (c) 0 . 075 osmol (d) 1.20 osmol
Box 6A
23 . Which of these gases is the least harmful in causing acid rain?
(a) CO 2 (b) S0 2 (c) S03 (d) N0 2
Box 6 B
2 4 . Which gas can cause the bends by forming lethal bubbles in the blood?
(a) He (b) O 2 (c) N2 (d) CO 2
Box 6C
25 . Which pollutant gas in air causes the conversion of marble in monuments to
gypsum?
(a) S03 (b) CO 2 (c) HCI (d) CO

26. How does gypsum (CaS0 4 · 2H2 0) cause flaking of marble monuments?
(a) gypsum has a larger volume than marble ; it expands in contact with water
(b) gypsum is a basic salt ( c) gypsum is dissolved in water
(d) gypsum sublimes in air

Box 6 D
27. Which of these is not a symptom of low K+ concentration in the body?
(a) muscle cramps (b) tachycardia (c) hyperglycemia
(d) falling blood pressure

Box 6E
28. The emulsifying agent in mayonnaise is
(a) water (b) vinegar (c) egg yolk (d) olive oil
Box 6F
29. During hemodialysis a cellophane tube is suspended in an isotonic solution.
Which of these is not an ingredient of the isotonic solution?
(a) albumin (b) NaCI (c) KCI (d) glucose

Chapter 7 of "Introduction to General, Organic, and Biochemistry'; fifth edition:


Reaction Rates and Equilibrium

1. Chemical kinetics i s the study of


(a) equilibria (b) catalysis (c) reaction rates (d) reversible reactions

© Saunders College Publishing


30
2 . When the reaction 2HgO � 2Hg + O 2 was carried out in a 2 .0-L bulb, 2.0
moles of oxygen were prclduced in the first 5 minutes of the reaction. The rate of
reaction, in moles/L per minute, is
(a) 0.20 (b) 0.40 (c) 2.0 (d) 4. 0

3. The top of the energy hill is called the


(a) downhill reaction (b) uphill reaction (c) transition state
(d) energy of the reaction

4. At 200 ° C most collisions between 12 and H 2 molecules do not result in a reaction


to give two HI molecules because
(a) an H 2 molecule is more likely to collide with another H2 molecule than with
an 12 molecule
(b) the energy of most of the collisions is too low to result in a reaction
(c) H2 and 12 are unreactive
(d) the temperature is too high

5. In general, the fastest types of reaction are those between


(a) covalent molecules in the gaseous state
(b) covalent molecules in aqueous solutions
(c) covalent molecules in nonaqueous solutions
(d) ions in aqueous solutions

6 . I f a reaction takes 8 hours t o g o t o completion at 1 5 ° C, approximately how long


will it take at 45 ° C ?
(a) 1 hr (b) 2 hr (c) 8 hr (d) 6 4 hr

7. A catalyst increases the rate of reaction by


(a) increasing the activation energy
(b) decreasing the activation energy
(c) adding energy to the reacting molecules
(d) increasing the net energy released

8. When a reaction reaches equilibrium


(a) all reaction ceases
(b) the rate of the reverse reaction becomes zero
(c) the concentration of reactants and products are equal
(d) the rate of the forward reaction equals the rate of the reverse reaction

o Saunders College Publi"hing


31
9. The equilibrium expression for this reaction
4H202 + CH 4 � CO 2 + 6H 2 0
IS

[H202] [CH4]
a
( ) K=
[C02] [H20]

[C02] [H20]6
(c) K =
[H202]4 [CH4]

10. For the reaction

the equilibrium concentrations, in moles/L, were found to be: [N21 = 0. 70,


[H21 = 1 . 4 [NH31 = 3 . 0 . The equilibrium constant for this reaction is
,

(a) 0 . 2 1 (b) 3 . 1 (c) 4. 7 (d) 20

1 1 . The following are equilibrium constants for some reactions. In which is formation
of products most favored?
(a) 1 . 0 (b) 23 . 5 (c) 0.25 (d) 3 1 0-2
x

12. Which statement is false? An equilibrium constant for a particular reaction


(a) changes when the temperature increases
(b) changes when a catalyst is added
(c) does not change when an additional quantity of a reactant is added
(d) does not change when a product is removed

1 3 . The reaction

is exothermic. Which of the following will drive the reaction to the right?
(a) a decrease in temperature
(b) an increase in temperature
(c) the removal of CH4
(d) the addition of CO 2

14. If, in the reaction

the S02 is constantly removed by allowing it to react with water, the removal of
S02 will
(a) shift the equilibrium to the left (b) shift the equilibrium to the right
(c) speed up the rate of the reaction
(d) have no effect on the position of equilibrium

32
© Saunders CoUege Publi�hing
1 5 . A change in the temperature of a reaction at equilibrium will
(a) change the position of the equilibrium, but not the value of K
(b) change the value of K, but not the position of equilibrium
(c) affect only the rate of the reaction
(d) change both K and the position of equilibrium

Box 7A
1 6. A sustained body temperature above 1 07 ° F is
(a) fatal (b) kills bacteria (c) enables the optimal utilization o f oxygen
(d) enables the optimal utilization of the immune system

Box 7 B
1 7. I n some operations body temperature is lowered to reduce the brain' s oxygen
consumption. A safe body temperature in this procedure is (in ° C)
(a) 0 (b) 5 (c) 28 (d) 4 1

B ox 7C
1 8 . Enteric-coated aspirin will
( a) release its contents over a 24 hour period (b) neutralize stomach acid
(c) release its contents in the intestines only (d) dissolve in stomach acid

Box 7 0
19 . I n sunglasses with adjustable shading the dominant species incorporated in the
glasses are
(a) Ag+ and CI- (b) Ag and Ag+ (c) Cl 2 and Ag (d) Cl2 and CI-

Box 7 E
2 0 . The term "nitrogen fixation" refers t o which process?
(a) oxidation of atmospheric nitrogen to NO
(b) oxidation of atmospheric nitrogen to N0 2
(c) reduction of atmospheric nitrogen to NH3
(d) conversion of nitrogen to nitric acid, HN03, by bacteria in the roots of
alfalfa

2 1 . Which of these statements about the Haber process is true?


(a) it can operate at ambient (room) temperature
(b) it uses a catalyst that provides good yield rapidly at 500 ° C
(c) the catalyst shifts the reaction toward the product, NH3
(d) the catalyst shifts the reaction toward the reactants, N2 and H2

33

() Saunders CoUege Publishing


Chapter 8 of "Introduction to General, Organic, and Biochemistry': fifth edition;
Chapter 6 of "Introduction to Organic and Biochemistry': third edition:
A cids and Bases

1. An acid-base reaction, in the Bnmsted-Lowry theory, is


(a) the transfer of electrons (b) the transfer of a proton
(c) the transfer of an OH- ion (d) does not involve any transfer at all

2. A substance that is a weak base when dissolved in water is


(a) NaOH (b) H2 S (c) CH4 (d) NH3

3. The conjugate base o f HCI is


(a) CI- (b) H2 0 (c) H30+ (d) Cl2

4. Which is a diprotic acid?


(a) CH3 COOH (b) HCOOH (c) H2 S0 3 (d) HF

5 . Which o f these i s not a strong acid?


(a) HCI (b) H 2 S (c) HN0 3 (d) H2 S 04

6. Which of these is amphoteric?


(a) HCI (b) H 2 0 (c) NaOH (d) CO/-

7. Given the ionization constants, which of these is the strongest acid?


(a) HCOOH, 1 . 8 1 0-4 (b) HF, 3 . 5 1 0-4
x x

(c) H2 C03 , 4. 3 1 0-7 (d) H3 B03 , 7.3 1 0-1 0


X x

8. The acid HCI will react with all of these except


(a) CH3 COOH (b) K (c) CH3 NH2 (d) CaC03

9. A solution with a [H 3 0+ ] of 0 . 00 1 M has a pH of


(a) 3 (b) 4 (c) 7 (d) 1 1

10. The [H 3 0+ ] of a solution is 3 . 2


1 0-8 • The solution is
x

(a) strongly acidic (b) stLmgly basic (c) weakly acidic (d) weakly basic

1 1 . A pH of 8 means a [H30+] of
(a) 8 M (b) 0 . 8 M (c) 1 X 1 0-8 M (d) 8 x 1 0-8 M

12. If the [OH- ] of a solution is 1 x 1 0-1 M, the pH of the solution is


(a) 1 (b) 10 (c) 13 (d) 14

34

© Saunders CoUege Publishing


1 3 . A substance that changes color at a certain pH is called
(a) a buffer (b) an indicator (c) a conjugate acid (d) a salt

14. When the salt of a weak base and a strong acid is dissolved in water, the solution
will be
(a) acidic (b) basic (c) neutral
(d) any of these, depending on the particular acid and base

1 5 . The pH of an 0 . 1 M solution of KCI is approximately


( a) 1 (b) 5 ( c) 7 (d) 9

1 6 . If 0 . 1 mole of sodium acetate is added to 1 . 0 L of pure water (pH = 7), the pH


will
( a) rise to about 9 (b) rise to about 14 (c) fall to about 5 (d) fall to about 1

1 7 . Which of these, if dissolved in 1 . 0 L of pure water, will produce a buffer


solution?
(a) 0. 1 mole NaCI + 0. 1 mole KCI
(b) 0. 1 mole H30+ + 0. 1 mole OH-
(c) 0. 1 mole NaH2P04 + 0. 1 mole Na2 HP0 4
(d) 0 . 1 mole HCI + 0. 1 mole KCI

1 8 . Which of these solutions has the largest buffer capacity? A solution that is
(a) 0 . 0 1 M in acetic acid and 0. 0 1 Min sodium acetate
(b) 0. 1 M in acetic acid and 0 . 1 Min sodium acetate
(c) 0 . 5 M in acetic acid and 0 . 5 M in sodium acetate
(d) 1 . 0 M in acetic acid and 1 . 0 M in sodium acetate

19. In order to maintain a pH of 7. 4 in the blood the ratio of the


[HC03 -]/[H2C03] buffer must be
( a) 1 0: 1 (b) 1 : 1 0 (c) 1 : 1 ( d) 1 : 2

_0. The number of equivalents in 87 g of Mg(OH)2 is


(a) 1 (b) 2 (c) 3 (d) 4

2 1 . The normality of a 2 . 5 M H3P04 solution is


( a) 1 0 (b) 7.5 (c) 0 . 8 3 (d) 0 . 62

22. A solution of 5 0 . 0 mL of :m unknown base dissolved in water was titrated with an


0 . 20 M solution of H2S04. It required 20. 0 mL of the acid solution to reach the
end point. The normality of the basic solution was
(a) 0.080 N (b) 0 . 1 6 N (c) 0.20 N (d) 1 . 6 N

35

5 Yrs College Publishing


Box 8A
23 . Stomach acid is
(a) nitric acid (b) acetic acid (c) hydrochloric acid (d) phosphoric acid

Box 8 B
2 4 . If a strong base is accidentally splashed in the eye, the best procedure is to
(a) find a weak acid, such as boric acid, with which to wash the eye
(b) immediately wash the eye with copious amounts of cold water
(c) call a doctor
(d) none of these

Box 8 C
25. Which of these is not commonly found in drugstore antacids?
(a) CaC03 (b) AI(OH)3 (c) Mg(OH)2 (d) NaOH

Box 8 0
26. The most common fire extinguisher uses a gas, heavier than air, i n the form o f a
foam. This gas is
(a) CO (b) NH3 (c) Cl 2 (d) CO 2

Box 8E
27. Acidosis can be caused by
(a) hypoventilation (b) hyperventilation (c) excessive vomiting (d) hysteria

Box 8 F
28. The pKa oflactic acid is 3 . 8 5 . A buffer solution in which the concentration of
lactate ion is ten times that of lactic acid has a pH of
(a) 2 . 00 (b) 2 . 8 5 (c) 3 . 8 5 (d) 4 . 85

29. If we wish to make a buffer solution of pH 7.5 1 , using H2 P0 4 - and HPO/- (pKa
of H 2 P0 4 - 7 . 2 1 ), the molar ratio [ HP0 4 2-]/ [ H2 P0 4 -] should be
=

(a) 2 to 1 (b) 10 to 1 (c) I to 2 (d) 1 to 1 0

Chapter 9 of "Introduction to General, Organic, and Biochemistry'; fifth edition;


Nuclear Chemistry

1 . Radioactivity was discovered by


(a) Becquerel (b) Marie Curie (c) Pierre Curie (d) Rutherford

2. Which kind of radiation is a stream of electrons?


(a) alpha (b) beta (c) gamma (d) delta

36

© Saunders CoUege Publishing


3 . Which of these statements about the wavelength of electromagnetic radiation is
false?
(a) It is the distance between any crest of the wave and the next crest
(b) It can be measured in meters
(c) Waves of the shortest wavelength have the highest frequency
(d) It is equal to the speed of light multiplied by the frequency

4. Which form of radiation penetrate s the human skin?


(a) radio waves (b) alpha particles (c) gamma rays (d) microwaves

5 . Which o f these pa11icles has a charge o f + 1 and an atomic mass unit o f 0.0005 5 ?
(a) neutron (b) proton (c) electron (d) positron

6 . Which isotope is the most stable?


(a) carbon- 1 1 (b) carbon- 1 2 (c) carbon- 1 3 (d) carbon- 1 4
7 . Deuterium is the name o f which o f these isotopes?
(a) hydrogen- l (b) hydrogen-2 (c) hydrogen-3 (d) carbon-6

8 . W e can detect radioactivity


(a) by hearing the very faint sound it gives off
(b) by a burning sensation in the eyes (c) by smell
(d) our senses are unable to detect radioactivity
9 . The conversion o f carbon- 1 4 t o nitrogen- 1 4 is an example o f which type of
emission?
(a) alpha (b) beta (c) gamma (d) positron

1 0. l �!PO is an alpha emitter. The product is


(a) polonium- 1 90 (b) lead 1 9 4 (c) lead- 1 90 (d) astatine- 1 94
-

I I . Which of the following does not change the number of protons in the nucleus?
(a) alpha emission (b) beta emission (c) gamma emission
(d) all of these change the number of protons in the nucleus

1 2 . The half-life of carbon- 14 is 5700 years. How many years will it take for one
gram of C- 1 4 to decay to 0 . 25 g?
(a) 1 1 ,400 (b) 2850 (c) 5700 (d) 1 7, 1 00

1 3 . Tellurium- 1 3 2 has a half-life of 78 hours. If you have sample of 1 84 mg of this


isotope, how much will be left after 1 3 days?
(a) 1 1 . 5 mg (b) 23 mg (c) 46 mg (d) 1 3 8 mg

14. The unit mCi is used to measure what characteristic of radiation?


( a) intensity (b) energy (c) penetrating power (d) ionizing power

37
ers CoUege Publishing
1 5 . A nurse working in a radiation lab wears a badge that monitors the amount of
exposure. The level of exposure to radiation is measured in which units?
(a) microcuries (b) millirems (c) rads (d) roentgens

1 6 . What is the principal source of the radiation humans are exposed to from natural
sources?
(a) radon in the air (b) cosmic rays (c) radium inside the bones
(d) terrestrial rocks and buildings

1 7. Tritium is used medically to


(a) measure the water content of the body (b) perform lung ventilation scans
(c) diagnose thyroid malfunction (d) detect eye tumors

1 8 . A radioactive isotope that is used for therapy is


(a) cobalt-60 (b) hydrogen-2 (c) helium-4 (d) uranium-23 8 .

1 9 . The energy o f the sun i s derived from a nuclear reaction i n which hydrogen
isotopes are transformed into helium. This is an example of
(a) fission (b) fusion (c) alpha emission (d) positron emission

20. When a uranium-23 5 nucleus is struck by a neutron, among the reaction products
are three neutrons that can strike additional uranium-23 5 nuclei. This is called
( a) background reaction (b) tagging (c) a self propagating chain reaction
(d) a hydrogen bomb

2 1 . When a nucleus of curium-244 is bombarded with an alpha particle the result is


one proton, two neutrons and the nucleus of a heavy element. This nucleus is
(a) berkelium-247 (b) berkelium-245 (c) californium-246 (d) califomium-245

22. In nuclear plants boron rods are used to control the rate of fission. B oron
(a) slows down the alpha particles (b) absorbs neutrons
( c) cools the emerging steam (d) absorbs gamma rays

Box 9A
23 . What name did Rutherford give to rays that were not deflected by a magnet?
(a) alpha (b) beta (c) gamma (d) delta

Box 9 B
2 4 . In carbon- 14 dating of an artifact the number of disintegrations per minute is
counted with a scintillation counter. Which particle or ray is emitted?
(a) alpha (b) beta (c) gamma (d) proton

38

@ Saunders CoUege Publishing


Box 9C
25 . Radon-222 is a radioactive decay product of uranium-23 8 . Why does this isotope
pose greater health hazards than the original uranium?
(a) it is a solid rather than a gas (b) it decays much slower than U-23 8
(c) it enters our lungs with the air we breathe
(d) The statement is incorrect. Radon-222 poses lesser hazards than U-238

Box 9D
26. The hydroxyl free radical (-O-H) is formed when high energy radiation interacts
with
(a) oxygen (b) ozone (c) water (d) hydrogen

Box 9E
27. The particles employed in PET scans are
(a) protons (b) positrons (c) electrons (d) neutrons

Box 9F
28. According to Einstein' s equation 1 mg of matter converted to energy can yield
3
(a) 8 . 8 1 0 1 J (b) 8 . 8 1 0 1 0 J (c) 8 . 8 1 07 J (d) 3 1 08 J
x x X x

29. Which statement best explains how the fusion of deuterium and tritium nuclei
into helium nuclei provides the enormous quantity of energy that comes from our
sun?
(a) the mass of the reactant nuclei is somewhat less than the mass of the
product nuclei
(b) the lost mass is converted to energy according to Einstein' s equation
(c) deuterium nuclei are not very stable
(d) all of these answers

Box 9G
30. Why were pills of potassium iodide (KI) given to children who were exposed to
radioactive fallout from the Chemobyl accident?
(a) KI is a salt substitute and NaCI was affected by the radiation
(b) the nonradioactive iodide in the pills diluted the radioactive iodide that
came from the fallout
(c) KI absorbs radiation (d) none of these

Chapter 10 of "Introduction to General, Organic, and Biochemistry'; fifth edition:


Organic Chemistry. Alkanes

1 . The synthesis of which organic compound first disproved the doctrine of "vital
force"?
(a) uric acid (b) urea (c) CO2 (d) vitamin A

39
s.aders CoUege Publishing
2. Organic compounds are generally obtained by
(a) synthesis only (b) isolation and/or synthesis
(c) isolation only (d) using petroleum as the only source

3 . Which element normally forms 3 bonds?


(a) oxygen (b) bromine (.c) nitrogen (d) carbon

4. Which of these properties is characteristic of organic compounds?


(a) bonding that is mostly ionic (b) a melting point above 3 00 ° C
(c) generally slow reactions (d) insolubility in organic solvents
3
5. Which statement i s false? An Sp orbital
(a) can contain two electrons (b) has two lobes of equal size
(c) can overlap with an s orbital (d) is a hybrid orbital

6. Hydrocarbons that contain at least one double bond are called


(a) aromatic (b) alkanes (c) alkenes (d) alkynes

7. The compound CH3(CH2) sCH3 is called


(a) n-hexane (b) isohexane (c) n-heptane (d) n-octane

8. The starred carbon in this structure CH3-*CH - CH3 is


I
CH3
(a) primary (b) secondary (c) tertiary (d) quaternary

9 . Among these compounds, which are isomers?


1 . CH3CH2 - O-CH3 2. CH3CH2-O-CH2CH3
3 . CH 3 C H 2 C H2 0 H 4. CH2=CH-O-CH3
(a) 1 and 3 (b) 2 and 3 (c) 1 and 2 (d) 1 and 4
1 0 . CH 3 CH3 CH3
I I I
CH2- CH2-CH-CH2CH3 and CH3CH2-CH-CH2CH2CH3 are
( a) structural isomers (b) identical molecules
(c) different molecules that are not isomers (d) none of these

1 1 . The IU PAC name for CH3CH2CH2-CH-CH2-CH-CH3 IS


I I
CH3 CH2
I
CH 3
(a) 6-ethyl-4-methylheptane (b) 4,6-dimethyloctane
(c) 2-ethyl-4-methylheptane (d) 3 , 5 -dimethyloctane

40
e Saunders CoUege Publishing
1 2 . The name of the group CH3CH2-CH- is
I
CH3
(a) isobutyl (b) sec-butyl (c) tert-butyl (d) isopropyl

1 3 . 1 ,2-Dibromo-2-methylbutane has the structure


(a) CH3-CH-CH2-Br
I
CH2CH2-Br
CH2CH3
I
(c) CH2-C-Br (d) Br-CH2-CH-CH-CH3
I I I I
Br CH3 Br CH3
14. In naming alkyl halides containing four different halogens which prefix is cited
first?
(a) fluoro (b) chloro (c) bromo (d) iodo

15. is called

(a) l -chloro-3 -bromocyc1opentane


(b) l -bromo-2-chlorocyc1ohexane
( c) l -bromo-3 -chlorocyclopentane
(d) I -bromo-2-chlorocyc1opentane

16.

(a) structural isomers (b) stereoisomers


(c) different molecules that are not isomers (d) identical molecules

1 7 . How many stereoisomers o f 1 ,4-dibromocyclohexane are there?


( a) 1 (b) 2 (c) 4 (d) 6

1 8 . In which of these solvents are alkanes least soluble?


(a) benzene (b) chloroform (c) water (d) dimethyl ether

41
e Saunders CoUege Publishing
1 9. The number of structural isomers with the formula C3H 6 CIBr is
(a) 4 (b) 5 (c) 6 (d) 7

20. The combustion of one mole of cyclohexane, C 6 H 1 2, produces how many


moles of CO2 ?
(a) 3 (b) 6 (c) 1 2 (d) 8

2 1 . Compounds with the formula R-O-R are


(a) ketones (b) ethers (c) aldehydes (d) esters

22. In 2-butanol the CH3CH2CHCH3 part of the molecule


(a) is the functional group (b) is the alcohol group
(c) is the alkyl group (d) contains only primary carbons

23 . What functional groups are present in CH3-CH=CH-CH2-OH?


(a) alcohol and alkyne (b) ketone and alkene
(c) aldehyde and ketone (d) alcohol and alkene

24. Which of these is a ketone?


(a) CH3CH2-CH-CH3 (b) CH3-C-CH2CH3
I "
OH 0
(d) CH3-O-CH2CH3
B ox 1 0A
25 . A synthesis in which one obtains as many compounds from as few building
blocks as possible is called
(a) combinatorial (b) rational (c) shotgun (d) unique

Box 1 0 B
26. The most stable shape of the cyc10hexane ring resembles a
(a) chair (b) table (c) boat (d) flat plane

Box 1 0C
27. If you put a few drops of hexane on your finger and then wipe it off, you will feel
the skin dry up . This happens because the hexane
(a) is poisonous (b) protects the skin
(c) dissolves the natural skin oils (d) extracts the water from the skin

Box 1 00
28. Which hydrocarbons are responsible for the " knoc king" in automobile engines?
(a) straight chain (normal) (b) highly branched
(c) long chain ( above 1 0 carbons) (d) short chain (below 5 carbons)

42

c) Saunders College Publishing


Box 1 0E
29. Carbon dioxide is
(a) produced by incomplete combustion of fuels (b) more poisonous than CO
(c) insoluble in water (d) a minor component of our atmosphere

Box 1 0F
30 . Which statement is incorrect?
(a) the accumulation of CO2 in the atmosphere may cause the greenhouse effect
(b) CO2 in the air reflects back the earth' s infrared radiation
(c) the amount of CO2 in our atmosphere is gradually increasing
(d) CO2, once formed, remains in the atmosphere forever

Box 1 0G
3 1 . Taxol is a promising drug in treatment of
(a) ovarian cancer (b) heart attack (c) stroke (d) prostate cancer

Chapter 1 1 of "Introduction to General, Organic, and Biochemistry': fifth edition;


Chapter 3 of "Introduction to Organic and Biochemistry ': third edition:
Alkenes, Alkynes, and Aromatic Compounds

1. CH3 -CH-C-CH2-CH, is called



I II
Br CH2
(a) 2-bromo-3 -methyl- l -pentene
(b) 3 -bromo-2-ethyl- l -butene
(c) 2-bromo- l -ethyl-2-butene
(d) 3 -bromo-2-methyl- l -pentene

2. aCH1 is called

(a) I -methyl-2,5-cyclohexadiene
(b) 3 -methyl- l ,4-cyclohexadiene
(c) I -methyl-2, 6-cyclohexadiene
(d) 5 -methyl- l ,4-cyclohexadiene

3 . Which statement i s false?


(a) another name for alkene is olefin
(b) propylene and propene are two names for the same compound
(c) a compound with the formula CsH12 can be an alkene
(d) compounds with double bonds are given names ending with ene

43

e Saunders College Publishing


4. CH3-CH=CH-CH=CH-CH=CH-CH3 is called
(a) 2,4,6-octene (b) 2,4, 6-octatriene (c) 1 , 3 , 5 -octadiene (d) 3 , 5 , 7-octatriene

5. The vinyl group i s


(a) CH2=CH-CH2- (b) CH3-CH=CH- (c) CH2=CH- (d) CH2=C=CH-

6. A C=C double bond is made


(a) of a sigma bond and a pi bond (b) of two sigma
2 bonds
(c) of two pi bonds (d) by the overlap of four Sp orbitals

7. The compound 2-methyl-2-butene has


(a) cis and trans stereoisomers
(b) completely free rotation around all carbon- carbon bonds
(c) 1 20 ° angles at the carbon - carbon double bonds
(d) only sigma bonds
8 . Which o f these compounds exist a s cis and trans isomers?
1 . propene 2. I -butene 3 . 2-pentene 4 . 2 - methyl - 2 -pentene
(a) only 4 (b) 3 and 4 (c) all of them (d) only 3

9. The six atoms of a C=C double bond remain in one plane because
(a) if one of the p orbitals rotated
3 the bond would break
(b) the two carbon atoms use Sp orbitals
(c) the geometry is tetrahedral
(d) all of these

10. Which of these will not give an addition product when added to I -butene?
(a) F2 (b) Br2 (c) HBr (d) Cl2

1 1 . Which of these will give a negative test when treated with Br2 in carbon
tetrachloride?
(a) CH3CH=CHCH3 (b) CH3CH2CH2CH2CH3 (c) CH3 CH=CHCH2CH=CHCH3
(d)

o
1 2 . When HCI is added to I - methy1cyclohexene the principal reaction product is
(a) I -chloro- l - methylcyclohexane
(b) l -chloro-2-methylcyclohexane
(c) I -chloro-2-methylcyclohexene
(d) 3 -chloro-2-methylcyclohexene

44

© Saunders CoUege Publishing


1 3 . Which reagent will bring about the conversion of fumaric acid to malic acid:
HOOC " H

/
C=C " -- HOOC-CH-CH2-COOH
/
H COOH I
OH
(a) H20 + H+ (b) Br2 (c) HOBr (d) HI

14. In the addition of HX to a double bond, the hydrogen goes to the carbon that
already has more hydrogens . This is a statement of
(a) the double bond rule (b) Markovnikov ' s rule (c) LeChatelier' s principle
(d) the rule of "less is better"

1 5 . The addition polymer -CH2-CH-CH2-CH- is called


I I
CN CN
(a) teflon (b) polypropylene (c) polyacrylonitrile (d) polystyrene

1 6. The polymer polyvinyl chloride is made by polymerizing which monomer?


(a) CH3 -CH2CI (b) CH2=CCI2 (c) CHCI=CHCI (d) CH2=CHCI

1 7 . S aran is an example of a polymer made of more than one kind of monomer


(vinylidine chloride and vinyl chloride) . This kind of polymer is called a
(a) block polymer (b) branched polymer (c) copolymer
(d) condensation polymer

1 8 . The entity CI-CH=CH2 in the manufacturing of polyvinyl chloride is called


(a) a branched polymer (b) a monomer (c) a repeating unit (d) a dimer

1 9 . The compound CH3-C = C-CH2CH2-CI is called


(a) l -chloro-4-pentyne (b ) l -chloro-3 -pentyne
(c) 5-chloro-2-pentene (d) 5-chloro-2-pentyne

20. An sp hybrid orbital can be found


(a) at a triple bond carbon (b) at a double bond carbon
( c) at a single bond carbon (d) at a benzene ring carbon

2 1 . Which of these statements about the structure of benzene is mise ?


(a) it contains an aromatic loop (sextet) of electrons
(b) the 1 2 atoms of benzene all lie in one plane
(c) the 6 carbon-carbon bonds have different lengths
(d) the benzene ring is greatly stabilized by resonance

45
e Saunders CoUege Publishing
22. CI----©--CI is called

(a) ortho-dichlorobenzene (b) para-dichlorobenzene

BT6l
(c) meta-dichlorobenzene (d) dichlorophenol

23 . is called

"r'Br
NH2
(a) para-dibromoaniline (b) 2,4-dibromoaniline
( c) 2,5-dibromotoluene (d) 2,5-dibromoaniline

24. The group formed when one hydrogen atom is removed from a benzene ring is
called
(a) phenyl (b) benzyl (c) cyclohexyl (d) toluyl

2 5 . Heating benzene with concentrated sulfuric acid gives


(a) benzene sulfonic acid (b) benzyl sulfate
(c) phenyl sulfate (d) chi oro benzene

26. Which of these is a heterocyclic nonaromatic compound that contains nitrogen in


the ring?
( a) tetrahydropyran (b) pi peri dine (c) pyridine (d) pyrimidine

27. Which of the following are heterocyclic aromatic compounds?

l. 3· CO� 4.

� 1 #

(a) 3 and 4 (b) 4 only (c) 2 and 4 (d) all four

Box 1 1A
2 8 . Which statement is incorrect ?
(a) vitamin A has 5 double bonds
(b) vitamin A has four isoprene units
(c) the body can obtain ':i Lamin A from p-carotene
(d) vitamin A is the same as p-carotene

46

© Saunders College Publishing


Box 1 1 B
29. Pheromones
(a) usually have at least fi ve functional groups per molecule
(b) are used by insects as sex attractants
( c) are most often produced by male insects
(d) are not species specific

Box 1 1 C
3 0. Which of these names refers to visual pigments?
(a) l l - cis-retinal (b) all-trans-retinal (c) vitamin A (d) rhodopsin

3 1 . Which statement is true?


(a) Only rods are used in color vision.
(b) Rods are found only in the central part of the retina.
(c) The eye has different retinals for blue, green and red colors.
(d) The cones have different opsins for blue, green and red.

Box 1 1 D
32. Which statement is false ? Ethylene is
(a) not very reactive (b) used in warehouses to ripen fruits
(c) obtained from petroleum (d) acts as a messenger in animal cells

Box 1 1 E
3 3 . The intermediate that forms in the addition of HBr to an alkene is called
(a) an olefin (b) a carbocation (c) a hydronium ion (d) a hydrogen ion

Box 1 1 F
34. Natural rubber is
(a) a polymer that contains alternating cis and trans double bonds
(b) all- trans polyisopr e ne (c) all-cis-polyisoprene (d) polyisopropylene
-

Box 1 1 G
3 5 . All these statements about fused aromatic hydrocarbons with at least four rings
and one angular junction are true except
(a) they have more hydrogen than carbon atoms (b) they cause cancer
(c) they are present in cigarette smoke (d) they are present in chimney soot

Chapter 12 of "Introduction to General, Organic, and Biochemistry'; fifth edition;


Chapter 4 of "Introduction to Organic and Biochemistry'; third edition:
Alcohols, Phenols, Ethers, and Halides

1 . Which of these compounds i s not an alcohol?


(a) CH3-CH-OH (b) HO-CH2CH2-OH (c)
I
CH3

47

C Saunders College Publishing


2. The IUPAC name for CH3CH2-CH-CH2CH2CH3 is
I
CH2-OH
(a) I -heptanol (b) 2-ethyl- l -pentanol
(c) 2-propyl- l -butanol (d) 3 -methyl- l -hexanol

3. Which o f these are secondary alcohols?


1.
o-0H 2 . HO-CH2-CH-CH2-OH
I
CH3
(a) 2 only (b) 3 only (c) 1 and 3 (d) 1 and 2

4. An example of an important triol is


(a) glycerol (b) ethylene glycol (c) methyl alcohol (d) isopropyl alcohol

5 . Zaitsev' s rule applies to


(a) oxidation of alcohols to aldehydes
(b) oxidation of alcohols to carboxylic acids
(c) oxidation of alcohols by KMn04 (d) dehydration of alcohols to alkenes

6. When 2-hexanol is dehydrated the maj or product is


(a) I -hexene (b) 2-hexene (c) 3 -hexene (d) 1 , 3 -hexadiene

7. Which of these reactions can be undergone by all alcohols?


(a) dehydration to an alkene (b) oxidation to a ketone
(c) oxidation to an aldehyde (d) none of these

8. In the conversion of wine to vinegar


(a) ethyl alcohol is oxidized to acetic acid
(b) ethyl alcohol is reduced to acetic acid
(c) ethyl alcohol is oxi dized to formic acid
(d) methyl alcohol is reduced to formic acid

9. Which alcohol is an important component of alcoholic beverages?


(a) CH3CH20H (b) CH3 0H (c) HOCH2CH20H (d) CH3-CH-CH3
I
OH
10. Which compound is used as an antifreeze in automobiles?
(a) isopropyl alcohol (b) glycerol (c) ethanol (d) ethylene glycol

1 1 . An alcohol that is nontoxic is


(a) methanol (b) glycerol (c) wood alcohol (d) ethylene glycol

48

e Saunders College Publishing


12. The most important chemical property of phenols is
(a) they are weak bases (b) they can be oxidized to ketones
(c) they are weak acids (d) they are strong acids

13. The structure of the antioxidant BHT (butylated hydroxytoluene) is

(c) (d)

14. Another name for meta-cresol is


(a) 2-methyltoluene (b) 3 -methyltoluene
(c) 4-methyltoluene (d) 3 -methylphenol

15. The most important ether is


(a) diphenyl ether (b) dimethyl ether (c) diethyl ether (d) ethyl methyl ether

16. The smallest heterocyclic ether is


(a) dioxane (b) ethylene oxide (c) dimethyl ether (d) tetrahydrofuran

1 7. Which ether is added to gasoline to reduce carbon monoxide pollution?


(a) methyl tert-butyl ether (b) dimethyl ether
(c) diethyl ether (d) cyc10hexyl phenyl ether

18. Which of these is the least soluble in water?


(a) CH30H (b) CH3CH20H (c) CHlCH2)4CH20H (d) HOCH2(CH2)4CH20H

19. Which of these has the lowest boiling point?


( a) CH3-O-(CH2 )sCH3 (b) CH3(CH2)4CH2 0H (c) phenol (d) HO-CH2CH2-OH

49

C Saunders College Publishing


20. From a knowledge of which compounds form hydrogen bonds, we can conclude,
comparing compounds of the same molecular weight, that
(a) alcohols are more soluble in water than ethers
(b) alcohols have higher boiling points than ethers
(c) alkyl chlorides have higher boiling points than alcohols
(d) ethers have lower boiling points than alkanes

2 1 . Which group of compounds is noted for their foul odors?


(a) phenols (b) ethers (c) alcohols (d) thiols

22. Which statement is correct?


(a) thiols can be easily reduced to disulfides
(b) disulfides have two SH functional groups
(c) disulfides can be easily reduced to thiols
(d) disulfides can be easily oxidized to thiols

23 . The most common use of CI2C=CCI2 is as a


(a) local anesthetic (b) dlY cleaning solvent (c) refrigerant (d) disinfectant

24. CFCs, such as Freon- 1 2, formerly used in refrigerators and aerosol sprays, have
been replaced by internati onal agreement. This was done because CFCs
(a) destroy the protective ozone layer in the upper atmosphere
(b) are poisonous
(c) contain fluorine, which when released is poisonous
(d) The statement is false. CFCs are still used in aerosol sprays.

B ox 1 2A
25 . The maximum alcohol cO�lcentration in an alcoholic beverage obtained by
fermentation is
(a) 5% (b) 1 3 % (c) 25% (d) 40%

Box 1 2 8
26. The compound disulfuram
(a) is usually a permanent cure for alcohol addiction
(b) is a stimulant (c) is a depressant
(d) works by preventing acetaldehyde from being oxidized

27. Which of these statements about methanol is false?


(a) methanol causes blindnes s
(b) there is n o antidote for methanol poisoning
(c) methanol in the body is oxidized to formaldehyde
(d) methanol in the blood causes acidosis

Box 1 2 C
28. What functional group(s) IS (are) present in nitroglycerine?
(a) ester (b) ether (c) ether and ester (d) ketone

50

© Saunders CoUege Publishing


Box 1 2 D
29. Which phenol is used as an antiseptic in throat lozenges?
(a) methyl salicylate (b) thymol (c) 2-chlorophenol
(d) 4-n-hexylresorcinol

Box 1 2 E
3 0 . Which of these anesthetics is not an ether but a halide?
( a) enflurane (b) isoflurane (c) halothane (d) nitrous oxide

Box 1 2 F
3 1 . Which statement about deodorants is false?
(a) body odor is caused by bacterial action on body secretions
(b) modem deodorants contain the antibacterial agent hexachlorophene
(c) modem deodorants contain the antibacterial agent triclosan
(d) modem deodorants are phenols and ethers as well as halides

Box 1 2G
3 2 . The active ingredient in marijuana is
(a) diethyl ether (b) ethanol (c) phenol (d) tetrahydrocannabinol

Box 1 2 H
3 3 . Which of these statements is false?
(a) DDT is biodegradable (b) chlordane is an effective insecticide
(c) the use of DDT has increased crop yields
(d) because of widespread use, DDT has even been found in mother ' s milk

Box 1 2 1
34. Goiter is caused by a deficiency in the hormone
(a) iodine (b) insulin (c) threonine (d) thyroxine

Chapter 13 of "Introduction to General, Organic, and Biochemistry'; fifth edition;


Chapter 5 of "Introduction to Organic and Biochemistry '; third edition :
Aldehydes and Ketones

1 . Which of these classes of compounds does not contain a carbonyl group?


(a) ether (b) ketone (c) aldehyde (d) carboxylic acid

Which o f these compounds is neither an aldehyde nor a ketone?


ero
2.
(a) CH3 -C-CH3 (b) CH3 -C-OCH3 (c) CH3-CH-C-H (d)
II II I II
o 0 CH3 0

51
, s-.ders College Publishing
3. This group -C- i s called
II
o
(a) aldehydo (b) carboxyl (c) keto (d) carbonyl

4. The IUP AC name for formaldehyde is


(a) methanal (b) ethanal (c) butanone (d) propanone

S. The IUPAC name of the compound CH3CH2-C-CH-CH=CH-CH3 is


II I
o Cl
(a) 4-chloro-2-hepten-S-one (b) 4-chloro-S-hexen-3 -one
(c) 4-chloro-S-hepten-3 -one (d) 4-chloro-3-butenyl ethyl ketone

6. Which o f these names for CH3CH2-C-CH2CH3 i s incorrect?


II
o
(a) diethyl ketone (b) 3 -pentanone (c) 3 -carbonylpentane
(d) all three names are correct
7. Which compounds have generally pleasant odors and are used in perfumes?
(a) alkanes (b) alcohols (c) aldehydes (d) ketones

8. Which of these is least soluble in water?


(a) CH3-C-H (b) CH3 (CH2)6 -C-H (c) CH3CH20H (d) HO-CH2CH2-OH
II II
o 0
9. Which of these has the lowest boiling point?
(a) CH3CH20H (b) H-C-H (c) CH3-C-CH3 (d) HO-CH2CH2-OH
II /I
o 0
1 0 . When 2-methyl-2-propanol is treated with a mild oxidizing agent the oxidation
product is
(a) an aldehyde (b) a ketone (c) an ether (d) there is no reaction

1 1 . The principal product obtained by mild oxidation of CH3 CH2CH2-C-H is


/I
o
(a) CH3CH2-O-CH2CH3 (b) CH3CH2CH2COOH (c) CH3CH2CH2CH2-OH
(d) CH3-O-CH2CH2CH3

1 2 . In Benedict ' s test


(a) an aldehyde is oxidized (b) a silver mirror forms
(c) the copper(II) ion i s oxidized (d) all of these

52

© Saunders College Publishing


1 3 . Which of these will give a positive Tollens ' test?
(a) CH3-C-CH3 (b) CH3-O-CH2CH3 (c) CH3-C-CH-CH3 (d)
II II I
(f0
o O OH

14. A hemiacetal has which of these groups on the same carbon?


(a) OH and OR (b) two ORs (c) two OHs (d) three ORs

1 5 . Which of these is not an acetal?


(a) CH3CH2-CH-OCH3 (b) CH3-CH-OCH3 (c)
I I
OCH2CH3 OH

Q- CH2-r-H 2CH,OH
(d) all three of these are acetals
16. + � The product of this reaction is

?CH3
o

Q-CH2-� H
H
(a)

1 7 . The cyclic hemiacetal fonned from HO-CH2CH2CH2CH2-C-H

C':>-.OH 0
has the structure II
o
( aJ

Q
(b) (c) (d)

18.
o
OCH3
0 0 OH �CH'
I
CH3-C-H
I
OCH3
When the above compound is hydrolyzed with an acid catalyst, the products are
CH30H and
(a) CH3-C-H (b) CH3-C-CH3 (c) CH3CH2-C-H (d) CH3CH20H
II " 1/
0 0 0

53
19. Which of these can form a cyclic hemiacetal?
(a) CH3 -C-CH2CH2CH2-C-H (b) HO-CH2-C-CH2-OH
II II 1/
0 0 0
(c) HO-CH2CH2CH2CH2-C-H (d) H-C-CH2CH2CH2-C-H
II 1/ II
0 0 0

20. In keto-enol tautomerism of simple compounds which compound is more stable,


in most case?
(a) the enol form (b) the keto form (c) both are equally stable
(d) neither one can normally be isolated

21. An aqueous solution of formaldehyde, called formalin, is used to embalm


cadavers. This use has been declining because
(a) formalin smells bad (b) formalin hardens the cadavers too much
(c) most bacteria now resist formalin (d) formalin is carcinogenic

22. Which compound is used in nail polish?


(a) paraldehyde (b) acetaldehyde (c) acetone (d) propanal

23 . Which aldehyde or ketone is used as a tear gas in Mace?


(a) benzaldehyde (b) vanillin (c) muscone (d) a-chloroacetophenone

Box 1 3A
24. The compound chloral hydrate is used as
(a) a weight-reducing drug (b) a sleeping tablet (c) an antiseptic
(d) a compound used to treat alcoholism

Box 1 3B
25. Bakelite, Formica and Melmac are trade names for three dimensional giant
molecules (polymers) . In all these the common monomer is
(a) acetaldehyde (b) acetic acid (c) acetone (d) formaldehyde

Box 1 3C
26. Which of these has the lowest LD50 (that is, is the most toxic)?
(a) nicotine (b) NaCI (c) arsenic (d) acetaldehyde

Box 1 30
27. Which of these is used as a flavoring for margarine?
(a) biacetyl (b) benzaldehyde (c) muscone (d) vanillin

54

C Saunders CoUege Publishing


Chapter 14 of "Introduction to General, Organic, and Biochemistry'; fifth edition;
Chapter 7 of "Introduction to Organic and Biochemistry'; third edition:
Carboxylic Acids and Esters

1. This compound CH3CH2CH2COOH is called


(a) propionic acid (b) palmitic acid (c) valeric acid (d) butyric acid

Questions 2 and 3 refer to these choices:


(a) CH3 -C-CH3 (b) CH3 -C-OCH3 (c) CH3 -C-H (d) CH3 -C-OH
II " 11 "
o 0 0 0
2 . Which is a carboxylic acid?

3 . Which i s a carboxylic ester?

4. Citric acid is
(a) a monocarboxylic acid (b) a dicarboxylic acid
(c) a dicarboxylic hydroxy acid (d) a tricarboxylic hydroxy acid

5. Which statement i s true? Carboxylic acids


(a) have higher boiling points than alcohols of the same molecular weight
(b) are all soluble in water (c) have weaker hydrogen bonds than alcohols
(d) cannot form hydrogen bonds with another carboxylic acid

6. Which of these bases will convert a carboxylic acid to its salt?


(a) OH- (b) NH3 (c) C032- (d) all of these

7. If propionic acid CH3CHzCOOH is dissolved in water, and the pH is raised to 1 2,


what statement can we be sure of ?
(a) only CH3CH2COOH and not CH3 CH2COO- is present
(b) only CH 3 CH2COO- and not CH3 CH2COOH is present
(c) equal quantities of both are present
(d) under these conditions CH3CH2 COOH has been reduced to
CH3 CH2CH2 0H

8. This compound CH3 CH2COO- K+ i s called potassium


( a) propionate (b) benzoate (c) acetate (d) butyrate

9. Benzoic acid and diphenyl ketone are both solids insoluble in water. A mixture of
these compounds can be separated if we
(a) dissolve the mixture in benzene and boil off the benzene
(b) add the mixture to a solution of HCI in water, and filter
(c) add the mixture to a solution of NaOH in water, and filter
(d) it is impossible to separate a mixture of two solids when both are
insoluble in water

55
Saunders CoUege Publishing
1 0 . Which of these compounds is most soluble in water?
(a) CH3 CH2CH2CH2CH2 COOH (b) CH3 CH2 CH2 CH2COO- Na+
(c) CH3CH2CH2CH2COOH (d) CH3CH2CH2CH2CH20H

1 1 . This compound CH3 CH2CH2CH2-0-C-CH2CH3 is called


II
o
(a) benzyl propionate (b) propyl butyrate
(c) n-propyl propionate (d) n-butyl propionate

H+
12. CH3-CH-C-OH + CH30H
I "
CH3 0
The product of this reaction i s
(a) CH3-CH -C-OCH3 (b) CH3-CH-O-C-CH3
I " I II
CH3 0 CH3 0
(c) CH3-CH-C-CH3 (d) CH3-CH-O-C-OCH3
I II I /I
CH3 0 CH3 0
1 3 . Which of these, when combined with CH3 CH2CH20H and an acid catalyst, will
form n-propyl formate?
(a) CH3 COOH (b) CH3CH2COOH (c) HCOOH (d) none of these

14. Which of these has the lowest boiling point?


(a) CH3COOH (b) HCOOCH3 (c) CH3CH2COOH
(d) They all have the same boiling point

1 5 . Which of these is an anhydride?


(a) CH3-C-CI (b) CH3 -C-O-C-CH3 (c) CH3-C-OCH3 (d) CH3-C-CH3
II II II II II
o 0 0 0 0

1 6 . In preparing a carboxylic ester, which pair of compounds will give the highest
amount of product?
(a) an acyl chloride and an alcohol (b) a carboxylic acid and an alcohol
(c) two different carboxylic acids (d) two different alCohols

1 7 . The hydrolysis of a certain compound produces HCOOH and CH3CH20H in


equal molar amounts. The compound must be
(a) CH3-C-OCH3 (b) H-C-OCH3 (c) CH3CH2-C-CH3 (d) H-C-OCH2CH3
II II 1\ \I
o 0 0 0

56

© Saunders College Publishing


18. The hydrolysis of carboxylic esters in the presence of a base is called
(a) dehydration (b) esterification (c) saponification (d) acidifi c ation
+ �
ow
19. CH3 CH2-C-OCH3 H20
II
o
The products of this reaction are
(a) CH3COO- and CH3CH20H (b) CH3CH2COo- and CH30H
(c) CH3CH2COOH and CH30H (d) CH3COOH and CH3CH20H

20. Pyrophosphoric acid, an anhydride of phosphoric acid, has the following structure
o 0 0 0 0 0 0
II II II II II II "
(a) -0-P-OCH3 (b) HO-P-O-P-OH (c) -O-P-P-O- (d) HO-P-0-CH2-O-P-OH
I I I I I 1 I
0- OH OH 0- 0- OH OH
21. This compound 0 0 IS
II II
HO-P-O-P-OCH2CH3
I I
OH OH
(a) an ester only (b) an anhydride only (c) both an ester and an anhydride
(d) neither an ester nor an anhydride

22. Nitroglycerine is
(a) an inorganic ester (b) an anhydride of nitric acid
(c) a mixed anhydride of nitric and sulfuric acids
(d) made from glycerol and sulfuric acid

Box 1 4A
23 . all-trans-Retinoic acid is a natural m orph o gen . This means that
(a) it helps to develop epithelial tis sues
(b) it helps to promote night vision
(c) it helps the development of limbs in embryos
(d) all of these

Box 1 4 B
24. Calcium propionate is used as a
(a) soothing ingredient against insect bites
(b) antifungal agent against athlete ' s foot
(c) prevention of diaper rash (d) food preservative

Box 14C
Methyl anthranilate is used to supply the flavor of
25 .
(a) apples (b) bananas (c) grapes (d) pineapples

57

Ie Saunders College Publishing


B ox 1 4 D
26. Which statement is false?
(a) aspirin is both an ester and a carboxylic acid
(b) aspirin is made by esterification of benzoic acid with acetic anhydride
(c) the origin of aspirin goes back centuries to when doctors recommended
chewing the bark of willow trees to relieve pain
(d) aspirin reduces fever and swelling

B ox 1 4 E
27. The two monomers used to manufacture Dacron are
(a) ethylene glycol and succinic acid (b) glycerol and succinic acid
(c) glycerol and terephthalic acid (d) ethylene glycol and terephthalic acid

2 8 . Polyesters belong to a class of compounds called


(a) addition polymers (b) condensation polymers
(c) three dimensional networks (d) all of these

Box 1 4 F
29. Polyesters made of lactic acid and glycolic acid
(a) can be used as absorbable staples in surgery
(b) are used as a fiber in carpets (c) are used as tire cords
(d) are manufactured as s!leets under the name of Mylar and used in computer
tapes.

Chapter 15 of "Introduction to General, Organic, and Biochemistry': fifth edition;


Chapter 8 of "Introduction to Organic and Biochemistry': third edition:
Amines and Amides

1. An example o f a primary amine is


(a) CH3 -NH-CH2CH3 (b) (CH3)2NH (c) CH3CH2NH2 (d) (CH3)3N

2. The name o f this compound IS

(a) N-ethylaniline (b) ethylanilinamine


(c) ortho-phenylethylamine (d) methyl phenyl ether

3. Pyrimidine is
(a) a primary amine (b) a heterocyclic aromatic amine
(c) a heterocyclic nonaromatic amine (d) an aliphatic tertiary amine

58

© Saunders College Publishing


4. The boiling point of ethylamine is higher than that of propane but lower than that
of I -propanol. This means that
(a) a molecule of ethyl amine cannot form a hydrogen bond with another
molecule of itself
(b) hydrogen bonds between two amino groups are weaker than those between
two alcohol groups
(c) the intermolecular attraction between two amine molecules i s merely dipo le ­

dipole interaction
(d) the intermolecular interaction between two amines is of an ionic nature

5. Which of these is most soluble i n water?


(a) CH3CH2CH2NHCH3 (b) CH3CH2CH2CH2NH2 (c) (CH3CH2)3N
(d) NH2CH 2CH2CH2NH2

6. Cadaverine
(a) is a monoamine with four carbons (b) has a bad odor
(c) is a heterocyclic compound (d) all of these

7. Which of these types of compounds forms salts with acids?


(a) ketones (b) amines (c) alcohols (d) carboxylic esters

8. The product formed when H C I reacts with (CH3)2NH is


(a) CH3CH2NH3+ CI- (b) CH3NH2+ CI- (c) (CH3)2NH2+ CI- (d) (CH3)2N- CI-

9. CH2CH3 is an example of a
+ '
CH3-N-CH2CH3 F-
I
CH2CH3
(a) tertiary amine (b) quaternary amine
(c) tertiary ammonium salt (d) quaternary ammonium salt

1 0 . The reaction of a carboxylic acid with an amine yields


(a) a salt (b) a substituted amine (c) an amide (d) there is no reaction

1 1 . An amine that is insoluble in water can be made to dissolve by adding it to an


aqueous solution of
(a) NaOH (b) HCI (c) a different amine (d) an amide

1 2 . Which of the following can be classified as an amide?


(a) CH3CH2-C-CH3 (b) CH3 CH2NHCH3 (c) CH3CH2-C-NH-CH3 (d) CH3C=N
/I II
o 0

59

Saunders CoUege Publishing


13. ��-NH2 is called

(a) benzamide (b) phenylformamide (c) aminobenzoic acid


(d) N-phenylacetamide

14. Amides are


(a) acidic (b) basic (c) neutral (d) some are basic, some neutral

15. When CH3 -C-O-C-CH3 reacts with CH3CH2NH2 the products are
II II
o 0
(a) CH3-C-NH-CH2CH3 + H20 (b) CH3-C-NH-CH2CH3 + CH3COOH
� �
(c) CH3CH2-C-NH-CH3 + H20 (d) CH3CH2-C-NH-CH3 + CH3COOH
II II
o 0
1 6 . When CH3-C-NH-CH3 is hydrolyzed in basic solution the products are
II
o
(a) CH3COOH and CH3NH2 (b) CH3COO- and CH3NH2
(c) CH3 COOH and CH3NH3+ (d) CH3 COO- and CH3NH3+

1 7.
When <Q)--- � -NH2 is hydrolyzed in acid solution the products are
o

(a) (0) and HCOOH

(c) <Q)--- C OOH and NH3 (d) <Q)--- C OOH and N�+

1 8 . An alkaloid is best descrihed as any


(a) basic compound obtained from a plant
(b) tertialY amine obtained from a plant
(c) basic compound obtained from either an animal or a plant
(d) diamine obtained from an animal

60

© Saunders CoUege Publishing


1 9 . The poisonous alkaloid in the hemlock plant is
(a) atropine (b) strychnine (c) coniine (d) codeine

20. Nicotine
(a) is found in coffee as well as tobacco
(b) has both a heterocyclic aliphatic ring and a heterocyclic aromatic ring
(c) is a primary amine (d) is a stimulant only when ingested by chewing

21. The extract of the plant belladonna contains a compound that dilates the pupils of
the eye. This compound is
(a) coniine (b) morphine (c) quinine (d) atropine

22. Heroin
(a) is a synthetic ester made from morphine
(b) is isolated from the henbane plant
(c) reaches the brain much more slowly than morphine
(d) all of these

23 . Which of these is not a pain killer?


(a) Demerol (b) quinine (c) morphine (d) codeine

Box 1 5A
24 Amphetamines in "pep pills" have structures similar to the hormone
(a) insulin (b) glucagon (c) adrenalin (d) progesterone

Box 1 5 B
25 . The designation ' Hel as in Neo-Synephrine · Hel on the label of a decongestant
drug means the drug
(a) contains hydrochloric acid
(b) is an amine and is present in the form of its salt
(c) was extracted from a plant using Hel (d) is not pure

Box 1 5C
26. Which among the common non-prescription pain killers is an amide?
(a) acetaminophen (b) aspirin (c) ibuprofen (d) all of these

27. Some of the non-prescription pain relievers are also antiinflammatory agents.
Which pain killer does not reduce inflammation?
(a) naproxen (b) aspirin (c) ibuprofen (d) acetaminophen

Box 1 5 D
28. Nylon belongs to a class of polymers known as
(a) polyamides (b) polyesters (c) polystyrenes (d) polyethylenes

61

() Saunders College Publishing


Box 1 5E
29. Which statement about urea is false? Urea is
(a) an amine (b) a diamide (c) used as a fertilizer
(d) used by higher animals to get rid of nitrogenous waste material

30. B arbiturates are


(a) aromatic amines (b) aromatic amides
(c) heterocyclic amines (d) heterocyclic amides

B ox 1 5 F
3 1 . Tranquilizers of the diazepam class (for example, Valium) have an amide group
(a) in a six membered heterocyclic ring
(b) in an aliphatic chain
(c) in a seven membered heterocyclic ring
(d) in an eight membered heterocyclic ring

Box 1 5G
32. Which statement about cocaine is false?
(a) it is found in coca leaves
(b) its use as a local anesthetic in tooth extraction is legal
(c) addiction to cocaine can be transferred from a mother to her baby
(d) "crack" is a smokable form of cocaine

Chapter 16 of "Introduction to General, Organic, and Biochemistry'; fifth edition;


Chapter 9 of "Introduction to Organic and Biochemistry'; third edition:
Carbohydrates

1. Which molecule contains a chiral carbon?


(a) CH3-CH-CH2CH3 (b) CH3-CH-CH2CH3 (c) CH3-C-CH2CH3
I I II
CH3 OH 0
(d) CH3-CH-CH3
I
OH
2. How many chiral carbons are in CH2-CH-CH-CH-CH-CHO
I I I I I
OH OH OH OH OH
(a) none (b) 4 (c) 5 (d) 6

3 . A dextrorotatory substance
(a) rotates unpolarized light (b) is given the symbol (-)
(c) rotates the plane of polarized light clockwise
(d) rotates the plane of polarized light counterclockwise

62

© Saunders CoUege Publishing


4. A mixture of equal amounts of a pair of enantiomers is called
(a) an anomer (b) a diastereomer (c) a racemic mixture (d) a chiral mixture

5. In the Fischer projection the two bonds represented by horizontal lines


(a) come out at you from the plane of the paper
(b) go behind the plane of the paper (c) are in the plane of the paper
(d) are sometimes in the plane and sometimes not

6. All properties of a pair of enantiomers are identical excep t


1 . they have different specific heats
2. they rotate the plane of polarized light by different amounts
3 . they rotate the plane of polarized light in different directions
4. they react at different rates with chiral compounds
5 . they react at different rates with all compounds, chiral and nonchiral
(a) 1 and 4 (b) 2 and 4 (c) 3 and 4 (d) 2 and 5

7. For which of these formulas are there four stereoisomers?


(a) CH3CH2-C-H (b) CH3 -CH-CH2-CH-CH3
II I I
o OH Br
(c) CH3-CH-CH.,-C-H (d) CH3-CH-CH-CH2-CH-Br
I � II I I I
Br 0 OH OH CI
8. How many stereoisomers are possible for CH3-CH-CH-CH2-CH-CH3
I I I
B r OH CH3
(a) 2 (b) 4 (c) 8 (c) 1 6

9. Which carbon atom in a-D-glucose is the hemiacetal carbon?


(a) C- l (b) C-4 (c) C-5 (d) C-6

1 0. D-glucose and D-galactose are


(a) enantiomers (b) anomers (c) a racemic mixture (d) diastereomers

1 1 . How many aldopentoses can exist (open-chain forms only)?


(a) 2 (b) 4 (c) 8 (d) 1 6

12. The configuration at whi ch carbon atom determines if a monosaccharide is D or L?


(a) the lowest-numbered chiral carbon (b) the highest-numbered chiral carbon
(c) the chiral carbon closest to the aldehyde or keto group

63
(d) the carbon of the primaIY alcohol group

C Sallooers College Pll bli'lhing


1 3 . a-D-glucose when dissolved in water at equilibrium has
(a) only pyranose ring structures (b) only furanose ring structures
(c) both pyranose and furanose rings (d) pyranose rings and open chains

1 4 . Which monosaccharide is both a pentose and an aldose?


(a) fructose (b) glucose (c) galactose (d) ribose

1 5 . Which monosaccharide is the sweetest?


(a) fructose (b) glucose (c) mannose (d) ribose

16. When D-galactose is reduced the product is called


(a) galactitol (b) sorbitol (c) galacturonic acid (d) galactosamine

1 7 . The compound CHPH IS

Q
0
0H

OH
HO
NH
I
c=o
I
CH3

(a) a uronic acid (b) a fn. ctose derivative (c) a sugar phosphate
(d) an amino sugar derivative

1 8 . The disaccharide sucrose is compo sed of


(a) D-glucose and D-galactose (b) D-glucose and D-fructose
(c) D-galactose and D-fructose (d) D-mannose and D-glucose

19. An example of a nonreducing sugar is


(a) lactose (b) glucose (c) fructose (d) sucrose

20. Amylose often exists as a helix in solution. The molecular feature that allows this
molecule to curl up is which glycosidic linkage?
(a) the a( 1 � 6) (b) the P( 1 �4) (c ) the a( 1 �4) (d) the a( 1 �3)

21. Which polysaccharide has only P( 1 �4) glycosidic linkages?


(a) cellulose (b) glycogen (c) amylose (d) amylopectin

22. Cellulose is insoluble in water because


(a) it does not have enough polar functional groups
(b) all its alcohol groups are strongly hydrogen bonded to each other in a
crystalline matrix
(c) all its alcohol groups are on the inside of a helix
(d) all its potential hydrogen bonding groups are on the outside of a helix

64

€;I Saunders CoUege Publi'ihing


23. Where would hyaluronic acid be found in greatest abundance?
(a) blood (b) brain (c ) urine (d) vitreous of the eye

24. Heparin
(a) helps to coagulate blood (b) is an acidic polysaccharide
(c) contains nitrate ester groups (d) has a disaccharide repeating unit

Box 1 6A
25 . In the three dimensional structure of P-D-glucose the hydrogens are all in
(a) axial positions (b) equatorial positions
(c) axial positions except at C- I (d) equatorial positions except at C- I

Box 1 6 B
2 6. In the ABO blood group system a-D-galactose is the only antigen present in which
blood group?
(a) A (b) B (c) 0 (d) AB

27. Which statement is true?


(a) people with AB blood are universal donors
(b) people with 0 blood are universal donors
(c) people with B blood can only accept blood from other people with B blood
(d) people with 0 blood are universal acceptors
Box 1 6C
28. The deficiency of which enzyme causes severe galatosemia?
(a) galactose- I -phosphate uridinyl transferase (b) galactokinase
(c) galactose oxidase (d) galactose isomerase

Box 1 60
29. Drugs often contain inactive ingredients, called fillers. These influence
(a) the molecular weight of the active ingredient
(b) the solubility of the active ingredient in stomach acid
(c) the potency of the drug (d) the rate of delivery of the drug
Box 1 6E
30 . One of the steps that takes place in the eyes as a person ages, and th at may lead to
detached retina and blindness is
(a) the collagen and hyaluronic acid molecules combine
(b) the amount of collagen greatly decreases
(c) the hyaluronic acid forms liquid pockets in the otherwise collagen-rich gel
(d) the vitreous becomes too elastic

Box 1 6 F
3 1. Most chiral compounds that are sold as drugs are
( a) (+) enantiomers (b) ( ) enantiomers (c) racemic mixtures (d) optically active
-

65

C Saunders CoUege Publishing


3 2 . The teratogenic drug thalidomide, after being banned in the U. S . , has made a
comeback
( a) as a (+) enantiomer (b) as a (-) enantiomer
(c) as racemic mixture for non-pregnant women
(d) as anti-inflammatory drug for leprosy patients

Chapter 1 7 of "Introduction to General, Organic, and Biochemistry'; fifth edition;


Chapter 10 of "Introduction to Organic and Biochemistry'; third edition:
Lipids

1. The water-repellent property of lipids i s expressed by the word


(a) hydrophilic (b) hydrophobic (c) hydraulic (d) hydroponic

2. Which solvent dissolves lipids?


(a) water (b) sea water (c) diethyl ether (d) formic acid

3 . Which is a saturated fatty acid?


(a) oleic acid (b) linoleic acid (c) palmitic acid (d) arachidonic acid

4 . In unsaturated fatty acids the double bonds are


( a) all cis (b) all trans (c) both cis and trans
(d) cis-trans isomerism does not apply

5. Which C I S fatty acid i s a f,olid at room temperature?


(a) oleic acid (b) linoleic acid (c) linolenic acid (d) stearic acid

6 . Which fatty acid composition will yield a triglyceride that will most likely b e an
oil?
(a) 3 palmitic acid units (b) 2 palmitic and 1 oleic acid units
(c) 2 linoleic and 1 stearic acid units (d) 3 stearic acid units

7. Heating a fat with water and which of these reagents will make a soap?
(a) NaOH (b) HCI (c) HN03 (d) CaCl2

8. Treatment of oleic acid with hydrogen and a platinum catalyst yields


( a) lauric acid (b) palmitic acid (c) arachidonic acid (d) stearic acid

9. Which component o f glycerophospholipids may be replaced b y serine?


(a) a fatty acid (b) chol ine (c) glycerol (d) phosphate

10. When the alcohol sphingosine is linked to a fatty acid by an amide linkage, the
combination is called
(a) cerami de (b) cerebroside (c) ganglioside Cd) myelin

66

© Saunders College Publishing


1 1 . A certain cephalin is made of glycerol, a fatty acid, phosphate and ethanolamine.
Which components are connected by a carboxylic ester linkage?
(a) the fatty acid and phosphate (b) glycerol and ethanolamine
(c) glycerol and the fatty acid (d) phosphate and ethanolamine

1 2 . Sphingomyelin is made of
(a) sphingosine, a fatty acid, and phosphoryl choline
(b) sphingosine, a fatty acid, and a trisaccharide
(c) sphingosine, a fatty acid, and glucose (d) glycerol and a fatty acid

1 3 . All steroids contain at least how many rings?


( a) 1 (b) 2 ( c) 3 ( d) 4

14. The hydroxy group of cholesterol is on ring


(a) A (b) B (c) C (d) D

1 5 . Lipoproteins transport cholesterol from the liver to the peripheral tissues. In the
spherical lipoprotein complex
(a) unesterified cholesterol is on the surface of the sphere
(b) esterified cholesterol is on the surface of the sphere
(c) phospholipids are not present
(d) linoleic aci d is not an important constituent

1 6. Coated pits contain receptors for


( a) sphingomyelin (b) bil e acids (c) low density lipoproteins
(d) all of these

17. Which steroid in the body increases the concentration of glucose?


(a) testosterone (b) aldosterone (c) cortisol (d) estradiol

1 8 . RU4 8 6 is a drug used to


(a) decrease prostaglandin levels in blood
(b) decrease cholesterol levels in blood
(c) decrease sugar levels ; 11 blood
(d) prevent implantation of a fertilized ovum

1 9. The difference in structure between testosterone and estradiol is


(a) testosterone has two more OH groups than estradiol
(b) testo sterone has a CH3 and an H that is lacking in estradiol
(c) testosterone has an aromatic ring; estradiol does not
(d) there are no differences ; both have the same structure

20. Bile salts are


(a) sex hormone s (b) emulsifying agents
(c) membrane components (d) antiinflammatory agents

67

o Saunders College Publishing


2 1 . In plasma membranes the fluidity is
(a) enhanced by the presence of unsaturated fatty acids
(b) decreased by the presence of unsaturated fatty acids
(c) enhanced by high concentrations of cholesterol
(d) neither unsaturated fatty acids nor cholesterol are present in membranes

22. The common precursor of prostaglandins and leukotrienes is


(a) cholesterol (b) oleic acid (c) arachidonic acid (d) progesterone

23 . Which mode of action of a drug can prevent asthma attack?


(a) blocking the ring closure of arachidonic acid
(b) inhibiting the leukotriene uptake by its receptors
(c) enhancing the oxidation of arachidonic acid
(d) inhibiting prostaglandin synthesis

Box 1 7A
24. Which statement about olestra is false?
(a) It is a carboxylic ester (b) It cannot be digested
(c) The alcohol component is sucrose (d) It is a glyceride

Box 1 7 B
25. What oxidation products of unsaturated fats give rise to rancidity?
(a) aldehydes (b) carboxylic acids (c) carboxylic esters (d) ethers

B ox 1 7 C
26. The basic difference between soaps and detergents is in the
(a) anionic group (b) cationic group (c) the length of the hydrophobic tail
(d) unsaturation of the fatty acids

Box 1 7 0
27. What is the structural difference between fats and waxes?
(a) waxes are monoglycerides; fats are triglycerides
(b) waxes contain only unsaturated fatty acids
(c) waxes have long chain alcohols; fats have glycerol
(d) waxes contain only short chain fatty acids ; fats have long chain fatty acids

Box 1 7 E
28. Multiple sclerosis is a disease caused by the
(a) demyelination of nerve cells (b) deposits of cholesterol in the brain
(c) depletion of phospho glycerides in the brain
(d) improper accumulation of glycolipids in the brain

Box 1 7 F
29. In Tay-Sachs disease the missing enzyme is
(a) a-galactosidase (b) a ganglioside (c) sphingomyelinase (d) hexosaminidase A

68

© Saunders College Publishing


Box 1 7 G
3 0 . Lovastatin reduces the chance o f myocardial infarction by
(a) stimulating LDL receptor synthesis and blocking cholesterol synthe s i s
(b) allowing the enzyme HMG-CoA reductase to function
(c) by blocking LDL receptor synthe sis
(d) by increasing the concentration of cholesterol

Box 1 7 H
3 1 . Which is not an effect o f anabolic steroids?
(a) enhancement of muscular development
(b) excessive growth of facial hair (c) changing the voice to a higher pitch
(d) increase of acne

Box 1 7 1
3 2 . Oral contraceptive pills contain
(a) cholesterol analogs (b) progesterone analogs
(c) natural testosterone (d) bile salts
Box 17 J
3 3 . Aspirin reduces fever and inflammation by
(a) p re ve nti n g the release of arachidonic acid from th e membranes
(b) pre ve nt i n g the ri n g closure of arachidonic acid, and thus the formation of
prostaglandins
(c) preventing the formation of leukotrienes
(d) al l of these

Chapter 18 of "In troduction to General, Organic, and Biochemistry'; fifth edition;


Chapter 1 1 of "Introduction to Organic and Biochemistry'; third edition:
Proteins

1. The function of fibrinogen, a protein in the body, is


(a) transport (b) movemeut (c) structural (d) protection

2. The protein that stores iron in the liver is


(a) cytochrome c (b) alb umin (c) hemoglobin (d) ferritin

3. This amino acid ( C H 3 ) C H CH C OO H is called




- -

I
NH 2
(a) serine (b) alanine (c) valine (d) methionine

4. All amino acids found in proteins are L-amino acids excep t


(a) gl ut am i c acid (b) glutamine (c) serine (d) glycine

69

o Saunders CoUege Publishing


5. Which o f these is a basic amino acid?
(a) glycine (b) lysine (c) cysteine (d) valine

6. The amino acids that contain sulfur are


(a) cysteine and methionine (b) tIyptophan and tyrosine
(c) methionine and proline (c) tyrosine and cysteine

7. The pH at which the positive and negative charges of an amino acid balance each
other is called the
(a) isotonic point (b) isobaric point (c) isosbestic point (d) isoelectric point

8. The amino acid cysteine is special because it


(a) can be easily reduced (b) contains no nitrogen
(c) easily forms a dimer (d) contains a ring

9. At what pH will the lysine structure


. be NH 2 -(CH 2 ) 4 -CH-COO-
I
NH2
(a) 2 (b) 4 (c) 6 (d) 12
10. 0 0
" II
NH2 -CH-C-NH-CH-C-NH-CH-COOH
, I I
CH3 CH2 SH CH2 COOH
This tripeptide is designated
(a) Ala-Cys-Asp (b) Ala-Cys-Glu (c) Asp-Cys-Ala (d) Asp-Val-Ala

1 1 . The number of possible pentapeptides that can be made using all 20 amino acids i s
2
(a) 5 0 (b) 205 (c) 5 20 (d) infinite
x

12. The sequence of the amino acids in a protein is which kind of structure?
(a) primary (b) secondary (c) tertiary (d) quaternary

1 3 . Where in a protein structure are acidic groups located?


(a) only at the N-terminal end (b) only at the C-terminal end
(c) only on the Asp and Glu side chains (d) both b and c

14. SecondalY structures of proteins are held together by hydrogen bonding


between the
(a) C=O and N-H of a backbone
(b) C=O of a backbone and the N -H of a side chain
(c) two N-H groups of a backbone
(d) C=O of a side chain and the N-H of a backbone

70

© Saunders CoUege Publishing


1 5 . Which amino acids enable silk to form mainly pleated sheets?
(a) proline and hydroxyproline (b) alanine and glycine
(c) valine and phenylalanine (d) serine and threonine

1 6 . Which does not contribute to the tertiary structures of proteins?


(a) disulfide bridges (b) salt bridges (c) covalent cross linking
(d) intramolecular hydrogen bonding between backbones

1 7 . Which of these statements about the structure of hemoglobin is mIse?


(a) it contains a prosthetic group (b) it has a quarter-stagger arrangement
(c) it has four chains, called globins (d) it has a quaternary structure

1 8 . Proteins in living cells that help other proteins fold in the correct way are called
(a) enzymes (b) conjugated proteins (c) hydrophobic proteins (d) chaperone s

1 9 . Which structure is not affected by the denaturation of a protein?


(a) primalY (b) secondary (c) tertialY (d) quaternalY

20. A 6 M urea solution denatures proteins by


(a) oxidizing -SH bonds (b) breaking hydrogen bonds
(c) breaking hydrophobic interactions (d) reducing S S bonds
- - -

2 1 . In glycoproteins the carbohydrate portion is linked to the protein portion by which


kind of bond?
(a) covalent (b) ionic (c) hydrogen (d) hydrophobic

Box 1 8A
22. Which chiral combination of amino acids provides the sweet taste of aspartame?
(a) 0-0 (b) L-L (c) L-O (d) O-L

Box 1 8B
23 . Glutathione i s an unusual peptide because the linkage between glutamic acid and
its neighbor cysteine is through which carboxyl group of glutamic acid?
(a) alpha (b) beta (c) gamma (d) delta

Box 1 8C
24. Which groups react to form glycation products?
(a) alcohol and primary amine (b) aldehyde and tertiary amine
(c) aldehyde and primary amine (d) carboxylic acid and primary amine

71

o Saunders CoUege Publishing


Box 1 80
25 . Which of these is false? Hypoglycemic awareness
(a) is a symptom that the insulin level is high compared to the blood sugar
level
(b) can be prevented by using human insulin prepared by recombinant DNA
techniques
(c) is detectable by the symptoms of hunger, sweating and poor coordination
(d) can be reversed by eating sugar
Box 1 8E
26. Which statement about sickle cell anemia is false ?
(a) Homozygotes have no chance of surviving into adulthood.
(b) Red blood cells canying sickle cell hemoglobin behave normally when there is
an ample supply of oxygen.
(c) Sickle cell anemia help to acquire immunity against malaria.
(d) At low oxygen pressures the sickled cells can block capillaries.
Box 1 8F
27. Marfan' s syndrome is characterized by the insufficient cross-linking of
(a) collagen (b) insulin (c) hemoglobin (d) trypsin

Box 1 8G
28. In a number of tissues the quarter-stagger structure of collagen also contains
mineral materi al, mainly calcium hydroxyapatite crystals. Which tissue has the
highest mineral content?
(a) bone (b) cartilage (c) enamel (d) dentine

Box 1 8 H
29. What denaturation process is employed in straightening curly hair?
(a) washing with 70% alcohol (b) heat treatment
(c) the addition of heavy metal ions
(d) the use of a reducing agent followed by an oxidizing agent

3 0 . Photokeratectomy is a form of
(a) oxidation (b) reduction (c) cauterized incision (d) laser surgery

Chapter 19 of HIn traduction to General, Organic, and Biochemistry': fifth edition;


Chapter 12 of HIntroduction to Organic and Biochemistry': third edition:
Enzymes

1 . Most enzymes are proteins or glycoproteins. However, some enzymes,


presumably the oldest ones, are
(a) lipids (b) carbohydrates (c) ribonucleic acids (d) deoxyribonucleic acids

72

© Saunders CoUege Publishing


2. An enzyme that catalyzes the joining of two molecules is called a
( a) lyase (b) ligase (c) isomerase (d) transferase

3. The enzyme carboxypeptidase catalyzes the hydrolysis of a prote in


(a) at the N-terminal end (b) at the C-terminal end
(c) at the carboxyl side of lysine or arginine (d) randomly

4. Vitamin B 2 (riboflavin) is part of a complex enzyme system. Th e vitamin B2 part


is called the
(a) cofactor (b) coenzyme (c) apoenzyme (d) proenzyme

5 . A noncompetitive inhibitor
(a) binds to the active site of an enzyme (b) increases the rate of enzyme action
(c) alters the primary structure of the enzyme (d) none of these

6. The rate of an enzyme-catalyzed reaction is always directly proportional to the


(a) substrate concentrati on (b) square of the substrate concentration
(c) reciprocal of the substrate concentration (d) none of these
7 . If the optimum temperature of an enzyme catalyzed reaction is 3 7 ° C, it is likely
that the enzyme will be irreversibly inhibited at
(a) 90 ° C (b) 1 1 0 ° F (c) - 1 . 0 ° F (d) 3 00 K
8. Food preservation is often achieved by enzyme inhibition. Pickles are preserved
in storage by
(a) increasing the temperature (b) decreasing the temperature
(c) increasing the pH (d) decreasing the pH

9. Which statement is true?


(a) all enzymes are irreversibly inhibited at 40 ° C
(b) all enzymes are denatured at pH 3 . 0
(c) a change in the pH of the environment will hydrolyze an enzyme
(d) enzymes have an optimum pH at which they work best

1 0 . An explanation for the mechanism of competitive inhibition of an enzyme is that


(a) both the substrate and the inhibitor fit into the active site
(b) only the inhibitor fits into the active site
(c) the substrate fits into the active site even though the inhibitor changes the
shape of the enzyme by attachment to another site
(d) only the substrate fits into the active site

1 1 . At very high substrate concentration the maximum rate is achieved when


(a) no inhibitor is present (b) a competitive inhibitor is present
(c) a noncompetitive inhibitor is present (d) either a or b

73

e Saunders CoUege Publishing


1 2 . An enzyme that is activated when a portion of the chain is removed is called
(a) an inhibitor (b) a zymogen (c) an activator (d) none of these

1 3 . Which mode of enzyme c untrol is effected by a regulator?


(a) feedback control (b) activation of a proenzyme
(c) selection of an isoenzyme (d) allosterism

14. Two forms of lactate dehydrogenase, H4 and H2M2, are


(a) isomers (b) isozymes (c) ribozymes (d) zymogens

1 5 . Which enzyme is used to diagnose the severity of a heart attack?


( a) lactate dehydrogenase (b) acid phosphatase
(c) alkaline phosphatase (d) amylase

1 6 . Which enzyme is used to diagnose hepatitis?


( a) amylase (b) lactase (c) alanine aminotransferase (d) acid pho s p h atase

1 7 . Nerve gases operate by forming covalent bonds with the active site of the enzyme
acetylcholinesterase. Therefore
(a) the poisoning can be prevented by cleaving the acetylcholinesterase
(b) nerve gases can be removed by adding more acetylcholine t o the system
(c) nerve gases cannot be removed by adding more acetylcholine to the system
(d) this is an example of noncompetitive inhibition

Box 1 9A
1 8 . Succinylcholine interacts with acetylcholine receptors in the muscle end plate
because
(a) it is a carboxylic ester similar to acetylcholine
(b) the alcohol portion of the esters of the two compounds are the same
(c) the succinyl end of the molecule cannot fit into the receptor
(d) the receptor is hydrophobic and the succinyl group is m ore hydrophobic than
the acetyl group

Box 1 9 B
1 9 . Which protease enzyme is used in meat tenderizers?
(a) papain (b) pepsin (c) trypsin (d) chymotrypsin

Box 1 9C
20. The active site of pyruvate kinase contains two cofactors, K+ and Mg2+, which
help to
(a) transfer carboxyl groups from phosphoenolpyruvate
(b) cleave A TP (c) decarboxylate the pyruvate
(d) anchor the negatively charged sites of the substrate

74

© Saunders CoUege Publishing


Box 1 9 D
2 1 . Sulfa drugs kill microorganisms because they inhibit folic acid synthesis
(a) by competitive inhibition (b) by noncompetitive inhibition
(c) by denaturing the enzyme
(d) by interacting with the natural substrate, para-aminobenzoic acid

B ox 1 9 E
22. Certain metals are poisonous because they inhibit enzymes by oxidizing their
cysteine residues . One such metal is
(a) mercury (b) calcium (c) magnesium (d) iron

Box 1 9 F
23 . What is the mode of the antibacterial action of penicillin?
(a) it inhibits the transpeptidase that makes bacterial cell walls rigid
(b) it kills all bacteria that do not need a cell wall
(c) it prevents folic acid synthesis by bacteria
(d) it inhibits protein synthesis by bacteria

Chapter 20 of "IntroductiolJ to General, Organic, and Biochemistry '; fifth edition;


Chapter 13 of "IntroductiolJ to Organic and Biochemistry'; third edition:
Bioenergetics. Ho w the Body Con verts Food to Energy

1. The common catabolic pathway consists of


(a) the citric acid cycle and glucose metabolism
(b) oxidative phosphorylation and the chemiosmotic pump
(c) electron and H+ transport
(d) the citric acid cycle and oxidative phosphorylation

2. The outer membranes o f mitochondria are


(a) totally impermeable (b) permeable to small molecules and ions
(c) called cristae (d) permeable to ions but not to small molecules

3. The enzymes of the oxidative phosphorylation system are located in the


(a) cytosol (b) nucleus (c) matrix of the mitochondria
(d) cri stae of the mitochondria

4. Which is the least energetic reaction?


(a) ATP + H 2 0 � ADP + Pi (b) A T P + H20 � AMP + PPj
(c) ADP + H 2 0 � AMP + Pi (d) AMP + H20 � adenosine + Pj

5. In the molecule ATP, the A stands for


(a) alanine (b) adenine (c) adenosine (d) acetic acid

75

e Saunders College Publishing


6. The N in NADH stands for
(a) nicotine (b) nicotinic acid (c) nicotinamide (d) neomycin

7. Pantothenic acid is part of the structure of


(a) CoA (b) NAD+ (c) ATP (d) FAD

8. The active end o f the CoA molecule contains


(a) mercaptoethylamine (b) adenine (c) phosphate (d) pantothenic acid

9. All of these statements are true except


(a) the citric acid cycle is also called the Krebs cycle
(b) when NAD+ is reduced, it is converted to NADH
(c) when ADP is converted to ATP energy is given off
(d) the common catalytic pathway takes place in the mitochondria

1 0. The C 2 fragment that enters the citric acid cycle is


(a) fumarate (b) acetyl CoA (c) oxaloacetate (d) succinate

1 1 . How many CO 2 molecules are produced in one full tum of the citric acid cycle?
(a) 1 (b) 2 (c) 3 (d) it depends on which compound enters the cycle

1 2 . When isocitrate, a C-6 compound, is oxidized to a C-s compound the product is


(a) ex-ketoglutarate (b) succinate (c) fumarate (d) malate

1 3 . Th e number of NADH molecules produced in the citric acid cycle for e ac h C2


fragment is
(a) 1 (b) 2 (c) 3 (d) 4

1 4 . In the citric acid cycle what is converted to what?


(a) glucose to CO 2 and H2 0 (b) acetyl CoA to glucose
(c) pyruvic acid to lactic acid
(d) acetyl CoA to CO 2 , NADH, FADH2 , and GTP

1 5 . One molecule of FADH2 entering the oxidative phosphorylation system produces


how many A TP molecules?
( a) 1 (b) 2 (c) 3 (d) 4

1 6. In th e oxidative phosphOlylation system the electrons are tr ansported


(a) through the membranes of mitochondria into the outside media
(b) from the outside aqueous media through the outside membrane of
mitochondria to the proton translocating ATPase
(c) within the inner membrane of mitochondria along the enzyme system
containing flavoprotein-FeS protein-Q enzyme-cytochromes
(d) through the outer mem brane of mitochondria into the inner membrane space

76

© Saunders CoUege Publishing


1 7 . Cytochrome b is
(a) a proton translocating enzyme (b) located in the cytoplasm
(c) part of the citric acid cycle (d) part of the electron transport system

1 8 . The chemiosmotic theOlY was proposed by


(a) Krebs (b) Pauling (c) Mitchell (d) Watson

19. In the overall reaction of the electron and H+ transport chain:


4 H+ + 4e- + O 2 � 2H2 0
the O 2 comes from
(a) acetyl CoA (b) the air we breathe (c) the breakdown of pyruvate
(d) all of these

20. The proton translocating ATPase complex is located in the


(a) cytosol (b) outer membrane of the mitochondria
(c) inner membrane of the mitochondria (d) matrix

2 1 . How many A TP molecules are produced by each 2H+ that enter the proton
translocating ATPase?
(a) 1 (b) 2 ( c) 3 ( d) 4

22. One fi nal product o f oxidative phosphorylation is water, formed from 02'
electrons, and H+ ions. The O2 for this reaction is locate d in the
(a) mitochondria (b) cytosol (c) nucleus
(d) region just outside of l.he mitochondria

23 . How many ATP molecules are produced in the common catabolic pathway from
each entering C 2 fragment?
(a) 2 (b) 12 (c) 24 (d) 3 6

24. I n muscle contraction


(a) actin hydrolyzes ATP (b) myosin hydrolyzes ATP
(c) actin synthesizes A TP (d) myosin synthesizes ATP

Box 20A
25. Which statement is false?
(a) Nicotinic acid is an oxidation product of nicotine.
(b) Niacin and nicotinic acid refer to the same compound.
(c) Nicotinamide is a member of the vitamin D family.
(d) Nicotinic acid is part of the NAD+ molecule.

26. Nicotinic acid is part of


(a) FAD (b) NADH (c) ATP (d) CoA

77
o Saunders CoUege Publishing
B ox 2 0 B
27. A protonophore i s a c omp ound that facilitate s the transport of H+ ions across
membran e s without the exp enditure of energy. Which of these is a protonophore'J
(a) 2,4- dinitrophenol (b) picric acid (c) NADH (d) AMP

B ox 20C
28. Which statement about P-450 i s corre ct?
(a) It contains a copper ion. (b) It c ontains a zinc ion.
(c) It c an c onvert a c arcinogen to its active form.
(d) It resides in the cri stae of mitochondria.

Chapter 21 of HIl1 troductiol1 to Genera/, Organic, and Biochemistry'; fifth edition;


Chapter 14 of HIl1troductiol1 to Organic and Biochemistry'; third edition:
Specific Catabolic Path ways: Carbohydrate, Lipid and Protein Metabolism

1. The building blocks of proteins are found


(a) in fat depots (b) only inside specialized cells
(c) in amino acid p o o l s (d) in the nuclei of cells

2. The metab o l i c pathw ay i n which gluc o s e i s converted to lactic acid i s called


(a) gluconeogene s i s (b) glyc oly s i s ( c ) trans amination ( d) gly co g eno ly sis

3. The enzymes o f the glycolytic p athway are l o c ated


(a) in the cyto s o l (b) i n tl"l e matri c e s of mitochondria
(c) on the cri stae (d) on the outer membranes of mitochondria

4. In the first step of glyc o l y s i s a n A TP molecule i s u s e d up. I n this activation step


gluco s e is c onverte d to
(a) gluc o s e 6 -pho sphate (b) gluc o s e 2-phosphate
(c) fructo s e 2-pho sphate (d) fructo se

5. I n glycolys i s the hexo se mono sac chari des are first broken down to
(a) three C 2 fragments (b) two C3 fragments
(c) CO 2 and a p ento se sugar (d) all of the se happen simultaneously

6. Which m o l e cule i s l1 0 t p ar t of the glyco lytic p athway?


(a) phosphoenolpyruvate (b) 3 -phosphoglycerate
(c) dihydroxyacetone pho sphate (d) ornithine

7. The e n d product of glyc o l y s i s i s pyruvate . Pyruvate d o e s not accumulate because


(a) it is converted to acetyl coenzyme A
(b) it i s reduced to lactate (c) it i s reduc ed to ethyl alcohol
(d) all the se can happen il l different organisms

78

© Saunders College Publishing


8. The number o f ATP molecules produced from the complete metabolism
(glycolysis + common pathway) of a glucose molecule in the muscles is
(a) 1 2 (b) 24 (c) 3 6 (d) 4 8

9. Ethanol, the end product of glycolysis in yeast, is produced from pyruvate by


(a) oxidative decarboxylation (b) oxidation
(c) reductive decarboxylation (d) pyruvate dehydrogenase

1 0. B efore it can be oxidized, a fatty acid must be activated. This involves


(a) reduction of FAD to FADH2
(b) hydrolysis of A TP to AMP
(c) conversion of NADH to NAD+
(d) conversion of ADP to ATP
1 1 . The activated form in which a fatty acid enters the mitochondria is called
(a) acyl CoA (b) acetyl CoA (c) fatty acid I -phosphate
(d) an ethyl ester of the fatty acid

12. Each turn of the p-oxidation spiral of a fatty acid produces all of the following
(a) one FADH 2 and one NADH + H+ (b) one acetyl CoA
(c) one acetyl CoA, one FADH2 , and one NADH + H+
(d) two NADH and two FADH 2

1 3 . Which of these is a ketone body?


( a) succinate (b) fumarate (c) oxalate (d) acetoacetate
14. The urea cycle is part of the catabolism of
(a) carbohydrates (b) lipids (c) amino acids (d) glycerol

1 5 . The n i tr og e n of glutamic acid i s removed in an oxidative deamination. In this step


the nitrogen is released in the form of
(a) NH3 (b) NH/ (c) urea (d) uric acid

16. The condensation product of CO 2 , H2 0, NH/, and ATP that enters the urea cycle
is called
( a) citrulline (b) carbamoyl phosphate (c) arginine (d) ornithine

1 7. The glucogenic amino acids yield a carbon skeleton that is metabolized to


(a) urea (b) bilirubin (c) heme (d) pyruvate

Box 2 1 A
1 8 . Muscle cramps are caused by the
(a) accumulation of lactic acid (b) accumulation of pyruvic acid
(c) speeding up of the citric acid cycle
(d) speeding up of aerobic glycolysis

79

e Saunders College Publishing


Box 2 1 B
19. In the glycogen storage disease called Cori ' s disease the faulty enzyme is
(a) maltase (b) phosphorylase (c) debranching enzyme
(d) phosphotriose isomeHl se

Box 2 1 C
20. Under some stressful or diabetic condition acidosis may occur. This is most likely
caused by
(a) an excess of insulin (b) ketone bodies (c) an excess of water retention
(d) an increase in K+ concentration in cells

Box 2 1 D
21. MSG, which enhances the flavor of many foods, is the salt form of the amino acid
(a) methionine (b) glycine (c) serine (d) glutamic acid

Box 2 1 E
22. Polyubiquitin links to proteins targeted for destruction by an amide linkage
between
(a) a glycine of ubiquitin and an arginine of the protein
(b) a glycine of ubiquitin and a side chain amino group of lysine of the protein
(c) an alanine of ubiquitin and the N-terminal lysine of the protein
(d) an alanine of the ubiquitin and a serine of the protein

Box 2 1 F
23 . The amino acid that is not metabolized in the disease called PKU is
(a) leucine (b) proline (c) phenylalanine (d) glycine

Box 2 1 G
24. The yellowing of the whites of the eye that is a symptom of jaundice is caused by
the accumulati on of
(a) hemoglobin (b) heme (c) biliverdin (d) bilirubin

Chap ter 22 of "Introduction to General, Organic, and Biochemistry'; fifth edition;


Chap ter 15 of "Introduction ta Organic and Biochemistry': third edition:
BiosYl1thetic Path ways

1. It is biologically advantageous to have different pathways for anabolic and


catabolic processes. One advantage for such diversity is
(a) overcoming the effect of Le Chatelier' s principle
(b) the energy expense in the form of A TP can be dispensed with
( c) there is no need to store amino acids in the amino acid pool
(d) the ease of conversion of electrical energy to ATP

80

e Saunders CoUege Publishing


2 . Photosynthesis is a process that
(a) takes place in animal tissues only
(b) synthesizes fatty acids with the aid of sunlight
( c) uses up oxygen and produces carbon dioxide
(d) produces carbohydrates from CO2 and H20

3 . In gluconeogenesis
(a) glucose is synthesized from pyruvate
(b) glucose is broken down into two C3 fragments
(c) glucose is built into glycogen (d) glucose is converted to galactose

4. In UDP glucose the glucose is linked to the


(a) nitrogen of the uridill(; (b) carbon-2 of the ribose
(c) carbon-3 of the ribose (d) phosphate group

5. In fatty acid synthesis


(a) two C 2 fragments are condensed (b) a C2 and a C4 fragment are condensed
(c) two C3 fragments are condensed with the elimination of CO2 .
(d) a C 2 and a C 3 fragment are condensed with the elimination of CO2

6. The activated C2 fragment that starts fatty acid synthesis is


(a) acetyl CoA (b) malonyl ACP (c) acetoacetyl ACP (d) acetoacetyl CoA

7. Unsaturated fatty acid are


(a) directly synthesized from smaller fragments
(b) synthesized from the corresponding saturated fatty acid
( c) the oxidation products of cholesterol
(d) the reduction products of sphingosine

8 . In synthesis of glycerophospholipids the first compound (or ion) assembled is


(a) a triglyceride (b) activated choline (c) phosphatidate (d) none of these

9. Cholesterol is synthesized in the liver from


(a) UDP glucose (b) sphingosine (c) serine (d) C2 in the form of acetyl CoA

1 0 . Which amino acid is synthesized from a-ketoglutaric acid and ammonium ion?
(a) lysine (b) glutamic acid (c) asparagine (d) isoleucine
Box 22A
1 1 . The structure of chlorophyll is similar to that of
(a) adenine (b) heme (c) C oA (d) urea
Box 2 2 B
1 2 . The component of phospholipids that is present in excess in infants with hyaline
membrane disease is
(a) saturated fatty acid (1'. ) unsaturated fatty acid (c) choline (d) glycerol

81

e Saunders College Publishing


Box 2 2 C
1 3. The process by which tlyptophan is converted to indigo blue is
(a) a decarboxylation (b) an oxidation (c) a reduction (d) a hydrolysis

Chapter 23 of "Introduction to Genera/, Organic, and Biochemistry'; fifth edition;


Chapter 1 6 of ''introduction to Organic and Biochemistry'; third edition:
Nucleic A cids and Protein Synthesis

1. In an animal cell hereditalY information is located in the


(a) cytosol (b) mitochondria (c) nucleus (d) cell membranes

2. The purine bases in DNA are


(a) adenine and thymille (b) adenine and guanine
(c) cytosine and guanine (d) cytosine and thymine
3. Nucleosides are
(a) purine bases (b) pyrimidine bases
(c) bases plus sugar (d) bases plus sugar plus phosphate
4. The bases of the DNA mclecule are
(a) side chain groups (b) found outside of the double helix
(c) located only at the ends of the DNA molecule
(d) the backbone of the DNA molecule
5. The rule that adenine and thymine are present in equimolar quantities i n DNAs
from all sources, and likewise for guanine and cytosine, is associated with the
name of
(a) Chargaff (b) Crick (c) Watson (d) Rosalind Franklin
6. This formula represents the pairing of which bases?

(a) G-C (b) A-C (c) A-T (d) G-T


7. All these statements describe the differences between DNA and RNA except
(a) DNA is double stranded; RNA is single stranded
(b) DNA has thymin�; RNA has uracil (c) DNA has deoxyribose; RNA has ribose
(d) DNA has 1 phosphate for each sugar; RNA has 2 phosphates for each sugar.

82

© Saunders CoUege PubUshing


8. The function o f polymerases is to
(a) align the complementary bases
(b) transport the complementary bases to their respective positions
(c) open up the double helix in the middle
(d) join the nucleotide backbones

9. Okazaki fragments are


( a) regions of G-C paired nucleotides (b) about a thousand nucleotides long
(c) about a million nucleotides long (d) synthesized in the 5 ' � 3 'direction

10. Ribosomes are small spherical bodies


( a) made of RNA and proteins (b) residing in the nucleus of a cell
(c) made of RNA only (el.) made of DNA and RNA

1 1. Which of these statements is false?


(a) tRNA participates in translation (b) rRNA participates in translation
(c) RNA is synthesized outside the nucleus
(d) mRNA participates in transcription

12. The step in which a nucleotide sequence on DNA is copied onto an RNA
molecule is called
(a) replication (b) transcription (c) translation (d) transfiguration

13. A codon is
(a) a triplet of bases on mRNA (b) a triplet of bases on tRNA
(c) a triplet of bases on DNA (d) a triplet of amino acids in the ribosome

14 To which part of the tRNA molecule does the specific amino acid attach?
(a) the anticodon loop (b ) the 3 ' -OH end (c) the 5 ' -OH end
(d) only the portions where the bases are paired by hydrogen bonding

15. In the genetic code there


(a) are three initiation signals (b) is one initiation signal
(c) are six termination signals (d) is one termination signal
1 6 . In transcription the newly synthesized RNA
(a) runs in the 5 ' to 3 ' direction (b) runs in the 3 ' to 5 ' direction
(c) runs in either direction (d) is terminated by the enzyme polymerase

1 7 . The process whereby an amino acid reacts with an ATP molecule to form amino
acid-AMP is called
(a) initiation (b) elongation (c) termination (d) activation
1 8 . When the anticodon of a methionine tRNA binds to mRNA and the mRNA is
attached only to the 40S ribosome, the process is called
(a) initiation (b) elongati'.in (c) termination (d) activation

83

o Saunders College Publishing


1 9 . In the elongation step of protein synthesis, the binding of tRNA takes place on
(a) the DNA molecule (b) the P-site of the 60S ribosome
(c) the 40S ribosome (d) the A-site of the 60S ribosome

20. Histones are


(a) the noncoding sequences of DNA (b) the coding sequences of DNA
(c) the spliced exons in the mature mRNA
(d) the basic proteins in the nucleosomes

2 1 . The coding s e que nc e s of DNA are called


(a) introns (b) exons (c) satellites (d) histones

22. In regulating gene expression in eukaryotic cells the selective binding proteins
that bind to the p romo te r sites are called
(a) mutagens (b) nucl eosomes (c) transcription factors (d) RNA polymerase

23 . Metal binding fingers contain the ion


2 2 2
(a) Fe2+ (b) Zn + (c) Cu + (d) Mn +

24. Which statement is false?


(a) mutations may occur during re plication
(b) all changes in an mRNA codon result in a change in an amino acid in the
protein coded for
(c) mutations may occur during transcription
(d) mutations may be harmful or harmless

25. A pl asm i d consists of


(a) double s tranded DNA (b) double stranded RNA
(c) both DNA and RNA (d) protein molecules

26. Which statement is incorrect ? Restriction endonucleases


(a) cleave single stranded RNA (b) cleave double stranded DNA
(c) cleave pl asmids (d) produce sticky ends

Box 23A
27. The anticancer drug fluorouracil acts by
(a) depriving cancer cel ls of carbohydrate nutrients
(b) making DNA synthesis go too fast
(c) interfering with the manufacture of thymidine
(d) removing fluorine from the body

Box 2 3 B
28. Viral infection can be prevented or cured by
(a) sulfa drug (b) penicillin (c) immunization (d) aspirin

84

© Saunders CoUege Publishing


Box 23C
29. New drugs, such as indinavir, that are used in combating AIDS are
(a) HIV protease inhibitors (b) HIV reverse transcriptase inhibitors
( c) acting against Pneumocystis carinii
(d) weakened HIV virus particles acting as a vaccine
B ox 230
30. Transgenic mice created to test the efficacy of certain drugs against AIDS
became blind from cataracts. Upon administration of a certain drug the mice did
not develop cataracts. These transgenic mice contained
(a) HIV protease linked to an insulin promoter
(b) HIV reverse transcriptase linked to an aA crystallin promoter
(c) HIV reverse transcriptase linked to an insulin promoter
(d) HIV protease linked to an aA-crystallin promoter
Box 23E
3 1. Which statement is false?
(a) the hemoglobin in all species of mammals has an identical sequence of
amino acids
(b) scientists can tell the genetic relationships of different species by looking at
amino acid sequences of their proteins
(c) the proteins of apes have amino acid sequences similar to those of humans
(d) some amino acids in the same proteins of different species appear in the
same positions in the �,equences
Box 23F
32. The Ames test is designed to look for
(a) mutagens (b) carcinogens (c) resistance to radiation (d) bacterial strains
Box 23G
33. A normal gene that i s mutated into a cancer- c aus ing gene by a virus is called
(a) an oncogene (b) a proto-oncogene (c) a sticky finger (d) a mutagen
Box 23H
34. In DNA fingerprinting
(a) whole DNA molecules are subjected to electrophoresis
(b) DNA molecules are hydrolyzed to their nucleotides by Hel
(c) DNA molecules are broken down to fragments by restriction enzymes and
the fragments are subjected to electrophoresis
(d) DNA molecules are transcribed to RNA molecules and the RNA molecules
subjected to electroph'Jresis
Box 2 3 1
35. Which statement is incorrect regarding the human genome
(a) It contains 3 billion base pairs (b) It contains 1 00,000 genes
(c) Each chromosome contains many DNA molecules
(d) It is distributed am ong 22 pai rs of chromosomes plus two sex chromosomes
85
C Saunders CoUege Publishing
Chapter 24 of "Introduction to Genera/, Organic, and Biochemistry': fifth edition;
Chapter 1 7 of "Introduction to Organic and Biochemistry': third edition:
Chemical Communication: Neurotransmitters and Hormones

1. The receptors on the surfaces o f cell membranes are


(a) lipids (b) carbohydrates (c) proteins (d) nucleic acids

2. The hair-like projections coming from the main body of a nerve cell are called
(a) dendrites (b) axons (c) synapses (d) vesicles

3. The messenger that initiates the fusion o f vesicles with the synaptic membrane
after which the neurotransmitters empty into the synapse is
(a) acetylcholine (b) Ca2+ (c) adrenalin (d) GABA

4. When acetylcholine is adsorbed onto a cholinergic receptor of a neuron


(a) the membranes of the neurons are destroyed
(b) the ion gates are opened and the ions freely flow across th e membranes
(c) the axon of the neuron breaks off
(d) the membranes of the neurons become rigid and breakable

5. Which o f these groups is hydrolyzed by an enzyme when ac etyl c ho l ine is


removed from its receptor?
(a) amide (b) acetal (c) anhydride (d) carboxylic ester

6. Curare, the poison of Amazon Indians, can cause death by perm anen tly relaxing
muscles. Curare works thi s way because it is
(a) a noncompetitive inhibitor of the enzyme that synthesizes acetylcholine
(b) a competitive inhibitor of the enzyme that synthesizes acetylcholine
(c) an inhibitor of acetylcholinesterase
(d) a noncompeti ti ve inhibitor of phosphodiesterase

7. Cyclic AMP is synthesized from


(a) ATP (b) ADP (c) AMP (d) guanine

8. One o f the functions o f cAMP is to


(a) speed up signals (b) slow down signals (c) amplify signals
(d) block signals

9. The neurotransmitter histamine is


(a) cholinergic (b) adrenergic (c) peptidergic (d) nicotinergic

10. Monoamine oxidase inhibitors are good antidepressants because they


(a) activate receptor sites (b) block receptor sites
(c) increase the concentration of monoamine neurotransmitters
(d) decrease the concentration of monoamine neurotransmitters

86

e Saunders CoUege Publishing


1 1. Ulcers are treated by
(a) antibiotics that act against Helicobacter pylori (b) 11;. receptor blockers
(c) H.. receptor blockers (d) a combination of (a) and (b)

1 2 . A neurotransmitter that controls the perception of pain in the brain is


(a) GABA (b) enkephalins (c) taurine (d) histamine

1 3 . Insulin' s mode ofaction


(a) affects the synthesis of an enzyme (b) activates one or more enzymes
(c) affects transcription of a gene (d) affects the permeability of membranes

14. The hormone vasopressin is secreted in the


( a) beta cells of pancreatic islets (b) alpha cells of pancreatic islets
(c) posterior pituitary (d) adrenal cortex

Box 24A
15. The drug Cognex seems to improve the memory of patients with Alzheimer's
disease . It i s a potent inhibitor of
(a) acetylcholine transferase (b) p-amyloid protein
(c) acetylcholinesterase (d) release of K+ from vesicles

B ox 24B
16. Nerve gases such as Sarin are
(a) phosphonates (b) phosphates (c) pyrophosphates (d) dihydrogenphosphates

Box 24C
17. The botulinum toxin inhibits
(a) acetylcholinesterase (b) the release of acetylcholine from vesicles
(c) acetylcholine transferase (d) reabsorption of acetylcholine from the synapse

B ox 2 4 D
1 8. Parkinson ' s disease is due to the depletion of dopamine. We cannot administer
dopamine to relieve the symptoms because dopamine
(a) does not cross the blood/brain batTier
(b) is immediately oxidized by MAOs (c) is not soluble in water
(d) is immediately transported out of the cells by a dopamine transporter

Box 24E
19. The compound in red wine that causes hangover headaches is
(a) tyramine (b) monosodium glutamate (c) nitroglycerine (d) phenylethylamine

87

e Saunders College Publishing


Box 24F
20. Which statement regarding nitric oxide is incorrect ?
(a) it can cause degeneration and death of neurons
(b) it is synthesized when arginine is converted to arginosuccinate
(c) it participates in long term potentiation
(d) it fights herpes simplex infection
Box 24 G
21. Insulin-dependent diabetic patients
(a) do not have enough insulin receptors on their cells
(b) may have a defect in the pancreas that manufactures insulin
(c) have lower than normal glucose levels in the blood (d) all of the se

Chapter 25 of Hln troduction to General, Organic, and Biochemistry'; fifth edition;


Chapter 1 8 of Hln troductioll to Organic and Biochemistry'; third edition:
Body Fluids

1. The most abundant extracellular fluid is


(a) cerebrospinal fluid (b) aqueous humor (c) urine (d) interstitial fluid
2. Between which pair of fluids is there only a limited exchange of compounds?
(a) lymph and interstitial fluid (b) blood and interstitial fluid
(c) blood and cerebrospinal fluid (d) blood and lymph
3 . The most numerous cellular elements of blood are
(a) erythrocytes (b) thi·ombocytes (c) phagocytes (d) leukocytes

4. The function of platelets i s to


(a) help stop bleeding (b) cany oxygen to the tissues
(c) transport CO 2 from the tissues to the lung (d) maintain body pH
5 . The most abundant plasma protein is
(a) globulin (b) albumin (c) clotting factor (d) fibrinogen
6. How many heme units are in each hemoglobin molecule?
( a ) 1 (b) 2 (c) 4 (d) 6
2
7. When the first oxygen molecule binds to the Fe + of hemoglobin it changes the
shape of the four globin units, allowing the second oxyg�n easier access. This is
known as the
(a) Bohr effect (b) allosteric effect (c) Bowman effect (d) high altitude effect
8. Most CO 2 carried by the blood from the tissues t o the lungs is i n the form of
(a) carbonic acid (b) carbon dioxide gas dissolved in the plasma
(c) CO 2 bound to the N-terminal end of the globins
(d) CO 2 bound to the C-terminal end of the globins

88

© Saunders CoUege Publishing


9 . When the blood reaches which of these pH values would a patient b e said to have
the condition called alkalosis?
( a) 6 . 4 (b) 7 . 2 (c) 7 . 42 ( d) 7 . 8
1 0 . In which of these parts of the kidney is most of the water reabsorbed from the
urine?
(a) Bowman ' s capsule (b) Henle ' s loop (c) distal tubule (d) proximal tubule
1 1 . The daily urine output for an average person is
(a) one gallon (b) 3 L (c) 1 5 00 mL (d) 5 L
1 2 . Which nitrogen-containing compound does not exist in normal urine?
(a) ammonia (b) trinitrotoluene (c) creatine (d) urea

1 3 . An agent that increases the volume of urine is


(a) called a diuretic (b) called an antidiuretic (c) vasopressin (d) aldosterone
14. Baroreceptors that detect decreases in blood pressure are located in the
(a) neck (b) heart (c) kianeys (d) liver
1 5 . Aldosterone increases
(a) reabsorption of Na+ from the urine (b) water reabsorption in the kidneys
( c) blood pressure (d) all of these
1 6 . Which class of immunoglobulins are in secretions of tears, milk, or mucus?
(a) IgA (b) IgM (c) IgE (d) IgG
1 7 . The binding sites on immunoglobulins are located
(a) on the constant regions of the heavy chains
(b) on the constant regions of the light chains
(c) only on the vari able regions of the light chains
(d) on the variable regions of both the light and heavy chains
1 8 . Immunoglobulins bind to antigens by
(a) covalent bonds (b) ionic bonds (c) London dispersion forces
(d) none of these
19. Which is not a feature of the immunoglobulin superfamily?
(a) they are glycoproteins (b) they interact with antigens
(c) they are all Y shaped (d) they all participate in chemical communications
Box 25A
20. The difference in action between Dramamine and fexofenadine is
(a) They act on different histamine receptors.
(b) Dramamine is an agonist, fexofenadine an antagonist.
(c) Dramamine relieves seasickness, fexofenadine motion sickness.
(d) Dramamine penetrates the blood-brain barrier, fexofenadine d oes not.

89

C Saunders College Publishing


Box 25B
2 1 . What is the role of vitamin K in blood clotting?
(a) it is needed to produce prothrombin
(b) it activates fibrinogen to form fibrin (c) it activates the platelets
(d) it releases Ca2 + ions
B ox 25C
22. Which body fluid has the highest K+ ion concentration?
(a) intracellular fluid (b) blood plasma (c) blood serum (d) interstitial fluid
B ox 25D
23 . Osteoporosis in women past the menopause may be caused by lack of
(a) sodium ions (b) potassium ions (c) estrogens (d) chloride ions
Box 25E
24. Calcium channel blocker medications reduce blood pressure by
(a) relaxing the vascular smooth muscles
(b) decreasing the frequency and the pumping ability of the heart muscles
(c) increasing the volume of urine (d) decreasing the volume of urine

Box 25F
25. An example of a disease that has been completely eradicated by immunization is
(a) typhus (b) cholera (c) smallpox (d) all of these

Chapter 26 of "Introduction to Genera/, Organic, and Biochemistry'; fifth edition;


Chapter 19 of "Introduction to Orgal1J'c and Biochemistry'; third edition:
Nutritioll and Digestioll

1. Which of these is llOt considered to be a nutrient?


(a) water (b) vitamin C (c) tobacco (d) iron

2 . According to the U . S . Agriculture Department food pyramid, which o f these


foods are considered not necessalY for a healthy diet?
(a) starches (b) fats (c) fruits (d) meat, fish, eggs, and/or dairy pro du cts

3 . The basal caloric requirement for a young adult male is


(a) 1 800 Cal/day (b) 2500 Cal/day (c) 2900 Callday (d) 3 500 Cal/day

4. Which nutrient provides the essential building blocks for the body ' s own
proteins?
(a) carbohydrates (b) fats (c) minerals (d) plant proteins

5 . A disease that i s the result of diets deficient in protein IS


(a) kwashiorkor (b) marasmus (c) rickets (d) scurvy

90

© Saunders CoUege Publi�hing


6. Folic acid is
(a) an amino acid (b) a water-soluble vitamin (c) a fat-soluble vitamin
(d) a fatty acid

7. Which food ingredient provides dietary fiber?


(a) starch (b) gelatin (c) cellulose (d) ascorbic acid

8. Which vitamin, when deficient, causes night blindness?


(a) A (b) C (c) D (d) E

9. Which fat soluble vitamin acts as an antioxidant?


(a) C (b) B2 (c) K (d) E

1 0. Which vitamin B contains cobalt?


(a) pantothenic acid (b) B 1 2 (c) thiamine (d) biotin

1 1. Which mineral is needed for bone formation?


2 2 2
(a) Na+ (b) Fe + (c) Ca + (d) Mn +

12. Which enzyme helps to digest storage polysaccharides such as glycogen?


(a) trypsin (b) debranching enzyme (c) pepsin (d) lipase

13. Carboxypeptidase cuts


(a) protein molecules randomly
(b) protein molecules on the C =O side of arginine
(c) amino acids one at a time from the N-terminal end of a protein
(d) amino acids one at a time from the C-terminal end of a protein

Box 26A
1 4 . Parenteral nutrition is
(a) intravenous feeding (b) a diet void of milk products
(c) breast feeding of babies (d) none of these

Box 26B
1 5 . Which statement is false?
(a) large amounts of folic acid taken during pregnancy can prevent certain birth
defects
(b) there is no such thing as too much vitamin intake
(c) doses of vitamins high (� r than the RDA are usually harmless
(d) excess folic acid can be harmful

Box 26C
16. Hyperglycemia is present when the glucose content of the blood, in mglmL, is
(a) 50 (b) 85 (c) 1 00 (d) 1 5 0

91

C Saunders CoUege Publi"hing


Chapter 1 of "Introduction to Organic and Biochemistry': third edition:
Organic Chemistry and Chemical Bonding

1. The synthesis of which organic compound first disproved the doctrine of "vital
force"?
(a) uric acid (b) urea (c) C O 2 (d) vitamin A

2. In order to achieve a complete outer shell how many electrons must an atom of
strontium gain or lose?
(a) lose 2 (b) lose 3 (c) gain 1 (d) gain 2

3. If an atom X can accept two electrons from another atom in the process of
forming a stable ion, then the atom X
(a) has two protons in its nucleus (b) is a metal
(c) belongs to group IIA of the periodic table
(d) is capable of forming a molecule of the formula H2 X

4. When two fluorine atoms overlap to form a covalent bond, which atomic orbitals
overlap?
(a) two 3 d orbitals (b) two 2 s orbitals (c) a 2 s orbital and a 2p orbital
(d) two 2p orbitals

5 . A carbon atom forms how many bonds?


(a) 2 (b) 3 (c) 4 (d) 5

6. Which of the following formulas is incorrect because it contains an atom that


does not have its normal number of covalent bonds?
H
I
(a) H-C=N (b) H-C = O (c) O=C=O (d) F-C-F
I I I
H H H

7. The Lewis structure of a carbon atom is


.. .

(a) C (b) : C : (c) : C (d) ·C·


•• •

8. The Lewis structure o f the CO molecule i s


•• •• • •

(a) :C=O: (b) :C=O: (c) :C-O: (d) : C-O:

9. The Lewis structure of the nitrite ion (N0 2-) is


.. .. .. .. .. . . . . .. .. . . .-

( a ) [ : Q -N=O: ] (b) [:9.-N - �: ] (c) [ : O =N=O: ] (d) [ : O=N = O:]

92

© Saunders CoUege Publishing


10. The Lewis structure of NO shows a total of 1 1 outer-shell electrons This
molecule is an example of
(a) a free radical (b) a molecule that obeys the octet rule
(c) a molecule that contains only paired electrons
(d) a molecule that contains a triple bond

1 1. Which of these has a coordinate covalent bond?


(a) H-CI: (b)
- H-N-H (c) : CI-CI: (d) [ H-O-Hr
I I
'H H

12. Which of these molecules has a linear shape?


(a) HCN (b) H 2 0 (c) NH3 (d) CH4

1 3 . The shape of the molecule BCl3 is


(a) linear (b) planar triangular (c) tetrahedral (d) none of these

14. The number of degrees in the angle between the two bonds CI-C-CI in the
molecule CCl 4 is
(a) 1 80 (b) 1 20 (c) 90 (d) 1 09 . 5

15. Besides carbon, which element i s the most frequent constituent o f organic
compounds?
(a) hydrogen (b) phosphorus (c) nitrogen (d) sulfur

1 6 . Which statement is false? An sl' orbital


(a) can contain two electrons (b) has two lobes of equal size
(c) can overlap with an s orbital (d) is a hybrid orbital

1 7. Which element normally forms 3 bonds?


(a) oxygen (b) bromine (c) nitrogen (d) carbon

18. Which statement is false?


(a) the properties of a compound depend solely on its molecular formula
(b) the properties of a compound depend on its structural formula
(c) ethyl alcohol made in a chemical laboratory is exactly the same as ethyl
alcohol obtained by fermentation and distillation
(d) when a compound is manufactured by bacteria the process is called
biosynthesis

Box 1 A
19. Lithium carbonate is used in medicine as
(a) an antidepressive (b) an antacid (c) an expectorant (d) a diuretic agent

93

o Saunders CoUege Publishing


Box 1 B
20. When it picks up oxygen in the lungs, the iron in the center of a heme molecule
binds to the oxygen by which type of bond?
(a) ionic (b) polar covalent (c) nonpolar covalent (d) coordinate covalent

Box 1 C
21. A synthesis in which one obtains as many compounds from as few building
blocks as possible is called
(a) combinatorial (b) rational (c) shotgun (d) unique

Chapter 2 of "Il1troductiol1 to Orgal1ic al1d Biocbemistry': tbird editiol1:


Alkal1es

1. Hydrocarbons that contain at least one double bond are called


(a) aromatic (b) alkanes (c) alkenes (d) alkynes

2. Butane has how many carbons?


(a) 2 (b) 3 ( c) 4 (d) 5

3. Th e compound CH3(CH2)5 CH3 i s called


(a) l1-heptane (b) isohexane (c) l1-hexane (d) l1-octane

4. The starred carbon in this structure CH3 - * CH-CH3 is


I
CH3
(a) primalY (b) secondary (c) tertiary (d) quaternary
5. Among these compounds, which are isomers?
1 . CH3 CH2-O-CH3 2 . CH3 CH2-O-CH2CH3
3 . CH3 CH2CH20H 4 . CH2=CH-O-CH3
(a) 1 and 3 (b) 2 and 3 (c) 1 and 2 (d) 1 and 4

6. CH3 CH3 CH3


I I I
CH2-CH2-CH-CH2CH3 and CH3 CH2-CH-CH2CH2CH3 are
(a) structural isomers (b) identical molecules
(c) different molecules that are not isomers (d) none of these

7. The IUPAC name for 9


CH3CH2CH2- H-CH2- H "": CH3 9 IS

CH3 CH2
I
CH3
(a) 6-ethyl-4-methylheptane (b) 4 , 6 - dime thyl o ct an e
(c) 2-ethyl-4-methylheptal1e (d) 3 , 5 -dimethyloctane

94

© Saunders CoUege Publishing


8. The name o f the group CH3CH 2 -CH- is
I
CH3

(a) isobutyl (b) sec-butyl (c) tert-butyl (d) isopropyl

9. 1 ,2-Dibromo-2-methylbutane has the structure


(a) CH3 -CH-CH 2 -Br
I
CH 2C H7-Br

C H 2 CH3
I
(c) CH 2 - C -Br (d) Br-CH2 -CH-CH-CH3
I I I I
Br CH3 Br CH3

1 0 . In naming alkyl halides containing four different halogens which prefix is cited
first?
(a) fluoro (b) chloro (c) bromo (d) iodo

1 1. CI is called

Q Br
(a) l -chloro-3 -bromocyclopentane
(b) I -bromo-2-chlorocyclohexane
(c) I -bromo-3 -chlorocyclopentane
(d) I -bromo-2-chlorocyclopentane

12.

Q � and Me

HF HH
(a) structural isomers (b) stereoisomers
(c) different molecules that are not isomers (d) identical molecules

1 3 . How many stereoisomers of 1 ,4-dibromocyclohexane are there?


( a) 1 (b) 2 ( c) 4 ( d) 6

1 4 . In which of these solvents are alkanes least soluble?


(a) benzene (b) chloroform (c) water (d) dimethyl ether

95

Saunders College Publi�hing


15. The number of structural isomers with the formula C3H6 CIBr is
(a) 4 (b) 5 ( c) 6 ( d) 7

16. The combustion of one mole of cyclohexane, C 6 H 1 2 , produces how many moles of
CO2 ?
(a) 3 (b) 6 (c) 1 2 (d) 8

1 7. Chlorine reacts with alkanes in the presence of ultraviolet light. This type of
reaction is called
(a) combustion (b) substitution (c) addition (d) condensation

18. Which of these is not a product of the reaction of propane with c hlorine in the
,

presence of ultraviolet light?


(a) CI-CH 2 -CH 2 -CH 2 -CI (b)

19. Compounds with the formula R-O-R are


(a) ketones (b) ethers (c) aldehydes (d) esters

20. In 2-butanol the CH3CH2 CHCH3 part of the molecule


(a) is the functional group (b) is the alcohol group
(c) is the alkyl group (d) contains only primary carbons

2 l . What functional groups are present in CH3-CH=CH-CH 2 -OH?


(a) alcohol and alkyne (b) ketone and alkene
(c) aldehyde and ketone (d) alcohol and alkene

22. Which of these is a ketone?


(a) CH 3 CH 2 -CH-CH3 (b) CH3-C-CH2 CH3
I II
OH 0
(d) CH3 -O-CH 2 CH3
Box 2A
23 . The most stable shape of the cyclohexane ring resembles a
(a) chair (b) table (c) boat (d) flat plane
Box 2 B
24. If you put a few drops o f hexane o n your finger and then wipe i t off, you will feel
the skin dry up. This happens because the hexane
( a) is poisonous (b) protects the skin (c) dissolves the natural skin oils
(d) extracts the water from the skin

96

© Saunders College Publishing


Box 2C
25 . Which hydrocarbons are responsible for the "knocking" in automobile engines?
(c) long chain (above 10 carbons) (d) short chain (below 5 carbons)
(a) straight chain (normal) (b) highly branched

Box 2 0
26. Carbon dioxide i s
(a) produced by incomplete combustion o f fuels (b) more poisonous th an CO
(c) insoluble in water (d) a minor component of our atmosphere

Box 2 E
2 7 . Which statement is incorrect?
(a) the accumulation of CO 2 in the atmosphere may cause the greenhouse effect
(b) CO 2 in the air reflects back the earth ' s infrared radiation
(c) the amount of CO 2 in our atmosphere is gradually increasing
(d) CO 2 , once formed, remains in the atmosphere forever

Box 2 F
28. Taxol is a promising drug in treatment of
(a) prostate cancer (b) heart attack (c) stroke (d) ovarian cancer

97

e Saunders CoUege Publishing


ANSWERS TO EXAMINATION QUESTIONS

Chapter 1 of "Introduction to General, Organic, and Biochemistry'; fiflh edition


3 a 4 c 5 a 10 c
12 c 20 a
1 c 2 d 6 c 7 b 8 b 9 b
11 b 13 a 14 a 15 d 16 c 17 b 18 d 19 b
21 b 22 b 23 a 24 a 25 c

Chapter 2 of "hltroduction to General, Organic, and Biochemistry'; fifth edition


1 a 2 c 4 a
12 c 14 c 19 a
3 � 5 b 6 c 7 b 8 c 9 b 10 b
13 a
24 c 29 a 30 c
11 d 15 b 16 c 17 c 18 c 20 b
21 a 22 d 23 a 25 b 26 b 27 d 28 b
31 b 32 d

Chapter 3 of "Introduction to General, Organic, and Biochemistry'; fiflh edition


1 d 2 a 3 d 4 c 5 c 6 a 7 c 8 b 9 c 10 a
13 b 14 a 15 a 17 a
22 c 27 a
11 c 12 d 16 d 18 b 19 d 20 d
21 a 23 b 24 a 25 c 26 c 28 d 29 d

Chapter 4 of "Introduction to General, Organic, and Biochemistry'; fifth edition


3 c 4 a 10 a
c c 16 a a a c
1 c 2 b 5 d 6 d 7 b 8 d 9 d
11 b 12 b 13 14 15 c 17 18 19 20 d
21 b 22 a 23 b 24 d 25 a 26 d 27 b

Chapter 5 of "Introduction to General, Organic, and Biochemistry'; fiflh edition


2 c 5 a 7 a 9 a
c c c a c
1 c 3 b 4 c 6 d 8 b 10 b
20 d
c a a a
11 b 12 13 14 15 b 16 d 17 c 18 19

c
21 b 22 b 23 d 24 25 b 26 27 d 28 29 30 b
31 c 32 a 33

Chapter 6 0f "Introduction to General, Organic, and Biochemistry'; fifth edition


c b c c 6 b a 9 a
c c a 20 c
1 2 3 4 d 5 7 b 8 10 d
b
c 29 a
11 12 13 d 14 15 d 16 c 17 18 c 19 b
21 a 22 d 23 a 24 25 a 26 a 27 c 28 c

Chapter 7 of "Introduction to General, OrgaJlic, and Biochemistry'; fiflh edition


2 a 3 c 4 b 6 a 9 c 10 c
16 a 19 a 20 c
1 c 5 d 7 b 8 d
11 b 12 b 13 a 14 b 15 d 17 c 18 c
21 b

Chapter 8 of "Introductiol1 to General, Organic, and Biochemistry'; fiflh edition


Chapter 6 of '1ntroduction to Organic and Biochemistry'; third edition
3 a 4 c 8 a 9 a
12 c 15 c 16 a 19 a 20 c
1 b 2 d 5 b 6 b 7 b 10 d
13 b 14 a
c a a
11 c 17 c 18 d
21 b 22 b 23 24 b 25 d 26 d 27 28 d 29

99

() Saunders College Publishing


Chapter 9 of Ulntroduction to General, Organic, and Biochemistry'; fifth edition
1 a c 10 c
11 c a a a 16 a 17 a 18 a 20 c
2 b 3 d 4 5 d 6 b 7 b 8 d 9 b

c 26 c
12 13 14 15 b 19 b
21 b 22 b 23 24 b 25 c 27 b 28 b 29 b 30 b

Chapter 10 of Uhltroduction to General, Organic, and Biochemistry'; fifth edition


1 b 2 b 3 c 4 C 5 b 6 c 7 c 8 c 9 a 10 b

25 a a a
11 d 12 b 13 c 14 c 15 d 16 d 17 b 18 c 19 b 20 b

a
21 b 22 c 23 d 24 b 26 27 c 28 29 d 30 d
31

Chapter 1 1 of '1ntroduction to General, Organic, and Biochemistry'; fifth edition


Chapter 3 of '1ntroduction to Organic and Biochemistry'; third edition
c c 6 a 9 a 10 a
a a c 20 a
1 b 2 b 3 4 b 5 7 c 8 d

c a a
11 b 12 13 14 b 15 16 d 17 c 18 b 19 d

a a
21 22 b 23 d 24 25 26 b 27 c 28 d 29 b 30 d
31 d 32 33 b 34 c 35

Chapter 12 of '1ntroduction to General, Organic, and Biochemistry'; fifth edition


Chapter 4 of '1ntroduction to Organic and Biochemistry'; third edition
c a a a
a a c a
1 2 b 3 c 4 5 d 6 b 7 d 8 9 10 d

a a 30 c
11 b 12 c 13 14 d 15 c 16 b 17 18 19 20 b
25 b b
a
21 d 22 c 23 b 24 26 d 27 28 29 d
31 b 32 d 33 34 d

Chapter 13 of '1ntroduction to General, Organic, and Biochemistry'; fifth edition


Chapter 5 of '1ntroduction to Organic and Biochemistry'; third edition
1 a a 5 c 6 c
a c a c
2 b 3 d 4 7 d 8 b 9 b 10 d

26 a a
11 b 12 13 14 15 b 16 b 17 18 a 19 c 20 b
21 d 22 c 23 d 24 b 25 d 27

Chapter 14 of '1ntroduction to General, Organic, and Biochemistry '; fifth edition


Chapter 7 0f '1ntroduction to Organic and Biochemistry'; third edition
5 a a
a 16 a c
1 d 2 d 3 b 4 d 6 d 7 b 8 9 c 10 b
11 d b 15 b
c a a
12 13 c 14 17 d 18 19 b 20 b
21 22 23 c 24 d 25 c 26 b 27 d 28 b 29

Chapter 15 of '1ntroduction to General, Organic, and Biochemistry'; fifth edition


Chapter 8 of '1ntroduction to Organic and Biochemistry'; third edition
a b b 6 b b c 10 a
a a
1 c 2 3 4 5 d 7 8 9 d

a 26 a a ' a
11 b 12 c 13 14 c 15 b 16 b 17 d 18 19 c 20 b
21 d 22 23 b 24 c 25 b 27 d 28 29 30 d
31 c 32 b

1 00

© Saunders College Publishing


Chapter 16 of "Introduction to General, Organic, and Biochemistry'; fifth edition
Chapter 9 of ''Introduction to Organic and Biochemistry'; third edition
1 b b 3 c 4 c a 6 c a
11 c a 16 a 20 c
2 5 7 b 8 b 9 10 d
b
21 a a a 30 c
12 13 d 14 d 15 17 d 18 b 19 d

31 c
22 b 23 d 24 b 25 26 b 27 b 28 29 d
32 d

Chapter 1 7 of "hltroduction to General, Organic, and Biochemistry'; fifth edition


Chapter 1 0 of "Introduction to Organic and Biochemistry'; third edition
b 2 c 3 c c a 10 a
c a c c
1 . 4 a 5 d 6 7 8 d 9 b

a 22 c a a c a 30 a
11 12 a 13 � 14 . a 15 16 17 18 d 19 b 20 b

c
21 23 b 24 d 25 26 27 28 29 d
31 32 b 33 b

Chapter 18 of ''Introduction to General, Organic, and Biochemistry'; fifth edition


Chapter 1 1 of ' 'Introduction to Organic and Biochemistry'; third edition
c a c 10 a
12 13
5 b
a
1 d 2 d 3 4 d 6 7 d 8 9 d

21 a c 24 c a 27 a c
11 b a d 14 a 15 b 16 d 17 b 18 d 19 20 b
22 b 23 25 b 26 28 29 d 30 d

Chapter 19 of ''Introduction to General, Organic, and Biochemistry'; fifth edition


Chapter 12 of "Introduction to Organic and Biochemistry'; third edition
1 c 2 b a 10 a
13 a c c a
3 b 4 b 5 d 6 d 7 8 d 9 d

22 a
11 d 12 b d 14 b 15 16 17 18 b 19 20 d
21 a 23 a

Chapter 20 of ''Introduction to General, Organic, and Biochemistry'; fifth edition


Chapter 13 of "Introduction to Organic and Biochemistry'; third edition
1 d 2 b 3 d 4 d 5 c 6 c a a c
13 c c c
7 8 9 10 b
11 b 12 a 14 d 15 b
a c a c
16 c 17 d 18 19 b 20
21 22 a 23 b 24 b 25 26 b 27 28

Chapter 21 of "Introduction to General, Organic, and Biochemistry'; fifth edition


Chapter 14 of "hltroduction to Organic and Biochemistry'; third edition
c 2 b 3 5 b c c
12 c 13 14 c a c
1 a 4 a 6 d 7 d 8 9 10 b

22 23 .c
11 a d 15 b 16 b 17 d 18 19 20 b
21 d b 24 d

Chapter 22 of "Introduction to General, Organic, and Biochemistry'; fifth edition


Chapter 15 of ''Introduction to Organic and Biochemistry'; third edition
1 a 2 d 3 a 4 d 5 d 6 a 7 b 8 c 9 d 10 b
11 b 12 a 13 b

101

� Saunders College Publi'lhing


Chapter 23 of "Introduction to General, Organic, and Biochemistry'; fifth edition
Chapter 16 of "Introduction to Organic and Biochemistry'; third edition
1 c c a 5 a c 10 a
11 c a a a
2 b 3 4 6 7 d 8 d 9 b

c 25 a a a
12 b 13 14 b 15 b 16 17 d 18 19 d 20 d

31 a a
21 b 22 23 b 24 b 26 27 c 28 c 29 30 d
32 33 b 34 c 35 c

Chapter 24 of "Introduction to General, Organic, and Biochemistry'; fifth edition


Chapter 1 7 of "Introductioll to Organic and Biochemistry'; third edition
1 c a c a c 10 c
c c a a
2 3 b 4 b 5 d 6 7 8 9 b
11 d 12 b 13 d 14 15 16 17 b 18 19 d 20 b
21 b

Chapter 25 of "Introduction to General, Organic, and Biochemistry'; fifth edition


Chapter 18 of "Introductioll to Organic and Biochemistry'; third edition
a a a
11 c a a a
1 d 2 c 3 4 5 b 6 c 7 b 8 9 d 10 d

a a c
12 b 13 14 15 d 16 17 d 18 c 19 b 20 d
21 22 23 24 b 25 c

Chapter 26 of "Introduction to General, Organic, and Biochemistry'; fifth edition


Chapter 19 of "Introduction to Organic and Biochemistry'; third edition
c a 5 a a
c a
1 2 b 3 4 d 6 b 7 c 8 9 d 10 b
11 12 b 13 d 14 15 b 16 d

Chapter 1 of "Introductioll to Organic and Biochemistry'; third edition


2 a 6 a 9 a 10 a
a a a a
1 b 3 d 4 d 5 c 7 d 8 b

a
11 d 12 13 b 14 d 15 16 b 17 c 18 19 20 d
21

Chapter 2 of "Introduction to Organic and Biochemistry'; third edition


1 c 2 c a a 6 b 7 d c 10 c
c c c
3 4 c 5 8 b 9

a a
11 d 12 d 13 b 14 15 b 16 b 17 b 18 19 b 20
21 d 22 b 23 24 c 25 26 d 27 d 28 d

1 02

C Saunders CoUege Publishing


ANSWERS TO EVEN-NUMBERED END-OF- CHAPI'ER
PROBLEMS

Chapter 1 of "Introduction to General, Organic, and Biochemistry'; fifth edition:


Matter, Energy, and Measurements

1 . 16 It deals with the structure and properties of matter and the transformations from
one form of matter to another.
1. 18 (a) ariything that has mass and takes up space (b) the science that deals with
matter
1 .20 Scientists will accept a theory if it agrees with all known facts. But if new facts
arise that conflict with the theory, the theory is wrong.
2
1 .22 (a) 3 . 75 1 0 1 (b) 6.29 1 04 (c) 9. 1 1 0- (d) 2.8 1 0-7
x X X x

1 .24 (a) 6.48 1 07 (b) 1 .6 l Os (c) 4.69 lOs (d) 2.8 1 O- 1 s


x x x x

1 .26 (a) 1 . 3 1 l O s (b) 9.4 1 04 (c) 5. 1 3 7 10-3


x x X

1 .28 4.45 1 06
x

1.30 (a) gram (b) meter (c) liter


1 .32 (a) 1 000 mL (b) 1 mL
1 .34 See Sec. 1 .4.
1 .36 (a) 77 ° F, 298 K (b) 1 04 ° F, 3 13 K (c) 482 °F, 523 K (d) -45 9 ° F, 0 K
1.38 (a) 0.0964 L (b) 27.5 cm (c) 45 700 g (d) 4.75 m (e) 2 1 .64 mL (f) 3290 cc
(g) 44 mL (h) 0.7 1 1 kg (i) 63 .7 cc U) 73 000 mg (k) 83 400 mm (1) 0.36 1 g
1 .40 50 mileslhr
1 .42 7.5 kgIL
1 .44 35.9 mL
1 .46 22 1 g
1 .48 (a) oxygen 1 .492 gIL; carbon dioxide 1 . 977 gIL
(b) Yes. The carbon dioxide smothers flames.
1 .50 Kinetic energy is the energy of motion. Potential energy is stored energy.
3 3
1 .52 (a) 2. 1 1 03 cal (b) 5 . 1 1 0 cal (c) 9.9 1 04 cal (d) 2.5 1 0 cal
x x x x

1 .54 It was a wrong hypothesis. The child did not have lead poisoning.
1 .56 1 90 mg
1 .5 8 13.6 g
1 .60 to generate heat by muscle action
1 .62 (a) 4 (b) 1 (c) 3 (d) 5 (e) 5 (f) 1 (g) 3
1 .64 (a) 92 (b) 7.3 (c) 0.68 (d) 0.0032 (e) 5.9
1 .66 (a) 1 .53 (b) 2.2 (c) 0.00048
1 .68 The answer is 497. The number with the fewest significant figures is 0.000450,
which has three.
1.70 (a) volume (b) volume (c) mass (d) heat (e) density (f) heat (g) temperature
(h) velocity

103

C Saunders College Publishing


1 . 72 56 kg, assuming that the room is empty
1 . 74 1 .24 101 0 erg
x

1 . 76 Kinetic: (b)(d)(e) Potential: (a)(c)


1 .78 (a) The moon has a much lower gravity than the earth. (b) no
1 . 80 See Sec. 1 .9.
1 . 82 (a) mercwy (b) 162 degrees (c) carbon
1 . 84 The quart is the better buy.
1 . 86 (c)

Chapter 2 of "Introduction to General, Organic, and Biochemistry'; fifth edition:


A toms

2.8 Elements: (a)(f)(g) Compounds: (b)(k) Mixtures: (c)(d)(e)(h)(i)G)


2. 10 (a) oxygen (b) lead (c) calcium (d) sodium (e) carbon (f) sulfur
(g) iron (h) hydrogen (i) potassium
2. 12 See Sec. 2.3 .
2. 14 No. CO and CO2 are different compounds, and each obeys the law of constant
composition.
2. 16 (a) 52 (b) 1 9 (c) 79 (d) 210 (e) 226
2. 1 8 (a) scandium (b) titanium (c) silver (d) thorium (e) argon
2.20 (a) 7 (b) 4 1 (c) 1 12 (d) 50
2.22 (a) 22 (b) 1 04 (c) 3 5 (d) 128 (e) 7 (f) 2 3 8
2.24 a name given to both neutrons and protons
2.26 (a) a particle with an unequal number of protons and electrons (hence it has a
charge)
(b) atoms with the same number of protons, but different numbers of neutrons
2.28 (a) 1 7e, 1 7p, 20n (b) 1 8e, 1 9p, 20n (c) 80e, 82p, 125n
(d) 54e, 53p, 74n (e) 44e, 46p, 60n (f) 74e, 74p, 1 12n
2.30 1 .022 amu
2.32 As, P, N; I, P ; Ne, He; Mg, Ca, Ba; K, Li
2.34 (a) aluminum (b) arsenic (c) gallium (d) aluminum
2.36 (a) 2 (b) 8 (c) 18
2.38 In all three cases the o'.lter-shell configurations are the same, except for the shell
number:
(a) Na: 3 s Cs: 6s (b) 0: 2s2p4 Te: 5 s5p4 (c) C: 2s2# Ge: 3 s4#
Each has a different number of filled inner shells.
2.40 (a) P : I s2s2Ji; P- : I s2s2]f'
4 2
(b) S : I s2s2]f'3 s3p ; S -: I s2s2]f'3 s3]f'
2
(c) P : I s2s2]f'3 s3jJ; p3- : same as S -
2.42 7s

1 04

© Saunders CoUege Publishing


2.44 because elements in the same column have the same electron configuration in
their outer shells
2.46 lowest-to-highest: K, Li, AI, B, C, CI, 0, N, Ar
2.48 See B ox 2A.
2.50 Calcium is an essential element in human bones and teeth. Since strontium
behaves much like calcium, strontium-90 gets into our bones and teeth and gives
off radioactivity for many years directly into our bodies.
2.52 because copper is easier to obtain from its ores (lower temperature required)
2.54 (a) I s (b) 2 s2p (c) 3 s3p3 d (d) 4 s4pt dl f
2.56 (a) s-p (by s-p (c) s-jI
2.58 a cluster of two or more atoms connected by chemical bonds
2.60 (a) P (b) K (c) Na (d) N (e) Br (f) Ag (g) Ca (h) C (i) Sn (j) Zn
2.62 1 s-2s2p, 1 s-2s3s, etc.
2 3
2.64 (a) Ne (b) K+ (c) Br- (d) Ba + (e) H- (f) Sc +
2.66 99.9%
2
2.68 6.0 10 1 protons
x

2.70 Any state with an energy higher than the ground state. An atom can be in only
one unique excited state at any time.

Chapter 3 of "Introduction to General, Organic, and Biochemistry'; fifth edition:


Chemical Bonds

3 . 14 (a) lose 1 (b) gain 1 (c) none (d) lose 3 (e) lose 2 (f) gain 2
(g) gain or lose 4 (h) gain 2 (i) none G) gain 3
3. 16 (a)(c)(d)(l)
3.18 A charge of +6 is far too concentrated for a small ion like 06+.
3 .2 0 Yes, copper is a transition element, and the octet rule cannot be applied to
transition elements.
3 . 22 See Sec. 3 . 3 .
3 .24 LiBr LiCI04 Li20 LiN03 Li2S04 Li3P04 LiOH
CaBr2 Ca(CI04)2 CaO Ca(N°3)2 CaS04 Ca3(P04)2 Ca(OH)2
CoBr3 Co(C104)3 C0203 CO(N°3)3 COiS04)3 CoP04 CO(OH)3
KBr KCI04 K20 KN03 K2S04 K3P04 KOH
CuBr2 Cu(CI04)2 CuO CU(N03)2 CuS04 CUlP04)2 CU(OH)2
2 2
3 .26 (a) Na+ Br- (b) Fe + SO/- (c) Mg + PO/-
2
(d) K+ H2P04- (e) Na+ HC03- (f) Ba + N03-
3 .28 See Sec. 3 . 4 .
3 .30 See Sec. 3 . 4 .

1 05

e Saunders CoUege Publishing


3 .32 (a) The left C has five bonds. (b) correct
( c) The second C has three bonds; the 0 has one bond.
(d) The F has two bonds. (e) correct
(f) The second C has five bonds.
3 .3 4 A chlorine atom only fonns one bond, so it cannot fonn a double bond.
3.36 A molecular fonnula gives ratios of atoms; a structural fonnula shows which
atoms are connected to which; a Lewis structure shows all outer-shell electrons.

.
3.38 (a) 8 (b) 1 8 (c) 24 (d) 2 0 (e) 32 (f) 24 (g) 32
3 .40 (a) H-C =N: (b) H-O-N=O: (c) :0: (d) H
I I ..
• I

.
H-O-S-O-H H-C-C-O:
. . .

t.o-c-o:]
I
• •
I 1\
. •

••

:0: H 0:
2
. . .

(e) -
I ' II ••
:0:
3 .42 (a)(c)(e)(g)
3 .44 : C =N:- yes
3 .46 (a) linear (b) planar triangular (c) tetrahedral
3 .48 (a) CI (b) N (c) 0 (d) CI (e) 0 (f) S (g) 0
3 . 50 Yes. The negative charge is at the S atom. The positive charge is halfway between
the hydrogens.

J .
3 . 52 Ionic: (d)(h); Polar covalent: (b)(c)(e)(f); Nonpolar covalent: (a)(g)

r :9: r . :9: l - r � 1
2

� �-N=O:J
(c)
� �-C-2:J
(a) - (b)
l:9.-�-9:
: : 2
-

3 . 54
. . . . . . I , ,

2Na+ K+ 2Cs+
I

2
(d) H + (e) :0: -
. .

I I ..
[:: O-HU - :O-P-O-H
41 41

H-N-H •

I I' 1\ ..
••

H :0:
3 .5 6 (a) KBr (b) CaO (c) PL(OH)2 (d) CUlP04)2 (e) Li2S04 (f) Fe2S3 (g) NH4HS03
(h) Mg(CH3 COO)2 (i) Sr(H2P04)2 G) Ag2C03 (k) �rC12 (1) Ba(Mn04)2 (m) HgO
3 .58 (a) CO (b) BrCI (c) PCl s (d) SiBr4 (e) N203 (f) SF6
3 . 60 component of bones and teeth
3 . 62 Na+, CI-
3 . 64 radiopaque medium in x-ray work
3 . 66 It is bound to two nitrogen atoms by covalent bonds and to the other two nitrogen
atoms by coordinate covalent bonds.

1 06

© Saunders CoUege Publishing


3 .68 (a) K2 S (b) AI(N03 )3 (c) PbS03 (d) Ca(HC03)2 (e) Hg(H2P04)2 (f) BaH2
3 .70 (a) sodium sulfite (b) potassium pennanganate (c) strontium cyanide
(d) iron(U) oxide or ferrous oxide (e) ammonium carbonate (f) lithium nitrite
(g) sodium bicarbonate
3 . 72 Compounds (b) (c) (e) and (f) are polar.
3 . 74 (a)
3 .76 When Lewis structures are drawn, it is seen that the N in NCl3 has an unshared
pair of electrons, while the Al in AlCl3 does not. Hence, in the VSEPR, N has 4
pairs (and a pyramidal geometry), while Al has only 3 (and a planar triangular
geometry).
3 .78 (a) nonmetals (b) metals
3 . 80 coordinate covalent
3 .82 No, the sentence refers to sodium and potassium ions, not to the metals.

Chapter 4 of "Introduction to General, Organic, and Biochemistry'; fifth edition:


Chemical Reactions

4. 1 6 (a) 70.9 amu (b) 3 9 . 9 amu (c) 123 . 9 amu (d) 28.0 amu (e) 4 . 0 amu
.

4. 1 8 (a) 1 . 89 moles CO2 (b) 4.65 moles glycerol


(c) 0. 12 mole NaH2P04 (d) 0. 159 mole quinine
4.20 (a) 1 8 . 1 moles 0 atoms (b) 1 .23 moles Br atoms (c) 4.2 x 1 04 moles 0 atoms
(d) 0.40 mole Hg atoms (e) 3 . 8 x 1 0-s mole N atoms
4.22 The same; that is, just about 2: 1
22
4.24 5.68 10- g
x

4.26 6. 1 1 0 1 8 molecules of cholesterol


x

4.28 (a) 12 + H2 � 2H1


(b) 2Al + 3 02 � Al20 3
(c) 2N a + Cl2 � 2NaC I
(d) CaC0 3 � CO2 + CaO
4.30 (a) the study of weight relationships in chemical reactions (b) a solution in which
the solvent is water
4.32 7.33 moles H2
4.34 1 3 8 g C2H40
4.36 7.5 g CO2
4.38 0. 1 3 g C
4.40 73 .2 g of trichloroaniline
4.42 85 .6%
4.44 85.2%

1 07

e Saunders CoUege Publishing


4.46 Equations (a) and (b) are already balanced.
(c) 2Sc3+(aq) + 3 S0/-(aq) � S C2(S04)3( S)
2 2
(d) Sn +(aq) + 2Fe +(aq) � 2Fe 3+(aq) + Sn(s)
(e) 2K(s) + 2H20(1) � 2K+(aq) + 20H-(aq) + Hlg)
2
4.48 Fe \aq) + 20H-(aq) � Fe(OH)2( S)
4.50 NH/ + OH- � NHlg) + H20
4.52 Soluble: (c) (e) Insoluble: (a)(b)(d)(f)
4.54 Oxidation is ( 1) loss of electrons (2) gain of oxygen or loss of hydrogen.
Reduction is ( 1 ) gain of electrons (2) loss of oxygen or gain of hydrogen.
4.56 (a) Pb gets oxidized; Ag+ gets reduced.
(b) Ag+ is the oxidizing agent; Pb the reducing agent
4.58 (a) An exothennic process is one that gives off heat.
(b) An endothermic process is one that requires heat.
4.60 1 9 600 cal will be given off.
4.62 8 8 . 6 g iron
4.64 The CH4 is oxidized to CO2, The H202 is reduced to H20.
4.66 95 ° C
4.68 A mole of lead (207.2 g) has more mass that a mole of silver ( 1 97.0 g).
4.70 (a) 1 9.2 moles Fe20 3 (b) 1 . 6 1 x 1 0 3 g CO
2
4.72 2As 3 +(aq) + 3 S -(aq) � AS2S 3 (S) The spectator ions are NH/ and N03-.
4.74 893 amu
4.76 H2 kin excess.
4.78 (a) An exergonic process is one that gives off energy in any fonn, not
necessarily heat.
(b) An endergonic process is the opposite.
4.80 an ion that is present, but does not participate in a reaction
4. 82 (a) C4HS + 602 � 4C02 + 4H20 (b) C4HS is oxidized and O2 reduced.
(c) O2 is the oxidizing agent and C4HS the reducing agent.

Chapter 5 of "Introduction to General, Organic, and Biochemistry'; fiflh edition:


Gases, Liquids and Solids

5 . 12 See Sec. 5.2.


5 . 14 (a) 10.3 1 atm (b) 0.033 atm (c) 0.759 atm (d) 1 attri
5. 16 stays the same
5. 18 342 ° C
5 .20 6.2 L
5 .22 The temperature remains 27 ° C
5 .24 4.35 atm
5 .26 1 600 L

1 08

© Saunders CoUege Publi�hing


5 .28 1 . 97 atm
5 . 3 0 (a) 2. 3 3 moles (b) no
5.32 2.8 L
5 . 34 96 ° C
-

5 . 3 6 (a) 24 moles (b) 770 g


5 . 3 8 S02 : 2.86 giL CH4 : 0. 7 14 giL
5 .40 2.68 1 0 24 molecules
x

5 .42 NH3 first, ClOH180 2 last


5 .44 MW = 36 amu,
5 .46 (b)
5 .48 (a) hydrogen bonds (b) London di spersion forces (c) dipole-dipole interactions
5 . 5 0 hexane
5 . 52 (a) 80 mm Hg (b) 1 3 0 mm Hg (c) 3 80 mm Hg
5 .54 (a) HCI < HBr < HI Boiling points increase with molecular weight.
(b) O 2 < HCI < H2 02 Here the molecular weights are about the same, but the
strength of interaction increases: London < dipole < hydrogen bond.
5 . 56 See Sec. 5 . 1 1 .
5 . 5 8 sublimation
5 . 60 conversion of a solid directly to a gas
5 . 62 The entropy increases. In the gaseous state the molecules are spread over a much
larger volume, so there are far more different arrangements in space. This is much
more random, resulting in a larger entropy.
5 . 64 The pulsating blood 5tans to flow in the lower arm; the blood pressure is equal to
the extemal pressure applied. This is the systoli c pre ssure
.

5 . 66 CO is bound to hemoglobin and does not allow it to carry oxygen. Oxygen under
high pressure in the hyperbaric chamber is dissolved in the plasma and is carried to
the tissues without the aid o f hemoglobin.
5 . 68 because the growth of ice crystals killed the affected cells
5 . 70 In evaporation a liquid i', converted to a gas; in sublimation a solid is converted
directly to a gas without going through the liquid state.
5 . 72 See Sec. 5 . 1 0.
5 . 74 (c) water
5 . 76 There is a lot of empty space between gas molecules, so electromagnetic radiation
(including visible light) passes through without much interaction.
5 .78 NH3, because it has a lower mole cul ar weight. 1 .68 times faster
5 . 80 HCI
5 . 82 40 ° C •
5 . 84 See Sec. 5 . 9 .

1 09

� Saunders CoUege Publishing


Chapter 6 of "Introduction to General, Organic, and Biochemistry'; fifth edition:
Solutions and Colloids

6. 14 Glucose is the solute and water the solvent.


6. 1 6 (a) wine, vinegar (b) honey, tea (c) oxygen in rivers, household ammonia
(d) air
6. 1 8 If they are completely soluble in each other, they cannot form a supersaturated
solution (or a saturated solution, either).
6.20 (a) water (b) diethyl ether (c) water
6.22 ( 1 ) Heat the mixture. (2 ) Add more water.
6.24 supersaturated
6.26 The beer is bottled under pressure. The solubility of CO2 in water increases with
increasing pressure. When the bottle is opened, the pressure decreases to 1 atm,
decreasing the solubility of C O2 , Hence the beer loses CO2 (goes flat).
6.28 (a) Dissolve 86.5 g KC I in 1 63 . 5 g water.
(b) Dissolve 6.24 g NaOH in 149.76 g water.
(c) Dissolve 0.96 g formic acid in 299 . 04 g water.
6.30 In each case, to the given weight of solute add enough water t o make the total
volume specified.
(a) 9.0 g N aC I � 250 mL total volume (b) 3 1 g glycine� 625 mL total volume
(c) 5 . 96 g Na2 S04 � 43 . 5 mL total volume
(d) 1 0 . 9 g acetone� 5 1 8 mL total volume
6.32 To 1 . 0 g NaOH add enough water to make 2 50 mL of solution.
6.34 (a) 1.7 M (b) 0 . 3 3 2 M (c) 0.030 M
6.36 0.55 M
6.38 0.0305 M
6.40 0. 109 M
6.42 (a) 72. 5 g (b) 42.0 g (c) 0.35 g
6.44 To 530 mL of 95% ethyl alcohol add 4.47 L of fruit juice.
6.46 2.5 ppm
6.48 The water molecules surround the anion with their positive ends turned toward the
negative ion.
6.50 Strong: (a)(c) Weak: (d) Non: (b)(e)
6.52 A water molecule forms a hydrogen bond with th� OH group o f ethanol.
6.54 between 1 and 1 000 nm
6.56 Colloidal particles are larger than about 1 nm The largest molecules in any gas are
.

much smaller than that. _


6.58 Brownian motion is the random motion of colloidal particles. The higher the
temperature, the faster the motion. The faster the motion, the more collisions, and
more aggregation of the particles, which will eventually settle out.
6.60 -5 .25 ° C

1 10

© Saunders College Publishing


6.62 79 g NaCI
6.64 Semipermeable membranes allow solvent and small molecules to pass through.
Osmotic membranes allow only solvent molecules to pass.
6.66 (a) compartment B (b) compartment B
6.68 from the KCI compartment (A) to the Na3P04 compartment (B)
6.70 globally
6.72 carbon dioxide in the air
6.74 See Box 6B .
6.76 CaC03 (s) + S03 (g) � c aS04 (s) + CO2 (g)
CaS04 (s) + 2H20 (I) � CaS04 • 2H2 S04 (s)
6.78 Hypoglycemia shifts K+ from the extracellular fluid into the cells; K+ controls nerve
transmission to the heart muscle. With lowered K+ concentration this control is
impaired .
6 . 80 Osmolarities: 0.6% NaCI 0.2 1 ; 0.04% KCI 0.0 1 1 ; 0.2% NaHC03 0.048; 0. 72%
= = =

glucose 0.040. Total osmolarity 0 .3 1 . This is very close to that of blood (0.30).
= =

6 . 82 The salt solution has a higher osmolarity than the cucumber. Therefore the water
moved out of the cucumber, causing shrinkage. The prune is a dried plum. Its
osmolarity is higher than that of the salt solution. Thus water flowed into the prune,
swelling it.
6. 84 The solubility of most substances increases with increasing temperature.
6.86 No. The KCI solution is less than 0 . 3 osmolar, and will be hypotonic.
6.88 0.25 micrograms
6.90 (a)
6.92 Add 2.9 mL of pure water to 1 . 0 mL sea water.

Chapter 7 of "Introduction to General, Organic, and Biochemistry': fifth edition:


Reaction Rates and Equilibrium

2
7.8 1 . 82 x 1 0- molelL · min
7. 1 0 1 . They don't have the ac:ivation energy.
2. They don't have the proper orientation.
7. 12 The breaking of chemical bonds i s always endothermic, because bonded atoms always
have a lower energy than non bonded atoms.
7. 14
transition state

III

© Saunders CoUege Publishing


7. 1 6 The higher the concentration, the more molecules there are in th e same volume, so
there are more collisions per unit time.
7. 1 8 As the reaction goes on, the concentrations of reactants decrease. In most cases
the rate is directly proportional to the concentrations of the reactants.
7.20 the nature of the reactants, the concentration, the presence of a catalyst
7.22 Crushing the marble increases the surface area, making the reaction faster.
7.24 No; no. We can start with any proportions of reactants, so at equilibrium they
won't have equal concentrations.
7.26 (a) CO2 + H20 � H2C03
(b) P40lO � P4 + 502
(c) 3 HF + PF3 � 3F2 + PH3
(d) 2CaO � 2 Ca + O2
7.28 K = 1 72
7.30 No. Equilibrium would be reached very quickly, but at equilibrium there would be
virtually no NH2- or H3 0+.
7.32 the set of conditions at which K = 1 .72
7.34 ( a) right (b) right (c) left (d) left (e) no shift
7.36 (a) left (b) right (c) left (d) right (e) no shift
7.3 8 yes, a change in temperature
7.40 not directly by heat, but by mobilizing the immune defense mechanism
7.42 See Box 7B .
7.44 The silver chloride is converted to silver and chlorine (the equation is given in Box
7D).
7.46 At _5 ° C the rate is 0.70 moles of HC I per liter per second; at 45 ° C the rate is 22.4
moles of HCI per liter per second.
7.48 2
[ PO/- ] [ H20 ]
(a) K =
[ OH ] 2 [ H2P04 ]
[ OH- ] [ H30+ ]
(c) K = [ H20 ] 2
7.50 0.026 M
7.52 Increase the pressure (or decrease the volume, which is the same thing), thus
increasing the concentration.
7.54 The activation energy must be less.

lestthan

T 1 00 kcal/mole

1 00 kcallmole
-L /\
A
1

1 12

© Saunders College Publishing


7.56 K = 50
7.58 No; see Figure 7.5.

Chapter 8 of "Introduction to General, Organic, an d Biochemistry'; fiflh edition;


Chapter 6 0f "Introduction to Organic and Biochemistry'; third edition:
Acids and Bases

8. 1 6 (a) HF + ' H2� � H30+ + F-


(b) HBr + H20 � H30+ + Br- ( 100%)
(c) H2S03 + H20 � H30+ + HS03-
(d) H2S04 + H20 � H30+ + HS04- ( 1 00%)
(e) HC03- + H20 � H30+ + CO/-
8. 1 8 (a) HS04- (b) H2B03- (c) 1- (d) OH- (e) H20 (f) NH2- (g) P O/- (h) HP O/-
2
(i) HS- (j) C0 3 -
8.20 Monoprotic: (b)(d) Diprotic: (a)(e) Triprotic: (c)
8.22 (a)(c)(d)
8.24 They react with (neutralize) each other, fonning water and salts.
3
8.26 (a) 1 0- M (b) 1 0-10 M (c) 1 0-7 M (d) 1 O-1 s M
8.28 (a) 8 (b) 1 0 (c) 2 (d) 0 (e) 7
Acidic: (c) (d) Basic: (a)(b) Neutral: (e)
8.30 (a) 8.5 (b) 1 . 2 (c) 1 1 . 1 (d) 6.3
Acidic: (b) (d) Basic: (a)(c)
2
8.32 (a) 1 0- M (b) 1 0-1 3 M (c) 1 0-10 M (d) l (f M (e) 1 0-7 M
Acidic: (b) (c) Basic: (a)(d) Neutral: (e)
8.34 Acidic : MgS04, Ca(N03 )2, NH4CI
Basic: Na3P04, K2C03, BaF
8.36 Acidic: (b)(c)(e)(h) Basic: (a)(f) Neutral: (d)(g)
8.38 (a) H3 0+ + CH3COO- � CH3COOH + H20
(b) OH- + CH3COOH � CH3COO- + H20
8.40 3 . 75 (p� of formic acid)
8.42 (a) 4.85
(b) The 1 M solution has ten times the buffer capacity of the 0. 1 M solution.
8.44 to determine the acid or base concentration of a solution
8.46 (a) 1 .29 (b) 0. 82 (c) 3 . 80 (d) 0.0942
8.48 0.089 N
8.50 (a) To 12 g of NaOH (s) add enough water to make a total volume of 400.0 mL .
(b) To 6. 1 g of Ba(OH)2 (s) add enough water to make a total volume of 1 .0 L.
8.52 0.57 N

1 13

e Saunders CoUege Publi�hing


8.54 0.900 N
8.56 (a) 1 .5 M (b) 16 M (c) 0.035 M (d) 2.8 M (e) 0.0309 M
8.58 magnesium hydroxide
8.60 strong bases
8.62 sodium bicarbonate
8.64 See Box 8E.
8.66 142 g
8.68 0.7 M
8.70 (a) 0 (b) 1 1 0° 1 molelliter
x =

8.72 8 10-2 M
x

8.74 0.900 M
8.76 The acid strength of an acid is inversely proportional to the base strength of its
conjugate base. Therefore the K. value of the conjugate acid of any base tells its
base strength, and a separate table of Ku values is not needed.
8.78 1 1 .2
8.80 (a) H30+ + F- � HF + H20
(b) OH- + HF .::;:::= F- + H20
8 . 82 NH3 + HCI ---+ NH/ + CI-
All the NH3 will be converted to NH4+ by the above reaction. None will remain.

Chapter 9 of "Introduction to General, Organic, and Biochemistry'; fiflh edition:


Nuclear Chemistry

9.8 By accident, a photographic plate placed near a radioactive rock became


blackened .
. 9. 1 0 (a) 4.0 1 0-5 cm or 4.0 102 nm blue light
x x ,

(b) 3 . 0 cm or 3 .0 107 nm microwaves


x ,

(c) 2.7 1 0-5 cm or 2.7 1 0 nm ultraviolet radiation


2
x x ,

(d) 2.0 1 0-8 cm or 0.20 nm x-rays


x ,

9. 12 Longest wavelength: (a) Highest energy: (c)


9. 14 (a) : H (b) l � O (c) ��Ca (d) �:Si
9. 1 6 europium- 15 1
9. 1 8 vanadium-5 1
9.20 berkelium-245
9.22 (a) l�Be ---+ l�B + .� e beta
(b) l��Eu* ---+ l��Eu + Y gamma
(c) l �i T1 ---+ l �gHg + +�e positron
(d) 2��pU ---+ ��U + �He
2 alpha

1 14

© Saunders College PubU"hing


9.24 gamma rays
9.26 a diagram that shows how much radioactive material is left after a certain time
9.28 16 mg
9.30 There are no ways.
9.32 (a) There will be little change after 2 h. The activity will be 24 mCi and
8.9 1 08 counts per minute.
x

(b) 3 mCi will remain in the thyroid, assuming none has been excreted.
9.34 They measure the same property (radiation intensity) in different ways. The
Geiger-Muller counter
' uses an ionized gas; the scintillation counter a phosphor.
9.36 30 meters
3
9.38 0.63 cm
9.40 Radon in the air. We cannot control it outside buildings. Inside buildings,
ventilation can be controlled.
9.42 (a) cobalt-60 (b) selenium-75 (c) strontium-85
'
(d) carbon- 1 1 , technetium-99m (e) mercury- 197, technetium-99m
9.44 Iodine- I 3 I concentrates in the thyroid. Yttrium-90 concentrates in the pituitary
gland.
9.46 (a) 4 (b) 1 (c) 2 (d) 3
9.48 helium
9.50 a neutron (�n)
9.52 See Sec. 9.8.
9.54 about 1 1 ,500 years ago
9.56 about 5730 years
9.58 It knocks electrons out of molecules, producing highly reactive free radicals that
react with molecules in tissues, damaging them.
9.60 Yes, they can detect small differences in the densities of tissues.
9.62 The reaction between a deuterium and a tritium nucleus produces a helium nucleus
and a neutron. The total mass of the helium nucleus and neutron is less than the
total mass of the deuterium and tritium nuclei. The difference in mass is converted
to energy.
9.64 nitrogen- I4
9.66 28 s
9.68 It is eliminated from the body long before the 602 days have passed.
9.70 A neutron is converted to a proton by emitting an electron.
2
9.72 1 .4 10- mrem
x

9.74 curium-246
9.76 the lowest possible energy state
9.78 zirconium-90

1 15

e Saunders College Publishing


Chapter 10 of "Introduction to General, Organic, and Biochemistry'; fiflh edition:
Organic Chemistry. Alkanes

10. 12 (a)(c)(d)(f)
10. 14 (a) KCI (b) NaOH (c) CH3 (CH2)s CH3
10. 1 6 (a) a compound whose origin and properties have been published in a scientific
journal (b) by extraction from a natural source and by synthesis
1 0. 1 8 the conversion of ammonium chloride and silver cyanate to urea
1 0.20 (a) CH3 -CH-CI CI-CH2-CH2-CI
I
CI
(b) CH3 CH2CH2 CH2CH3 CH3-CH-CH2CH3
I
CH3
1 0.22 C 4, N 3, 0 2, S 2, H 1, F 1, Br 1, CI 1, I 1
10.24 ( a) N has 4 bonds (b) both Cs have 5 bonds (d) 0 has one bond
(e) the left C has 5 bonds (f) the right C has 5 bonds, the left C has 3 bonds
(h) the N has 2 bonds All atoms in (c) and (g) have the correct number of bonds.
1 0.26 (a)
10.28 The real molecule has a zig-zag geometry. All the angles are about 1 09.5 0 •
10.30 Tertiary carbons are marked 1. Quaternary carbons are marked •.

(b)

10.32 (a)
( 1) CH3CH2CH2CH2-Br I-bromobutane
(2) CH3CH2-CH-CH3 2-bromobutane
I
Br
(3 ) CH3-CH-CH2-Br I-bromo-2-methylpropane
I
CH3
(4) CH3
I
CH3-C-Br 2-bromo-2-methylpropane
I
CH3

1 16

© Saunders CoUege Publishing


10.32 continued
(b)
( I ) CH3CH2CH2CH2CH2-F I -fluoropentane
(2) CH3CH2CH2-CH-CH3 2-fluoropentane
I
F
(3 ) CH3CH2-CH-CH;zCH3 3 -fluoropentane
, I
F
(4) CH3CH2-CH-CH2-F l-fluoro-2-methylbutane
I
CH3
(5) CH3
I
CH3CH2-C-CH3 2-fluoro-2-methylbutane
I
F
(6) CH3-CH-CH-CH3 2-fluoro-3-methylbutane
I I
F CH3
(7) F-CH2CH2-CH-CH3 l-fluoro-3-methylbutane
I
CH3 CH3
I
(8) CH3-C-CH2-F 1-fluoro-2,2-dimethylpropane
I
CH3
10.34 (a) CH3CH2-CH-CH2CH2CH2CH3
I
CH3
(b ) CH3
I
CH3-C-CH3
I
CH3
(c) CH3-CH-CH-CH-CH2CH2CH3
I I I
CH3 CH3 CH3

1 17

e Saunders CoUege Publishing


10.34 continued
(d) CI CI CI
I I I
CI-C-C-C-CI
I I I
CI CI CI
(e) CH3CH2CH2CH2-CH-aI2CH2CH2CH2CH3
I
CH-CH3
I
CH3
(f)

10.36 (a) ethyl (b) tert-butyl (c) isobutyl (d) chloro


(e) isopropyl (f) pentyl or n-pentyl
10.38 (a) cyclobutane (b) 1,3-dimethylcyc1opentane
(c) I, 1,2,4-tetramethylcyc1ohexane (d) 2-ethyl- l,3-dimethylcyclohexane
(e) isobutylcyclopentane or l-cyclopentyl-2-methylpropane
10.40
(2) CH3 CH2CH3
(1) 0
(3)

cyclopentane <>
methylcyclobutane
A
ethylcyclopropane


)L�t )t{
(4) CH CH' (5) (6)

CH3 H CH3 CH3


1 , I -dimethyl- cis- 1 ,2-dimethyl- trans- 1 ,2-dimethyl-
cyclopropane cyclopropane cyclopropane

10.42 (a) cis-l-bromo-3-methylcyc1opentane (b) . trans-l-chloro-4-ethylcyclohexane


(c) cis- l,2-dimethylcyc1opropane
1 0.44 (a) no (b) yes (c) yes (d) liquid (e) less dense
10.46 (a) C3H6 + 902 � 6C02 + 6H20
(b) C7H16 + 1 1 O2 � 7C02 + 8H20
(c) 2CsH1S + 25 02 � 1 6C02 + 1 8H20

1 18

e Saunders CoUege Publishing


1 0.48 alcohol, ether, aldehyde, ketone, carboxylic acid, carboxylic ester, amide
10.50 (a) ether, amine (b) halide, double bond (c) alcohol (d) halide, alcohol
(e) ketone, carboxylic ester (f) ketone, aldehyde
10.52 See Box l OA.
10.54 Gasoline dissolves body oils and extracts them from the skin, leaving the skin
"chapped." Petroleum jelly does not dissolve body oils and so they stay in the
skin, but it protects the skin from moisture.
10.56 Most of the CO2 in the atmosphere is converted to H2C03 in rain and in oceans,
and then to so�id carbonates.
10.58 four
10.60 (a) PCH3 (b) p CH3
P tl P p p
CH3 -CH2-CH -CH3 CH3- i qCH2-Cl
j
s

p CH3
(c)

1 0.62 ( 1 ) CHlCH2)sCH3 heptane


(2) CH3-CH-CH2CH2CH2CH3 2-methylhexane
,
CH3
(3) CH3CH2-CH-CH2CH2CH3 3-methylhexane
I
CH3
(4) CH3
I
CH3-C-CH2CH2CH3 2,2-dimethylpentane
I
CH3
(5) CH3-CH-CH-CH2CH3 2,3 -dimethylpentane
I I
CH3 CH3
(6) CH3-CH-CH2-CH-CH3 2,4-dimethylpentane
I I
CH3 CH3

1 19

€:> Saunders CoUege Publishing


10.62 continued
(7) CH3
I
CH3CH2-C-CH2CH3 3,3-dinne�ylpentane
I
CH3
(8) CH3CH2-CH-CH2CH3 3-e�ylpentane
I
CH2
I
CH3
(9) CH3
I
CH3-CH-C-CH3 2,2,3-trime�ylbutane
I I
CH3 CH3
10.64 (a) �e chemistry of carbon compounds
(b) a compound containing only C and H
(c) a hydrocarbon with only single bonds
(d) an atom or group of atoms �at substitutes for a hydrogen
10.66 for example: (a) CH3CH2CH2CH2CH2-0H
(b) CH3-CH2-C-OH (c) CH3-C-CH3 (d) CH3-O-CH2CH2CH3
II II
o 0
(e)
o-OH
10.68 There is a much greater percentage increase in molecular weight in going from C 1
to C2 or from C2 to C3 than from Cs to C9 or from C9 to C lO •

10.70 ( I ) Br-CH-CH2CH3 1, I-dibromopropane


I
Br
(2) Br-CH2-CH-CH3 1,2-dibromopropane
I
Br
(3) Br-CH2CH2CH2-Br 1,3 -dibromopropane
(4)
2,2-dibromopropane

1 20

© Saunders CoUege Publishing


Chapter 1 1 of "Introduction to General, Organic, and Biochemistry'; fiflh edition;
Chapter 3 of "Introduction to Organic and Biochemistry'; third edition:
Alkenes, Alkynes, and Aromatic Compounds

1 1 . 12 Alkenes: (a)(d) Alkyne: (c)


1 1 . 14 (a) propene (b) 2,3-dibromo-2-butene (c) 4,4,4-trichloro- 1-butene
(d) 2-methyl- 1-butene (e) 2-ethyl-4-methyl- 1-pentene
(f) 3 -methylcyclopentene (g) 4-chlorocyclohexene
1 1. 16 (a) becaus e the position of the double bond determines the numbering of carbons
( I-propene and not 3-propene)
(b) CH2=CH -CH -Cl (c) no
,
CI
1 1. 18 three
1 1 .20 (a) cis-2-pentene (b) trans- 1,4-dichloro-2-methyl-2-butene
(c) cis-2-chloro-3-heptene (d) cls-3 ,4-dimethyl-3-hexene
1 1 .22 (a) CI-CH2CH2-CI (b) CH3CH2-Br (c) CH3CH2-OH
(d) CH3CH3 (e) Br-CH2CH2-Br
Pt
+ �
1 1 .24
(a) CH3CH=CH2 H2 CH3 CH2CH3
(b) CH3CH=CH2 + HBr � CH3-CH-CH3
I
Br
(c) CH3CH=CH2 + Br2 � CH3-CH-CH2-Br
I
Br
H2S 04
(d) CH3CH=CH2 + H20 � CH3 - C H- C H3
I
OH
1 1 .26 The product actually formed in each case is written on the left side.

(b)

(c)

© Saunders College Publishing 121


1 1 .26 continued
(d) CH3CH2CH2-CH-CH2CH3 the only product in this case
I

(e) �CH3
Br
C(CH3
U "Br
Br
1 1 .28 not by a direct addition, since RCl + propene gives 2-chloropropane
1 1 .30 (a) -CH2-CH-CH2-CH-CH2-CH-
I I I
F F F
(b) -CH-CH -CH CH -CH CH
I I
- - ­

I I I I
CI CI CI CI CI CI
1 1 .32 one sigma, two pi
1 1 .34 (a) CH3-C = C-H (b) Br-CH2-C =C-CH2-Br
(c) CH3
I
I-C = C-CH-C-CH3
I I
Br CH3
1 1 .36

Neither represents the real benzene, which is an average of the two.


1 1 .3 8 The entire molecule (all 12 atoms of C6H4C12) lies in a single plane. No groups
are above or below this plane.
1 1 .40 Benzene can cause liver damage and cancer,
1 1 .42 (a) 1,2,3-trichlorobenzene (b) 3,4,5-trichlorotoluene
(c) 2,3,5,6-tetraiodophenol (d) 2,5-dimethylbenzoic acid
(e) 2-chloro-6-vinylbenzoic acid (f) 2-methyl-3-phenylpentane
(g) 1 ,2-diphenylethane
1 1 .44 (a) RN03, H2S04 (b) Br2, Fe (c) ftrst C12, Fe; then Br2, Fe, or the reverse
'
(d) ftrst RN03, H2S04; then concentrated H2S04, or the reverse
1 1 .46

CI

1 22

© Saunders CoUege Publishing


1 1 .48 (a) a compound that contains a ring in which at least one atom is not carbon
(b) a heterocyclic compound with an aromatic loop of electrons
1 1 .50

1 1 .52

1 1 .54 p-carotene: 40 carbons; vitamin A: 20 carbons


1 1 .56 See Box 1 1 C.
1 1 .58 See Box l ID.
1 1 .60 CH3 CH3
I I
C=CH2 + H+ � +C - CH3
I I
CH3 CH3
carbocation
CH3 CH3 CH3
I I I
C=CH2 � +C - CH2 - C - CH3
I I I
CH3 CH3 CH3
dimer
1 1 .62 because natural rubber is not the best material for all applications where an
elastic substance is needed
1 1 .64 A compound that causes cancer. Condensed aromatic hydrocarbons, such as

&N02 A
benzo(e )pyrene.

N
1 1 .66 C a)

o Br � Br N02 A
0 Br��
° Br
N02
(b)

1 23
� Saunders College Publishing
1 1 .68 (a) 2-Methyl-2-pentene does not have cis and trans isomers.
(b) The prefix meta may not be used if 3 or more groups are on a benzene
nng.
(c) Since C forms only four bonds, there cannot be two methyl groups
attached to C-2, which already has a single and a double bond.
(d) A lower number would be achieved by counting from the other end.
(e) A double bond takes precedence over a halogen.
(f) When a methyl group is on a benzene ring the parent name toluene should
be used.
(g) and (h) There cannot be a bromine attached to the carbons indicated, which
already have 4 bonds without it.
(i) There is a missing number. The position of the double bond is not given.
1 1 .70 A triple bond requires a linear arrangement of four atoms: C-C = C-C . If they
were in a 6-membered ring, the bonds would have to bend this way:
/
C=C
,
which is impossible.
C C
1 1 .72 An unsaturated compound has at least one double or triple bond. A saturated
compound has none.
1 1 . 74 (a) A double bond in a hydrocarbon must be given the lowest possible
numbers, in this case 1 and 2.
(b) The correct name is 3,4-dimethylcyc1ohexene.
2 3
1 1 . 76 sp: 4,5 Sp : 2,3 Sp : 1
1 1 .78 The atomic orbitals that make up the pi bond overlap less than those that make up
the sigma bond.

Chapter 12 of Ulntroduction to General, Organic, and Biochemistry'; fifth edition;


Chapter 4 of "Introduction to Organic and Biochemistry'; third edition:
Alcohols, Phenols, Ethers, and Halides

12.8 In an alcohol the O H is connected to an aliphatic carbon; in a phenol it is


connected directly to an aromatic ring.
12. 10 Primary: (a)(e)(h) Secondary: (b)(f)(g)(j) Tertiary: (d)(i)(k)
Primary and secondary: (c)
12. 12 See Problem 12. 10 and its answer.
12. 14 (a) HO-CH2CH2-CH-CH2CH3 (b) CH3-CH-CH-CH2CH2CH3
I I I
C� C� OH

1
(c) CH3-CH-CH-CH2CH2 CH3 (d) CH3-CH-CH-CH2CH2CH2CH3
I I I
OH CH3 CI OH

1 24

© Saunders CoUege Publishing


HOn
12. 14 continued
(e)

�Br
Br

rO
12. 1 6 (a), (e) n o reaction (b)

B
(c) CH3 CH2-C-CH2CH3 (d) CH3CH2CH2-COOH
II

©rCOOH
o
(f)

12. 1 8 (a) ethanol that has bef;n made unfit to drink (b) 100% alcohol
(c) a solution in which the solvent is ethanol
12.20 (a) phenol (b) para-bromophenol (c) ortho-methylphenol (ortho-cresol)
(d) 4-bromo-2-nitrophenol
12.22 (a) phenol (b) vanillin (c) BRA or BHT
12.24 (a) ethyl methyl ether (b) diethyl ether (c) diisopropyl ether (d) isopropyl
methyl ether (e) n-pentyl phenyl ether (f) tetrahydrofuran
12.26 ( 1 ) (a) (2) (e) (g) (3) (j )(k)(l) (4) (f) (5) (c)(d)(i)(m) (6) (b) (h)
12.28 They are acidic.
12.30 (a)(d)(e)
12.32 Soluble: (a)(c)
12.34 Ethylene glycol, with two OH groups, participates in more extensive hydrogen
bonding than I-propanol.
[ 0]
12.36 (a) 2CH3CH2-SH � CH3CH2-S-S-CH2CH3

12. 3 8 (a) chloroform (b) carbon tetrachloride (c) methyl chloride, ethyl chloride (d)
triiodomethane (iodoform) (e) dichlorodifluoromethane
12.40 because it is a threat to the ozone layer
12.42 The CO2 in the champagne causes the alcohol to get into the bloodstream faster.

1 25

C Saunders CoUege Publishing


12.44 It causes nausea, sweating, and other unpleasant symptoms because acetaldehyde
is built up. See Box 12B.
12.46 It blinds and kills.
12.48 4-n-hexylresorcinol
12.50 Advantages : easy to administer and safe. Disadvantages : highly flammable,
slow-acting, causes nausea, irritates the respiratory tract.
12.52 Hexachlorophene proved too toxic.
12.54 Hashish has a higher concentration ofthe active ingredient, THC.
12.56 Low iodine levels in the soil mean low iodine levels in the food; hence there is
not enough iodine available for the body to manufacture the hormone.
12.58 (b)(c)(a)
12.60 CH3SH is not capable of forming hydrogen bonds with other molecules of itself;
CH3 0H does form such bonds.
12.62 The boiling point of 2· hexanol is too high, hence its vapor pressure is too low for
rapid evaporation to take place.
12.64 (a) CH3CH2-COOH (b),(c) no reaction (d) CH3CH2-C-CH2CH3
"
o
12.66 Menthol is an alcohol, hence neutral; thymol is a phenol, hence acidic.
12.68 Because they have more -OH groups, ethylene glycol and (even more) glycerol
molecules are held more closely together by hydrogen bonds, making it harder
for the molecules to slide past each other.
12.70 Acidic: phenols, thiols Neutral: alcohols, ethers, halides, thioethers, disulfides

Chapter 13 of "Introduction to General, Organic, and Biochemistry'; fifth edition;


Chapter 5 of "Introduction to Organic and Biochemistry'; third edition:
Aldehydes and Ketones

13.8 Aldehydes have the fonnula R-C-H or H-C-H, where R = an alkyl group or
II II
o 0
aromatic ring. Ketones have the formula R-C-R' .
II
o
a k k
1 3 . 10 (a) H-C-CH2CH2-C-C-CH3 (b) Q (c)
II II If II
o 0 0

a C-H

o
II
1 26

© Saunders College Publishing


13. 10
(d) (y 2 2lr
continued

k
O
CH CH II
0
k
(e) HO-C-CH2CH2-C-CH2-C-NH2
II
0
II
0
k
°

13. 12 (a) benzaldehyde (b) acetone (c) methyl n-propyl ketone


(d) formaldehyde (e) acetaldehyde (t) acetophenone
1 3 . 14 (a) CH3 -CH-CH2-C-CH2CH3 (b)
I 1\
CH3 0

1 3 . 16 4-Heptanone is much bigger, and water cannot solvate the two nonpolar ends of
the molecule.
13. 18 Two molecules of 2-propanol form hydrogen bonds with each other; two
molecules of acetone cannot do this.
1 3 .20 (a) CH3 CH2-C-OH (b) no reaction (c)
II
o
(d) no reaction
1 3 .22 a ketone with a hydroxy group on the carbon next to the C=O group (an a­
hydroxy ketone)
13 .24 (b)(c)(e)
1 3 .26 because an aldehyde carbon is always a primary carbon and a ketone carbon is
always a secondary carbon
0+ 0-
1 3 .28 -C=O
I

1 3 .30 acetals: (a)(d) hemiacetals: (b)(e) neither: (c)(t)


13 .32 hemiacetals: (a)(b)(d) neither: (c)
13 .34 (a)
� C-CH3 (b) no reaction

g g + CH,OH

1 27

e Saunders CoUege Publishing


©( g '© C
(c) (d) HO-CH2-CH-CH2CH2-C-H
I II
+ CH30H OH 0
+ CH3 CH20H

(e) CH3 CH2-C-CH-CH2CH2-OH


II I
o CH3
13.38 (a) CH3 -CH=CH-OH
(b) two enol forms: CH3-CH=CH-CH3 and CH3-CH2-CH=CH2
r I
OH OH


(c)
(d)
OH

13 .40 to make polymers


13 .42 It indicates diabetes.
1 3 .44 Do not open the bottle inside a room; only in the open air or in a fume hood.
Even in the open air great care is needed.
13 .46 sleeping pill
1 3 .48 yes
1 3 .50 because the measurements are made in rats, and the results do not always cany
over to humans
1 3 .52 ketone, cyclic alkene
13 .54 (a) 5-chloro-2-pentanone (b) 3-phenylpropanal (c) 4-hexen-3-one
(d) 2,5-dimethy1cyclohexanone (e) vanillin
13 .56 Formaldehyde (b.p. -2 1 ° C) is a gas at room temperature. Acetaldehyde (b.p.
2 1 ° C) is a gas in the summer and a low-boiling liquid in the winter.
13.58 The enol form is stabilized by internal hydrogen bonding:
13 . 60 Ketones have pleasant odors. Short-chain aldehydes have unpleasant odors; long­
chain ones pleasant.
1 3 .62 (a) and (b)
1 3 .64
CH3CH2CH2 -CH
/O ......CH2
'o -CH2
I This is an acetal.

13 .66 It tautomerizes to form the more stable compound acetone: CH3 -C-CH3
II
o

1 28

© Saunders College Publishing


Chapter 14 of "Introduction to General, Organic, and Biochemistry'; fiflh edition;
Chapter 7 of "Introduction to Organic and Biochemistry'; third edition:
Carboxylic A cids and Esters

14. 1 0 (a) butanoic acid (b) 3-chlorobutanoic acid (c) 2,3-diphenylpropanoic acid
(d) 4-bromo-5,5-dichloro-3-methylpentanoic acid
14. 12 (a) formic acid (b) caproic acid (c) benzoic acid (d) succinic acid
(e) salicylic acid (1) stearic acid
14. 14 (a) hexanoic a9id (b) decanoic acid (c) 2,2-dimethylpropanoic acid
14. 16 ( a) unbranched carboxylic acids from C3 to C20
(b) The esters of most of them are found in fats.
14. 1 8 �O · · · H-O
-7 "-
CH3-C "
� C-CH3
O-H · · · 0

14.20 (c )(a)(b)
14.22 The -COOH group is responsible for the solubility. The CH3-(CH2)4- chain
prevents greater solubility.
14.24 a tricarboxylic acid
14.26 oxalic acid
14.28 (a) potassium butyrate (b) calcium succinate (c) ammonium palmitate
(d) magnesium caprate (e) sodium terephthalate
14.30 (a) HOOCCH2COOH (b) HOOCCH2COO- (c) -00CCH2COO­
14.32 It turns into the acid CH3COOH.
14.34 Add aqueous NaOH solution. Caprylic acid will be converted to sodium
caprylate, which is soluble in water. I-heptanol stays out of solution.
14.36

@g
The alcohol or phenol part of each carboxylic ester is circled. (b), (d), and (g) are
not esters.
(C) -H

(f) �
HO-CI
0 CI
o 0

(h)Q-C-CH2-C.J
� I
O�
I 'C:/'
o 0

1 29

@ Saunders College Publishing


14. 3 8 (a) n-propyl fonnate (b) methyl butyrate (c) n-propyl acetate (d) isopropyl
valerate (e) dimethyl glutarate (f) isobutyl myristate (g) tert-butyl caprylate
(h) phenyl benzoate
14.40 About the same solubility, because both fonn hydrogen bonds with water equall
well, and they have the same molecular weight.
14.42 Carboxylic acids: (a)(g)(k) Carboxylic esters: (d)(i) (j )(k)
Acyl chloride: (c) Anhydrides: (e)(h) None of these: (b)(f)
--+
H+
14.44 (a) CH3CH2CH2COOCH2CH3 + H20 CH3CH2CH2COOH + CH3CH20H

--+
OH-
(b) CH3CH2CH2COOCH2CH3 + H20 CH3CH2CH2COO- + CH3CH20H

14.46 (a) CH3COO- + CH30H

©rO-
(b) CH3COOH + CH30H

(f) CH3CH2COO- + HO-CH2CH2-OH


(g) CHlCH2)gCOO- + CH3-CH-CH20H
I
CH3
14.48 (a) o
"
CH3CH20-P-OCH2CH3
I
OH
(b) two, one, and none, respectively
14.50 0 0 0 , 0 0 0
II II \I II , II II
HO-P-OH + HO-P-OH + HO-P-OH --+ HO-P-O-P-O-P-OH + two H20
I I I I I I
OH OH OH OH OH OH
14.52 (a) 0 0 0 0 0 0
" II " II II II
-0-P-0-P-0-P-OCH3 + H20 --+ -O-P-OH + -0-P-0-P-OCH3
I I , , , I
0- 0- 0- 0- 0- 0-

© Saunders College Publishing


130
14.52 continued
(b) o 0 o o
II II \I II
-0-P-0-P-OCH3 � -O-P-OH + -0-P-OCH3
I I I I
0- 0- 0- 0-

14.54 0 O . .
/I
\I
HO-P-O-P-OH + ' CH3CH20H
I I
OH OH

14.58 to reduce or prevent the occurrence of strokes and heart attacks


14.60 See Box 14D.
14.62 They are made of a polyester that hydrolyzes in the body to its components,
lactic and glycolic acids. These are normal compounds in the body .
14.64 See if it dissolves in 5% NaOH solution; the acid dissolves, the ester does not.
14.66 Because the reaction with the carboxylic acid is an equilibrium, it is necessary to
shift the equilibrium to the right to get a good yield. With the other two reagents
this is not necessary.
14.68 They are acidic and form salts with bases.

14.70 Yi U0 ?!0
(a) CH3 - -OCH2CH3 (b) CH3- -0- -CH3 (c) CH3- -CI �
o 0
14.72 Base catalysis is better.
14.74 Less soluble. Ionic compounds are generally soluble in water; high-molecular
weight carboxylic acids are not.

1 31

© Saunders CoUege Publishing


Chapter 15 of "Introduction to General, Organic, and Biochemistry': fiflh edition;
Chapter 8 of "Introduction to Organic and Biochemistry': third edition:
Amines and Amides

1 5 . 12 one
1 5 . 14 (a) n-propylamine (b) diethylamine (c) ethyl-n-propylamine (d) tert-butylamine
(e) isopropyldimethylamine (f) ethylmethyl-n-propylamine (g) triphenylamine
(h) aniline (i) meta-nitro aniline (j) N-methylaniline (k) meta-vinyl-N-methyl­
N-phenylaniline (1) piperidine (m) 2,5-dichlorocyc1opentylamine (or l-amino-
2,5-dichlorocyc1opentane)
1 5 . 16 (a) (2) (b) ( 1)(2) (c) (2) (d) (2) (e) ( 1)(2) (f) (3) (g) (3) (h) (2) (i) (2) (j)( I )(2)
15. 1 8 The four electrons in the two double bonds and the two unshared electrons on the
nitrogen form an aromatic sextet.
1 5.20 Like alcohols, they form hydrogen bonds with water.
rAr
Y
1 5.22 (a) CH3CH2NH3 + HS04- (b) NH3+

Cl
CH,COO-

CH3
I +
(c) CH3 - CH-NH2 -CH3 H2P04-

©rcoo-
1 5.24 (a) ethylammonium chloride (b) diethylammonium bromide
(c) tetraethylammonium hydroxide (d) N,N-dimethylanilinium acetate
1 5.26 Nothing would happen to the alcohol. The amine would be converted to
CH3CH2CH2NH3+ Cl-.
1 5 .28 Add HC l .
15.30 In an amide the nitrogen is connected directly to a C=O group. In an amine the
three nitrogen bonds are connected to any combin�tion of hydrogens, alkyl, or
aryl groups (at least one alkyl or aryl), but not to a C=O group.
1 5.32 (a) formamide (b) N-ethylbenzamide (c) N,N-dimethylacetamide
(d) N-phenylbutyramide (e) N-methylpalmitamide
(f) ortho-bromo-N-ethyl-N-propylbenzamide
15.34 formamide and some of its N-substituted derivatives
15.36 No; add the amine to an acyl halide or an anhydride.

132

© Saunders CoUege Publi�hing


15.38 (a) +NH3CH2CH2CH2CH2NH3+ 2CH3COO-
(b) CH3-C-NH-CH2CH2CH2CH2-NH-C-CH3
\I II
o 0
1 5 .40 (a) and (d) H20 + OH- (b) and (c) H20 + H+
1 5.42 a basic compound obtained from a plant
1 5 .44 (a) amide, double bond (b) secondary amine
(c) phenol, tertiary amine, ether, double bond, secondary alcohol,
(d) ketone� tertiary amine
1 5 .46 See Box 15 k '

1 5 .48 N o. The HCI designation indicates that the phenyl ephrine was converted to its
ammonium salt by the addition of HCI to make it water-soluble.
15.50 acetaminophen
1 5 . 52 -NH-C-(CH2)g-C-NH-(CH2)g-NH-C-
II II II
o 0 0
1 5 . 54 The enhancement of the effect of one drug by another. Barbiturates can be lethal
when taken with alcohol.

0
15.56 no
15.58
II
One is the -C-OCH3 group. The other is the -O-C
o
II
0
-<Q) group ,

1 5 . 60 (a) 2-amino- l-phenylpropane (b) N-methyl-2-amino- l-phenylpropane


15.62 Add the mixture to a solution of HCI in water. The amine will be converted to its
salt, which will dissolve in the water; the alcohol will remain as a separate layer
that can be removed in a separatory funnel. The amine can be recovered by
making the water l ayer basic.
1 5 . 64 (a) It is an ionic compound (a salt). (b) It is a quaternary ammonium salt.
1 5 .66 (a) and (b) are purely synthetic compounds, made entirely in lab oratories. (c) is
not found in nature; it is made from an alkaloid. (d) has no nitrogen and is not
basic.
1 5 . 68 by adding a base, OH-
1 5 . 70 (a) ( 1) (b) (2) (c) ( 1) (d) ( 1 )
1 5 . 72 (a) malonamide (b) cyclopentylmethylamine
(c) cyclopentyltrimethylammonium acetate (d) pyrrole
(e) cadaverine, or 1,5-diaminopentane

133
e Saunders CoUege Publishing
Chapter 16 of "Introduction to General, Organic, and Biochemistry'; fifth edition;
Chapter 9 of "Introduction to Organic and Biochemistry'; third edition:
Carbohydrates

16.6 (a) O2 (b) CO2


16.8 only (c)
16. 10 (1) They rotate the plane of polarized light in opposite directions. (2) They react
at different rates with chiral substances.
16. 12 (a) 2 (b) no stereoisomerism (c) 4 (d) 4 (e) 4 (f) 8 (g) 4
16. 14 Oligo saccharides contain from 3 to 1 0 monosaccharide units; polysaccharides
from 1 0 to thousands of units.
16. 1 6 (a) polyhydroxy aldehydes or ketones, or compounds that will yield them on
hydrolysis (b) simple sugars of the formula CJl2n01l' where n 3 to 9
=

16. 1 8 No. They differ in configuration only at carbon 1 . Enantiomers differ at all
positions.
16.20 (a) - 1 12. 2 ° (b) It changes to -52. 7 ° .
16.22

OR OH
16.24 carbon 6
1 6.26They have numerous alcohol groups that form hydrogen bonds with water.
1 6.28(a)(b) See Sec. 16.6. (c) (d) See Sec. 16.8.
16.30 (a) The ( ) means that it rotates the plane of polarized light to the left.
-

(b) The common name is levulose; laevo means left in Latin.


16.32 sorbitol, also called glucitol
16.34
(a) COOH (b) COOH

H H

OH

1 34
© Saunders CoUege Publishing
16.36 (a) HO-H2C 0 (b) yes

HO-H,C 0 �::t:H
QOH OH
16.3 8 (a) Yes. It consists of the C- l in the lower (left) ring and the two oxygens to which
it is attached (one 0 links the two rings, the other is the 0 in the lower ring). (b)
Yes. It consists of the C- l in the upper (right) ring and the two oxygens to which it
is attached (one is an OH group; the other is the 0 in the upper ring. (c) yes (d)
This compound has a P( 1 �4) glycosidic linkage. Maltose has an ex( 1 �4)
glycosidic linkage.
1 6.40 The amylose forms helixes inside of which the 12 molecules assemble in long
polyiodine chains.
1 6.42 (a) p-n-galactose, p-n-N-acetylglucosamine-6-sulfate (b) P(1 �4) and P( 1 �3)
glycosidic linkages
1 6.44 It decreases; the fluid becomes more watery, less viscous; hence it provides less
lubrication.
1 6.46 Because in most cases nature makes only one of the possible stereoisomers of a
given structure, and the others won't fit.
1 6.48 No; The OH on C- l is in the axial position.
16.50 yes
16.52 B-type blood has B antigen. A person with A-type blood has antibodies against B
antigen. If a transfusion is given, the antibodies will clump the transfused red
blood cells and may cause death.
1 6.54 See Box 16C.
1 6.56 to provide an elastic gel that keeps the retina on the inner wall of the eyeball
16.58 It was found to cause birth defects.
16.60 (a) mannitol (b) mannuronic acid
1 6.62 n-glucose
16.64 An equilibrium is being established among an open-chain and two cyclic (ex and P)
forms (mutarotation).
1 6.66 (b) (d)
16.68 No; the rings are locked.
1 6.70 (a) maltose (b) amylose. amylopectin, starch
16.72 There are sulfate groups at C-2, C-3, and C-6, and also on the nitrogen.
1 6.74 because most of them are sweet
1 6.76 no

135

© Saunders College Publishing


Chapter 1 7 of "Introduction to General, Organic, and Biochemistry': fiflh edition;
Chapter 10 of "Introduction to Organic and Biochemistry': third edition:
Lipids

17.2 Fats have a higher energy yield per carbon atom than carbohydrates. the other
main form of energy storage in the body.
1 7.4 There are three. One of them is

This one has oleic acid in the middle. The other two have myristic and palmitic
acids in the middle.

17.6 (a)

: [
F� C F
OI

F F C
St OI 01 St St
01 St 01
St 01 01
A B C D E F G

(b) there are seven


(c) this is F:


CH2-O-C-(CH2)16CH3
I
CH-0-C-(CH2)7-CH=CH-(CH2)7CH3

I
CH2-OH

17.8 (a) stearic acid. because it has a greater molecular weight
(b) arachidic acid. because saturated acids have higher melting points than
unsaturated ones (see text)
17. 1 0 Among similar compounds melting points increase with increasing molecular
weight.

136

© Saunders CoUege Publishing


17. 12It is mostly made up of short chain fatty acids: C 1 2 and C 1 4.
17. 143 moles.
(a) 15 (b) 10 (c) 5
17. 1 6
The cis double bonds in the unsaturated fatty acids prevent the regular packing of
17. 1 8
the long aliphatic portions of the lipid bilayers, making them fluid-like.
17.20 Peripheral membrane proteins only touch the surface of the membrane; integral
membrane proteins penetrate the lipid double layer-they are partly or fully
embedded in it.

1 7.22 0
II
CH2-O-C-(CH2) 16 CH3 �with oil


ICH-0-C-(CH )
2)7CH=CHCH2CH=CH(CH2)4CH3

ICH2-0-��-0-CH2CH2-N(CH3)3 �
+

0- with vinegar
1 7.24 sphingomyelins, glycolipids
17.26 Fluidity. Cholesterol will not fit into the regular zig-zag packing of the fatty acid
chains. Therefore, it disturbs the crystalline (high-melting) portions in a manner
similar to the cis double bonds of unsaturated fatty acids.
17.28 in gallstones
17.30 the D ring
17.32 The LDL binds to the LDL-receptor molecules in the coated pits. It is then taken
inside the cells where the enzymes liberate cholesterol.
17.34 ( 1 ) The COCH3 side chain is coverted to OH. (2) Ring A loses the CH3 group and
becomes aromatic; its =0 is converted to -OR.
17.36 The functional group on C- l l does not participate in the binding of drug to
receptor.
17.38 Bile salts are synthesized from cholesterol, so they use up cholesterol which means
that less is deposited in the form of plaques.
17.40 (a) 3 alcohol groups; amide; carboxylate ion (b) 3 ketone groups; 2 alcohol
groups, C=C double bond (c) ketone group; 2 alcohol groups; carboxylic acid; 2
C=C double bonds (d) 2 alcohol groups; 4 C=C double bonds; carboxylic acid
17.42 (a) Olestra is not soluble in water because the long carbon chains prevent it, even
though the oxygens can form hydrogen bonds with water. (b) The olestra
molecules are the wrong size and shape for human enzymes.
17.44 Soaps are salts of carboxylic acids; detergents are salts of sulfonic acids.

137

© Saunders College Publishing


1 7.46 because soaps fonn precipitates in hard water; detergents do not
1 7.48 Degradation of the complex lipid must start with the monosaccharide at the end of
the chain. In Fabry's disease this is a-D-galactose.
17.50 Methenolone has an extra CH3 group and the double bond is in a different place.
17.52 It inhibits the enzyme phospholipase A2 .
17.54 It inhibits the synthesis of prostaglandins.
1 7.56 Taurine is a constituent of bile salts and as such emulsifies lipids and helps
enzymes to break them down.
17.58 It plays a role in the implantation of the fertilized ovum in the uterus.
17.60 stearic acid, sphingosine, phosphate, and choline.
17.62 1 75 mg/l00 mL or 1 . 75 gIL
17.64 It is the only one with an aldehyde group instead of a methyl connected to C- 13.
1 7.66 0.75 g

Chapter 18 of "Introduction to General, Organic, and Biochemistry'; fiflh edition;


Chapter 11 of "Introduction to Organic and Biochemistry'; third edition:
Proteins

1 8 .4 fibrous proteins
18.6 nonpolar: (b)(f)(g) polar but neutral: (d) (e) acidic: (c) basic: (a)
18.8 phenylalanine, tyrosine, tryptophan, and histidine.
1 8 . 10 cysteine
1 8 . 12 isoleucine, threonine
1 8 . 14 ( a) It is not chiral (b) It is a dimeric molecule (c) It contains a phenolic group
(d) There is no NH2 group; instead the N is bonded to the R.
1 8 . 1 6 Ionic compounds are solids at room temperature. Amino acids, b eing zwitterions,
are ionic compounds.
1 8 . 1 8 At low pH there is a positive charge; at high pH a negative charge; at the
isoelectric point both charges are present (zwitterion).
1 8.20 Phe-Phe-Phe Phe-Phe-Asn Phe-Asn-Phe Asn-Phe-Phe
Phe-Asn-Asn Asn-Phe-Asn Asn-Asn-Phe Asn-Asn-Asn
1 8.22 (a) Val-Phe-Trp-Asn (b) C-terminal end: Asn N-terminal end: Val
1 8.24 (a) The series of peptide linkages -C-NH-CH-C-NH-CH-
II I II ' I
O R O R
(b) the amino acid with the free NH2 group at the end of the peptide chain
1 8.26 (a) six (b) C-C-N-C
II I
O H

138
@ Saunders CoUege Publishing
1 8 .28 Proteins contain both positive and negative charges; they can react with acids,
taking a proton (the COO- becomes COOH), or a base by donating a proton (NH3+
becomes NH2), and the pH will not change much.
1 8.30 (a) 3 3 27 (b) 203 8000
= =

18.32 18 residues
1 8.34 (b)
1 8 . 3 6 The side chain of proline is part of a ring. This restricts its flexibility.
1 8 . 3 8 An intramolecular bond goes between two parts of the same chain. An
intermolecular bond goes between two different molecules. Intermolecular bonds
are found in the p-pleated sheet; intramolecular bonds in the a-helix and in some
antiparallel pleated sheets.
1 8 .40 The B chains would no longer be connected to the A chains. The tertiary structure
of both chains would be completely different.
1 8 .42 salt bridge
1 8.44 cystine is converted to cysteine
1 8 .46 Any disulfide bridges present will be broken and the protein's tertiary structure
disrupted.
1 8.48 The glutamic acid is linked to the cysteine by its y-carboxyl group rather than the
a-carboxyl group.
1 8.50 See Box 1 8D.
1 8.52 This is unlikely to be detrimental because Asp is similar in structure to Glu (they
are both acidic amino acids).
1 8.54 calcium hydroxyapatite
1 8.56 to destroy bacteria on the surface of the skin
18.58 the particular shape of the keratin molecules of the hair that are stabilized by
disulfide linkages
1 8. 60 more soluble because carbohydrates have many OR groups that fonn hydrogen
bonds with water
2 2
18.62 (a) 4 = 16 (b) 20 400
=

1 8.64 The protein-digesting enzymes in the stomach and intestines would hydrolyze it
before it could reach the blood.
1 8.66 It can stabilize 3-dimensional structures if it is oxidized and forms S-S bridges.
1 8 .68 (a) hydrophobic (b) salt bridge (c) hydrogen bond (d) hydrophobic
1 8.70 glycine
1 8.72 one positive charge: NH/

139
© Saunders College Publishing
Chapter 19 of "Intfoduction to General, Organic, and Biochemistry'; fifth edition,'
Chapter 12 of '1ntroduction to Organic and Biochemistry'; third edition:
Enzymes

19.2 No, some ribonuleic acids can also act as enzymes.


1 9.4 It lowers it.
19.6 trypsin
19.8 (a)
19. 1 0 (a) deaminations (b) hydrolyses (c) hydrogen removal (d) isomerizations
19. 12 (a) apoenzyme (b) coenzyme
19. 14 (a)
1 9. 16 No. An increase of l O o C may decrease enzyme activity or even inactivate the
enzyme.
19. 1 8 At 85 ° C the enzyme is irreversibly denatured. At - 1 0 ° C the structure of the
enzyme is not changed; reversible inhibition occurs because not enough energy is
available for the reaction.
1 9.20 (c)
1 9.22 The other amino acids are necessary to hold the enzyme into exactly that 3-
dimensional shape into which the substrate can fit. If significant changes were
made in the structure, the enzyme activity would decrease or di sappe ar.
19.24 Yes. This is feedback control.
1 9.26 It would degrade the proteins of our bodies.
1 9.28 an enzyme that is active in more than one form
1 9.30 There has been a heart atttack, but no hepatitis.
1 9.32 the liver
19.34 It is denatured by stomach acid and partially digested, thus becoming inactive.
19.36 Succinylcholine relaxes muscles, making it easier to insert a breathing tube.
19.38 to accomodate the =CH2 group of the PEP
1 9.40 para-aminobenzoic acid
19.42 the -SH group
19.44 (a) transpeptidase (b) It cross-links bacterial cell walls.
19.46 because dead heart muscles spill their enzyme contents into the serum
19.48 They are both basic amino acids.
19.50 Since maximum activity is slightly above pH 7, the enzyme works best in blood.
19.52 hydrolases
19.54 (a) ethanol dehydrogenase (b) esterase
19.56 Changes in pH, both higher and lower, affect acidic and basic side chains, and
consequently salt bridges, which changes conformation of the protein, making it
less active.

1 40

© Saunders CoUege Publishing


Chapter 20 of "Introduction to General, Organic, and Biochemistry': fifth edition;
Chapter 13 of "Introduction to Organic and Biochemistry': third edition:
Bioenergetics. How the Body Converts Food to Energy

20.2 (a) two (b) the citric acid cycle and oxidative phosphorylation
20.4 See Sec. 20.2.
20.6 (a) in the matrix (b) They are imbedded in the inner membranes of the
mitoch o n dri a .
20. 8 adenosine and inorgani c phosphate
20. 10 No, the storage is only for about a minute.
20. 12 The two electrons are in the bond connecting the newly arrived hydrogen to the 4
pos iti on o f the pyri dine ring.
20. 14 (a) flavin (b) ADP
20. 16 In NAD+ it is ribose; in FAD it is ribitol, a sugar alcohol (the reduced form of
ribose).
20. 1 8 (a) nicotinamide (b) riboflavin (c) pantothenic acid
20.20 a thiol ester
20.22 (a) the numbers tell how many carbon atoms are in each compound
(b) acetate, oxaloacetate, and citrate, in the order given
20.24 steps (D, Q), Q) , and G)
20.26 two
20.28 (a) 2 (b) NAD+
20.3 0 (a) succinate (b) oxaloacetate
20.32 No, they originate from the oxaloacetate portion.
20.34 conversion of succinate to fumarate
20.36 to carry electrons from NADH or FADH2 and deliver them to the O2 in the matrix
of the mitoch on dria
20. 3 8 The oxygen is reduced to water by the hydrogen ions and electrons derived from
food and produces energy in the form of ATP. The carbon in the food is converted
to CO2 during the process.
20.40 at the flavoprotein, Q enzyme and cytochrome c, stages
20.42 it is lower
20.44 because the energy built into ATP is generated by the osmotic pressure of the
accumulated protons across the semip ermeable inner membrane of the
mitochondrion
20.46 1 08 ATP molecules
20.48 nerve conduction
20.50 (a) because it was first made from nicotine (b) because vitamin producers felt that
the public would associate nicotini c acid with nicotine
20.52 It is passive transport, taking place without the expenditure of energy.
20.54 (a) GTP has guanine instead of adenine (b) about the same

141

© Saunders College Publishing


20.56 The CO2 is exhaled; the H20 is excreted by the body (urine, perspiration, etc.)
20.58 ( 1) a-Ketoglutaric acid is made by oxidative decarboxylation of isocitric acid. (2)
Oxaloacetic acid is made by oxidation of malic acid. (All these are in the form of
their salts.)
20.60 (a) 3 (b) 2
20.62 No. The CO2 comes from the alcohol end of the isocitrate, which was originally
the oxaloacetate that combined with acetyl CoA.

Chapter 21 of "Introduction to General, Organic, and Biochemistry'; fifth edition;


Chapter 14 of "Introduction to Organic and Biochemistry'; third edition:
Specific Catabolic Path ways: Carbohydrate, Lipid, and Protein Metabolism

2 1 .2 glycerol and fatty acids or monoglycerides


2 1 .4 The glucose must be chemically activated before any subsequent step can take
place.
2 1 .6 It occurs in the absence of O2•
21.8 glyceraldehyde 3-phosphate and dihydroxyacetone phosphate
2 1 . 10 oxidative decarboxylation
2 1 . 12 (a) energy consuming (b) energy yielding
2 1 . 14 (a) 2 (b) 0
2 1 . 16 (a) 6 (b) 6 (c) 36
21. 18 Glycolysis is the specific pathway by which the body gets energy from
monosaccharides. Glycogenolysis is the pathway in which glycogen breaks down
to glucose.
2 1 .20 ATP, because hydrolysis of an anhydride (P-O-P) linkage yields more energy
than hydrolysis of an ester (C-O-P) linkage
2 1 .22 (a) the mitochondria (b) they are transported by camitine acyltransferase
2 1 .24 (a) 5 turns (b) 7 turns
2 1 .26 Thiolase cleaves a C2 fragment from the chain and tranfers it to a CoA molecule;
thiokinase converts the RCOOH to RCO-S-CoA.
2 1 .28 fats, because their catabolism produces more ATP molecules than carbohydrates,
per carbon atom
2 1 .30 Yes, they can be metabolized via the common catabolic pathway.
2 1 .32 It is a transamination, the first step in amino acid catabolism.
2 1 .34 because they are toxic
2 1 .36 (a) 3 (b) 4
2 1 .3 8 (a) NH4+ (b) by converting it to urea in the urea cycle
2 1 .40 The breakdown product of heme, bilirubin, is normally removed from the blood
stream by the liver, later to be excreted via the gallbladder and intestines. High
bilirubin levels in the blood indicate that the liver is malfunctioning because it is
not sufficiently removing the bilirubin.

1 42
© Saunders CoUege Publishing
2 1 .42 the globin
2 1 .44 (a) enlarged liver, hypoglycemia, and ketosis (b) missing debranching enzyme (c)
symptoms: muscle cramps and limited capacity to exercise; missing enzyme:
phosphorylase
2 1 .46 the decarboxylation of the monosodium glutamate ingested
2 1 .48 The primary structure is essentially the same in yeast and in humans.
2 1 .50 by limiting the phenylalanine intake
2 1 .52 bilirubin
2 1 .54 (a) Lysine has one more CH2 group. (b) no
2 1 .56 2 moles of ATP .
2 1 .5 8 diabetes
2 1 .60 PEP is able to transfer phosphate to ADP and form one anhydride linkage,
producing ATP in step ® of glycolysis.
2 1 .62 amide, anhydride

Chapter 22 of "Introduction to General, Organic, and Biochemistry'; fiflh edition;


Chapter 15 of "Introduction to Organic and Biochemistry'; third edition:
Biosynthetic Path ways

22.2 ATP
22.4 No, most catabolic reactions take place in the mitochondria; anabolic reactions in
the cytosol.
22.6 (a) carbon dioxide (b) water (c) sunlight
22. 8 It has uracil instead of adenine.
22. 1 0 the same
22. 12 (a) the number of glucose units in the chain (b) a large number; more than a
thousand
22. 14 acetyl CoA
22. 1 6 It converts it to fatty acids and then to fats.
22. 1 8 (a) synthase (b) It picks up a C2 fragment from ACP, combines it with a C3
fragment attached to another ACP, and forms a C4 compound, releasing CO2 •
22.20 a thiol group
22.22 (c)(e)
22.24 when excess food is available
22.26 glyceroI 3-phosphate, palmitoyl CoA, lauryl CoA, and serine
22.28 No, the body synthesizes cholesterol from C2 fragments even if none is obtained in
the diet.
22.30 oxidative deamination
22.32 NH2-C-CH2-C-COO-
II II
o 0

© Saunders CoUege Pu
blishing
1 43
22.34 The chlorophylls have magnesium, while heme has iron.
22.36 (a) indigo blue (b) oxidation of tryptophan
22.3 8 acetyl CoA
22.40 phenylalanine
22.42 four; one in the formation of glycerol 3 -phosphate, one each for the two acyl CoA
molecules and the fourth for the activation of choline

Chapter 23 of "Introduction to General, Organic, and Biochemistry'; fifth edition;


Chapter 1 6 of "Introduction to Organic and Biochemistry'; third edition:
Nucleic Acids and Protein Synthesis

23 .2 chromosomes
23 .4 (a) in the chromosomes (b) outside the chromosomes
23 .6 See Fig. 23 . 1 .
23 . 8 guanine, thymine, uraciJ : cytosine
23 . 1 0 adenine, cytosine, guanine
23 . 12 A nucleoside contains a sugar and a base. A nucleotide also contains a phosphate
23 . 14 a chain of nucleotides (deoxyribose and phosphate units)
23 . 1 6 C-3 and C-5
23 . 1 8 It allowed the builders of the molecular mo del to pair A with T and C with G.
23 .20 hydrogen bonds between the base pairs
23 .22 Because of hydrogen bonding, only A fits opposite T, and only G fits
opposite C .
23 .24 See Fig. 23 . 5 .
23 .26 thousands
23 .28 to separate the strands of the double helix by opening a loop or bubble
23 . 3 0 in the 5 ' ... 3 ' direction
23 . 3 2 from 73 to 93 nucleotides
23 .34 The information in DNA is transferred to RNA, and is then expressed in the
structure of proteins.
23 .36 rRNA
23 . 3 8 the 3 ' end
23 .40 (a) the larger (60S) portion (b) See Sec. 23 . 10.
23 .42 They are the site of protein synthesis; they support mRNA;
. and they carry some of
the enzymes.
23 .44 327 - 2 (for initiation and termination) 325
=

23 .46 (a) It supports the mRNA. (b) It provides the A and P sites for protein synthesis.
23 .48 It is an electrostatic attraction.
23 .50 mRNA contains exons and introns when it is first synthesized, but before it
actually functions the introns are cut out.

1 44

© Saunders College Publi'lhing


23 .52 The DNA fragments into multiples of 1 80 base pairs.
23 .54 DNA and mRNA
23 .56 a functional unit containing DNA sequences that are structural genes, regulatory
genes and control sites
23 .58 electrostatic interactions and hydrogen bonding
23 .60 Restriction endonucleases cleave DNA at specific sites.
23 .62 In genetic engineering a whole new gene is inserted into the genome but in the
com that acquired insect resistance by natural selection, mutations occurre d in the
native genes.
23 .64 The anticancer drug fluorouracil interferes with the methylating enzyme that
manufactures thymidine, thereby inhibiting new DNA synthesis.
23 .66 It is the protein envelope of the virus. The ingredients come from the host cell.
23 .68 (a) reverse transcriptase (b) HIV- l protease
23 .70 by animal studies
23 .72 by a technique called gel electrophoresis
23 .74 the bacterium Haemophylus influenzae
23 .76 because every time an error is introduced it may cause a mutation that can be
harmful or even detrimental to the propagation of the species
23 .78 See Sec. 23 . 12.
23 .80 only by cautioning carriers of defective genes to select noncarrier partners or not
to have children
23 . 82 For each amino acid there is a specific enzyme that attaches it to its own specific
tRNA.
23 . 84 (a) a certain circular double-stranded DNA molecule (b) A gene is a double­
stranded non-circular DNA molecule, or a portion of such a molecule, that codes
for one protein molecule.

Chapter 24 of Hlntroduction to General, Organic, and Biochemistry': fifth edition;


Chapter 1 7 of Hlntroduction to Organic and Biochemistry': third edition:
Chemical Communication: Neurotransmitters and Honnones

24.2 an electrical signal


24.4 (a)(c)(d)(f) See Sec. 24.2. (b) (e) See Sec. 24. 1 .
24.6 acetic acid: carboxylic acid; choline : primary alcohol, quaternary ammonium ion
24. 8 the choline end
24. 1 0 C-3 and C-5
24. 12 14.6 kcal; ATP � cAMP + PPj � cAMP + 2Pj
24. 14 the same, except that guanine replaces adenine
24. 1 6 The cAMP is destroyed by the enzyme phosphodiesterase.
24. 1 8 the last step, when the ion gates are opened

145

© Saunders CoUege Publishing


24.20 It is oxidized by monoamine oxidases.
24.22 No, the two compounds block different histamine receptors.
24.24 Taurine is part of the structure of the bile salt taurocholate, and glycine is part of
glycocholate.
24.26 They are pentapeptides.
24.28 position 8. A neutral, hydrophobic side chain, Leu, in oxytocin is replaced by a
basic side chain, Arg, in vasopressin.
24.3 0 (a) (b) small molecules derived from amino acids (c) peptides (d) steroids
24.32 It is normally water soluble, but becomes insoluble in people with Alzheimer's
disease.
24.34 It prevents release of acetylcholine from the presynaptic vessels.
24.36 Dopamine cannot penetrate the blood-brain barrier.
24.38 Cheese contains tyramine, which dilates the arteries in the head, causing
headaches.
24.40 They contribute to the death of neurons.
24.42 Insulin-dependent patients do not manufacture enough insulin. Non-insulin­
dependent patients have enough insulin, but not enough receptors to which insulin
can bind.
24.44 Aldosterone, a steroid, penetrates the plasma membranes of the target cell and the
nucleus, affecting mineral metabolism.
24.46 Yes, decamethonium is a competitive inhibitor of acetylcholinesterase.
24.48 In alpha-alanine the NH2 group is on the 2 position of propionic acid; in beta­
alanine it is on the 3 position.

Chapter 25 of "Introduction to General, Organic, and Biochemistry'; fifth edition;


Chapter 18 of "Introduction to Organic and Biochemistry': third edition:
Body Fluids

25 .2 thymus, spleen, and lymph nodes


25 .4 erythrocytes, leukocytes, platelets or thrombocytes
25 .6 leukocytes
25 . 8 (a)(b) in the bone marrow (c) in the lymph nodes and spleen
(d) in the bone marrow and spleen
25. 10 (a) to provide proper osmotic pressure (b) to provide, immune reactions
(c) blood clotting
25. 12 See Sec. 25 .2.
25. 14 75% saturation, which means that, on the average, three O2 molecules are bound.
25. 1 6 4 = 1 00 mm Hg; 3 = 36 mm Hg; 2 = 27 mm Hg; 1 = 1 8 mm Hg;
O = O mm Hg
25. 1 8 (a) It decreases. (b) Bohr effect
25.20 four

1 46
© Sau.nders CoUege Publishing
25.22 In emphysema the lungs fill with water, and so cannot accept CO2 from the blood.
The accumulation of CO2 in the blood forms carbonic acid, which lowers the pH.
25.24 urea, creatine, creatinine, hippuric acid, and ammonia
25.26 They are neutralized by HC03' ions in the blood.
25.28 See Sec. 25.8.
25.30 Angiotensin, the active form of angiotensinogen, constricts blood vessels,
increasing blood pressure. Captopril inhibits formation of angiotensin from
angiotensinogen.
25.32 (a) IgE (b) It plays a part in allergic reactions.
25.34 the Fab fragments; they carry the variable regions that bind the antigen.
25.36 many communication molecules, such as cell adhesion molecules, have structures
similar to that of immunoglobulins. They fOlm a superfamily of compounds.
25.38 intermolecular attractions: London dispersion forces, dipole-dipole interactions,
and hydrogen bonds
25.40 drowsiness
25.42 See Box 25B.
25.44 (a) blood plasma, interstitial fluid (b) intracellular fluid
25.46 a lower concentration of estrogens, which help absorb calcium from the diet
25.48 Sodium ions are sunounded by water molecules (hydration). The lower the [Na+]
in the body, the less water is needed, decreasing the amount of water and the blood
pressure.
25.50 The concentration is higher. Since fibrinogen is removed when plasma becomes
serum, the albumin is dissolved in a smaller amount of material, and its
concentration is higher.
25.52 the IgA immunoglobulins
25.54 Less oxygen is canied; more is released to the tissues.
25.56 no
25.58 a drug that increases the volume of urine
25.60 vasopressin
25.62 Where the two circulations meet the oxygen pressure is the same. However, the
saturation of fetal hemoglobin is lower than that of adult hemoglobin. This means
that at the same oxygen pressure fetal hemoglobin is less saturated than the adult
kind, so·fetal hemoglobin can accept oxygen from adult hemoglobin.

Chapter 26 of "Introduction to General, Organic, and Biochemistry'; fifth edition;


Chapter 19 of'7ntroduction to Organic and Biochemistry'; third edition:
Nutrition and Digestion

26.2 (a) no (b) It is a carboxylic ester.


26.4 No, they are equally nutritious.

147
© Saunders College Publishing
26.6 bread, cereal, rice, and pasta (starches)
26. 8 the average daily requirement of calories for a resting body
26. 10 hypertension,cardiovascular disease, and diabetes
26. 12 fats
26. 14 (a) 2 (b) 10
26. 16 Add animal proteins to the diet.
26. 1 8 (a) 1200 to 1500 mL (b) none
26.20 night blindness
26.22 (a) vitamin B 1 2 (b) part of methyl-removing enzyme in folate metabolism
26.24 meat and dairy products
26.26 degradation of the nervous system
26.28 maltose
26. 30 No; amylose does not contain branches.
26.32 Trypsin hydrolyzes proteins only at an arginine or lysine residue; HCI does it
randomly.
26.34 No. High dosages of niacin have been shown to cause health problems.
26.36 pantothenic acid
26. 38 They contain the essential amino acids we canno t synthesize ourselves.
26.40 yes: fats, oils, and sweets
26.42 Nuts contain minerals such as Mn, Zn, Mg, and K, vitamins including niacin,
biotin, and vitamin E, and are also a rich source of essential fatty acids,
carbohydrates, and proteins.
26.44 P ep s in . Removal of part of the stomach reduces natural pepsin production.
26.46 It is a cofactor n e c e s s ary for the operation of certain enzymes.

Chapter 1 of "Introduction to Organic and Biochemistry'; third edition:


Organic Chemistry and Chemical Bonding

1 .6 (a)(c)(d)(t)
1.8 (a) KCI (b) N aO H (c) CH3(CH2)sCH3
1 . 10 the conversion of ammonium chloride and silver cyanate to urea
1 . 12 (a) lose 1 (b) gain 1 (c) none (d) lose 3 (e) lose 2 (f) gain 2
(g) gain or lose 4 (h) gain 2 (i) none (j ) gain 3
1 . 14 (a)( c)( d)(1)
1 . 16 A charge of +6 is far too concentrated for a small ion like 06+.
1. 18 See Sec. 1 . 3 .
1 .20 (a) Na+ Br- (b) Fe2+ SO/- (c) Mg2+ PO/­
(d) K+ H2P04- (e) Na+ HC03- ef) Ba2+ N03 -
1 .22 See Sec. 1 . 4 .
1 .24 S e e S e c . 1 .4.

1 48

© Saunders College Publishing


1 .26 (a) The left C has five bonds. (b) correct
(c) The second C has three bonds; the 0 has one bond.
(d) The F has two bonds. (e) correct
(f) The second C has five bonds.
1 .28 A chlorine atom only fonns one bond, so it cannot fonn a double bond.
1 . 30 A molecular fonnula gives ratios of atoms; a structural fonnula shows which
atoms are connected to which; a Lewis structure shows all outer-shell electrons.
1 .32 (a) 8 (b) 1 8 (c) 24 (d) 20 (e) 32 (f) 24 (g) 32
... . .. ..

1 . 34 (a) H-C =N: . (b) H-O-N=O: (c) :0: (d) H


I I
• •
•• •• • •

H-O-S-O-H H-C-C-O:
I

t �
I
••
II
• •

• •

:0:
. '
H 0:t'

(e) . O-C-O
. ' 2-
t II • •
:0:
1 .36 (a)( c)(e)(g)
1.38 :C=N:- yes
1 ,40 (a) linear (b) planar triangular (c) tetrahedral
1 .42 (a) spherical (b) dumbbell shape (c) a mixture of the spherical and dumbbell
shapes
1 .44 (a)
1 .46 no difference
1 ,48 lithium carbonate
1 . 50 It is bound to two nitrogen atoms by covalent bonds and to the other two nitrogen
atoms by coordinate covalent bonds.
1 . 52 (a) This ion has one more electron than is needed for a complete octet.
(b) The charge density is too high.
(c) This ion does not have a complete octet.
.. t o

1 . 54 :F : H :F : H
I I I I
:F-B + :P-H � :F-B-P-H
I I I I
:F. :
.
H :F : H
• t

1 .56 When Lewis structures are drawn, it is seen that the N in NCl3 has an unshared
pair of electrons, while the Al in AICl3 does not. Hence, in the VSEPR, N has 4
pairs (and a pyramidal geometry), while Al has only 3 (and a planar triangular
geometry).
1 . 58 (a) The carbon on the right has only six electrons in its outer shell.
(b) There are too many electrons. The compound C2H4 has a total of 12 electrons,
but this fonnula shows 14.

© Saunders College Publi'lhing


1 49
Chapter 2 of "Introduction to Organic and Biochemistry'; third edition:
Alkanes

9
C H3
I
. CH3- -CH3
C H3
2. 14 ' C 4,< N 3 , 0 2, S 2, H 1 , F 1 , Br 1 , CI I , I 1
2. 16 (a) N has 4 bonds (b) both Cs have 5 bonds (d) 0 has one bond
(e) the left C has 5 bonds (f) the right C has 5 bonds, the l e ft C has 3 bonds

t
(h) the N has 2 bonds · Al l atoms in (c) and (g) have the correct number of bonds.
2. 1 8 Teliiruy carbons are marked t . Quatemruy carbons are marked •.

(a) CH3 CH3 (b)

Q
. 1 I
CH3
CH3-C-CH2-CH-CH3
I t
CH3
(c) none CH3 CH3

2.20 (a)
( 1 ) CH3CH2CH2CH2-Br I -bromobutane
(2) CH3CH2-CH-CH3 2 -bromobutane
I
Br
(3) CH3-CH-CH2-Br I -bromo-2-methylpropane
1
CH3
(4) CH3
I
CH3-C-Br 2-bromo-2-methylpropane
I
CH3
(b)
( 1 ) CH3CH2CH2CH2CH2-F 1-fluorop entane
(2) CH3CH2CH2-CH-CH3 2-fluoropentane
I
F

1 50

© Saunders College Publishing


2.20 continued
(3) CH3CH2-CH-CH;zCH3 3 -fluoropentane
I
F
(4) CH3CH2-CH-CH2-F I -fluoro-2-methylbutane
I
CH3
(5) CH3
I
CH3CH2-C-CH3 2-fluoro-2-methylbutane
I .
F
(6) CH3-CH-CH-CH3 2-fluoro-3 -methylbutane
I I
F CH3
(7) F-CH2CH2-CH-CH3 1 -fluoro-3-methylbutane
I
CH3 CH3
I
(8) CH3-C-CH2-F 1 -fluoro-2,2-dimethylpropane
I
CH3
2.22 (a) CH3CH2-CH-CH2CH2CH2CH3
I
CH3
(b) CH3
I
CH3-C-CH3
I
CH3
(c) CH3-CH-CH-CH-CH2CH2CH3
I I I
CH3 CH3 CH3
(d) CI CI CI
I I I
CI-C-C-C-CI
I I I
CI CI CI
(e) CH3CH2CH2CH2-CH-Gl2CH2CH2CH2CH3
I
CH-CH3
I
CH3

151

© Saunders College Publishing


2.22 continued
(f)

2.24 (a) ethyl (b) tert-butyl (c) isobutyl (d) chI oro
(e) isopropyl (f) pentyl or n-pentyl
2.26 (a) cyclobutane (b) 1,3-dimethylcyclopentane
(c) 1 , 1 ,2,4-tetramethylcyclohexane (d) 2-ethyl- l,3-dimethylcyclohexane

l'CHJ
(e) isobutylcyclopentane or l -cyclopentyl-2-methylpropane
2.28

:2t (2) �
(3)

�e
methylcyclobutane ethylcyclopropane

�CH' 'ACH,
CH3WCH3
(4) (5)

l , l -dimethyl­
CH3 H
cis- 1 , 2 -dimethyl­
(6)

trans- I , 2-dimethyl­
cyclopropane cyclopropane cyclopropane

2.30 (a) cis l -bromo-3 -m ethylcyclopentane (b) trans-l-chloro-4-ethylcyclohexane


-

(c) cis- l ,2-dimethylcyclopropane


2.32 (a) no (b) yes (c) yes (d) liquid (e) less dense
2.34 (a) C3H6 + 902 -;. 6C02 + 6H20
(b) C7H 1 6 + 1 1 O2 � 7C02 + 8H20
(c) 2CsH 1 S + 2502 � 1 6C02 + 1 8H20
2.36 because in nearly all cases it produces a complex mixture of products
2.38 alcohol, ether, aldehyde, ketone, carboxylic acid, carboxylic ester, amide
2.40 (a) ether, amine (b) halide, double bond (c) alcohol (d) halide, alcohol
(e) ketone, carboxylic ester (f) ketone, aldehyde
2.42 Each molecule rapidly and spontaneously converts to the other.
2.44 Petroleum is a mixture of compounds with a wide range of boiling points. Each
use requires a much narrower boiling point range.
2.46 They absorb infrared light from the earth and reemit it back to the earth,
preventing energy from escaping.

1 52

© Saunders College Publishing


2.48 one at a tertiary, two at 6econdary carbons
2.50

(d)

(c) �
CH3

CH3

2.52 ( 1 ) Br-CH-CH2CH3 1 , I -dibromopropane


I
Br
(2) Br-CH2-CH-CH3 1 ,2-dibromopropane
I
Br
(3) Br-CH2CH2CH2-Br 1,3 -dibromopropane
(4) Br
I
CH3-C-CH3 2,2-dibromopropane
I
Br
2.54 There is a much greater percentage increase in molecular weight in going from C 1
to C2 or from C2 to C3 than from Cs to C9 or from C9 to C1 0•
2.56 for example: (a) CH3CH2CH2CH2CH2-0H
(b) CH3-CH2-C-OH (c) CH3-C-CH3 (d) CH3-O-CH2CH2CH3
II II
o 0

(e)
o- OH

1 53
© Saunders CoUege Publishing
2.58 ( 1 ) CH3(CH2)sCH3 heptane
(2) CH3-CH-CH2CH2CH2CH3 2-methylhexane
I
CH3
(3) CH3CH2-CH-CH2CH2CH3 3 -methylhexane
I
CH3
(4) CH3
I
CH3-C-CH2CH2CH3 2,2-dimethylpentane
I
CH3
(5) CH3-CH-CH-CH2CH3 2,3 -dimethylpentane
I I
CH3 CH3
(6) CH3-CH-CH2-CH-CH3 2,4-dimethylpentane
I I
CH3 CH3
(7) CH3
I
CH3CH2-C-CH2CH3 3,3 -dimethylpentane
I
CH3
(8) CH3CH2-CH-CH2CH3 3 -ethylpentane
I
I
CH2
CH3
(9)
2,2,3 -trimethylbutane

1 54

© Saunders CoUege Publishing

Das könnte Ihnen auch gefallen